You are on page 1of 122

Назва наукового напрямку (модуля): Семестр: 11

КРОК 2
Опис:
6 course 11-12 semestr
Перелік питань:
1. A 28-year-old patient has been taken to a hospital withe acute pain in the lower abdomen. There
was a brief syncope. The delay of menstruation is 2 months. Objectively: the patient has pale skin,
BP- 90/50 mm Hg, Ps- 110/min. Lower abdomen is extremely painful. Vaginal examination reveals
uterus enlargement. There is positive Promtov’s sign. Right adnexa are enlarged and very painful.
What is the most likely diagnosis?
A. * Right – sided tubal pregnancy
B. Right ovary apoplexy
C. Acute right-sided salpingoophoritis
D. Pelvioperitionitis
E. Incipient abortion
2. A 58-year-old female patient came to the antenatal clinic complaining of bloody light-red
discharges from the genital tracts. Menopause is 12 years. Gynaecological examination revealed
age involution of externalia and vagina; uterine cervix was unchanged, there were scant bloody
discharges from uterine cervix, uterus was of normal size; uterine adnexa were not palpable;
parametria were free. What is the most likely diagnosis?
A. * Uterine carcinoma
B. . Atrophic colpitic
C. Abnormalities of menstrual cycle of climacteric nature
D. Cervical carcinoma
E. Granulosa cell tumor of ovary
3. Full-term pregnancy. Body weight of the pregnant woman is 62 kg. The fetus has the longitudinal
lie, the fetal head is engaged to pelvic inlet. Abdominal circumference is 100 cm. Fundal height is
35 cm. What is the approximate weight of the fetus?
A. * 3kg 500 g
B. 4 kg
C. 2 kg 500 g
D. 3 kg
E. 4 kg 500 g
4. A newborn’s head is of dolichocephalic shape, that is front-to-back elongated. Examination of the
occipital region revealed a labour tumour located in the middle between the prefontanel and
posterior fontanel. Specify the type of fetal presentation:
A. * Posterior vertex presentation
B. Anterior vertex presentation
C. Presentation of the bregma
D. Brow presentation
E. Face presentation
5. A 30-year-old multigravida has been in labour for 18 hours. 2 hours ago pushing stage began. Fetal
heart rate is clear, rhythmic, 136/min. Vaginal examination reveals the complete cervical dilatation,
the fetal head is in the pelvic outlet. Sagittal suture corresponds with anteroposteror diemeter of
outlet the occipital fontanel is near the pubis. The patient has been diagnosed with primary uterine
inertia. What is the further tactics of labour management?
A. * Outlet forceps
B. Labour stimulation
C. Cesarean section
D. Skin- head Ivanov`s forceps
E. Tsovianov method
6. A baby was born by a young smoker. The labour was complicated by uterine inertia, difficult
delivery of the baby’s head and shoulders. The baby’s Apgar score was 4. Which of the following is
a risk factor for a spinal cord injury?
A. * Diffiult delivery of the head and shoulders
B. Young age of the mother
C. Pernicious habits
D. Uterine inertia
E. Chronic hypoxia
7. A patient complains of being unable to get pregnant for 5 years. A complete clinical examination
gave the following results: hormonal function is not impaired, urogenital infection hasn’t been
found, on hysterosalpingography both tubes were filled with the contrast medium up to the isthmic
segment, abdominal contrast was not visualized. The patient’s husband is healthy. What tactics will
be most effective?
A. * In-vitro fertilization
B. Insemination with husband`s sperm
C. ICSI within in-vitro fertilization program
D. Hydrotubation
E. Laperascopic tubal plasty
8. A 55-year-old patient whose menstruation stopped 5 years ago complains of vaginal dryness,
frequent and painful urination. Gynecologist revealed signs of atrophic colpitis. Urine analysis
revealed no peculiarities. Which locally acting product will provide the proper therapeutic effect?
A. * Vaginal suppositories “Ovestin”
B. Vaginal tablets “Tergynan”
C. Vaginal cream “Meratin Combi”
D. Vaginal gel “Metronidazole”
E. Vaginal cream “Dalacin”
9. A 49-year-old patient complains of itching, burning in the external genitals, frequent urination. The
symptoms has been present for the last 7 months. The patient has irregular menstruation, once
every 3-4 months. Over the last 2 years she presents with hot flashes, sweating, sleep disturbance.
Examination revealed no pathological changes of the internal reproductive organs. Complete blood
count and urinalysis showed no pathological changes. Vaginal smear contained 20-25 leukocytes in
the field of vision, mixed flora. What is the most likely diagnosis?
A. * Menopauzual syndrome
B. Cystitis
C. Trichomonas colpitis
D. Vulvitis
E. Bacterial vaginosis
10. Examination of a Rh-negative pregnant woman at 32 weeks of gestation revealed a four-time rise
of Rh-antibody titer within 2 weeks, the titer was 1:64. In the first two pregnancies the patient had
experienced antenatal fetal death due to hemolytic disease. What is the optimal tactics of pregnancy
management?
A. * Early delivery
B. delivery at 37 weeks of gestation
C. Screening for Rh-antibodies 2 weeks later and early delivery in case of further titer rise
D. Introdution of anti-Rh (D) immunoglobulin
E. Ultrasound for signs of hemolytic disease of the fetus
11. 3 months after the first labor a 24- year-old patient complained of amenorrhea. Pregnancy ended by
Caesarian section because of premature separation of normally located placenta. Blood loss is
2000 ml due to disturbance of blood clotting. Choose the most suitable investigation:
A. * Estimation of gonadotropin level
B. US of small pelvis
C. Progesteron assay
D. Computer tomography of head
E. Estimation of testosteron rate in blood serum
12. A 24-year-old primipara was hospitalised with complaints of discharge of the amniotic waters. The
uterus is tonic on palpation. The position of the fetus is longitudinal, it is pressed with the head to
pelvic outlet. Palpitation of the fetus is rhythmical, 140 bpm, auscultated on the left below the
navel. Internal examination: cervix of the uterus is 2,5 cm long, dense, the external opening is
closed, light amniotic waters are discharged. Point out the correct component of the diagnosis:
A. * Antenatal discharge of the amniotic waters
B. Early discharge of the amniotic waters
C. The beginning of the 1st stage of labour
D. The end of the 1st stage of labour
E. Pathological preterm labour
13. A 59-year-old female patient admitted to the maternity clinic with complains of bloody
discharge from the vagina. Menopause is 12 years. Speculum examination: uterus cervix was not
erosive, small amount of bloody discharge came from the cervical canal. Uterus is of normal size,
adnexa are not palpable. Fornices were deep and painless. What method should be applied for the
diagnosis specification?
A. * Diuatation and curatage
B. Laparoscopy
C. Culdocentesis
D. Extensive colposcopy
E. Culdoscopy
14. A 28-year-old woman has bursting pain in the lower abdomen during menstruation; chocolate-like
discharges from vagina are observed. It is known from the anamnesis that the patient suffers from
chronic adnexitis. Bimanual examination revealed a tumour-like formation of heterogenous
consistency 7х7 cm large to the left from the uterus. The formation is restrictedly movable, painful
when moved. What is the most probable diagnosis?
A. * Endometrioid cyst of the left ovary
B. Follicular cyst of the left ovary
C. Fibromatous node
D. Exacerbation of chronic adnexitis
E. Tumour of sigmoid colon
15. A 30-year-old parturient woman was delivered to a maternity hospital with full-term pregnancy.
She complains of severe lancinating pain in the uterus that started 1 hour ago, nausea, vomiting,
cold sweat. Anamnesis states cesarean section 2 years ago. Uterine contractions stopped. Skin and
mucous membranes are pale. Heart rate is 100/min, BP is 90/60 mm Hg. Uterus has no clear
margins, is sharply painful. No heartbeat can be auscultated in the fetus. Moderate bloody discharge
from the uterus can be observed. Uterus cervix is 4 cm open. Presenting part is not visible. The
most likely diagnosis is:
A. * Uterine rupture
B. Initial uterine rupture
C. Threatened uterine rupture
D. Premature detachment of normally positioned placenta
E. Compression of inferior pudendal vein

16. A 26-year-old woman has attended maternity center complaining of her inability to become
pregnant despite 3 years of regular sex life. Examination revealed the following: increased body
weight; male-type pubic hair; excessive pilosis of thighs; ovaries are dense and enlarged; basal
body temperature is monophasic. The most likely diagnosis is:
A. * Ovaries sclerocystosis
B. Inflammation of uterine appendages
C. Adrenogenital syndrome
D. Premenstrual syndrome
E. Gonadal dysgenesis
17. 28-year-old woman complains of increased intermenstrual periods up to 2 months, hirsutism.
Gynaecological examination revealed that the ovaries were enlarged, painless, compact, uterus had
no pecularities. Pelvic ultrasound revealed that the ovaries were 4-5 cm in diameter and had
multiple enlarged follicles on periphery. X-ray of skull base showed that sellar region was dilated.
What is the most probable diagnosis?
A. * Stein-Leventhal syndrome (Polycystic ovary syndrome)
B. Algodismenorrhea
C. Sheehan’s syndrome
D. Premenstrual syndrome
E. Morgagni-Stewart syndrome
18. A woman consulted a therapeutist about fatigability, significant weight loss, weakness, loss of
appetite. She has been having amenorrhea for 8 months. A year ago she born a full-term child.
Haemorrhage during labour made up 2 l. She got blood and blood substitute transfusions. What is
the most probable diagnosis?
A. * Sheehan’s syndrome
B. Stein-Leventhal syndrome
C. Shereshevsky-Turner’s syndrome
D. Homological blood syndrome
E. Vegetovascular dystonia
19. A parturient woman is 27 year old, it was her second labour, delivery was at full-term, normal
course. On the 3rd day of postpartum period body temperature is 36,8 0C, heart rate - 72/min, BP -
120/80 mm Hg. Mammary glands are moderately swollen, nipples are clean. Abdomen is soft and
painless. Fundus of uterus is 3 fingers below the umbilicus. Lochia are bloody, moderate. What is
the most probable diagnosis?
A. * Physiological course of postpartum period
B. Subinvolution of uterus
C. Postpartum metroendometritis
D. Remnants of placental tissue after labour
E. Lactostasis
20. A 37-year-old woman complains of sharp pains in her external genitalia, edema of the vulvar lips,
pain when walking. Objectively: body temperature is 38, 7 0C, heart rate is 98/min. Inside the right
lover part of labic major there is a dense, painful, tumor-like growth 5,0х4,5 cm in size; skin and
mucosa of the external genitalia are hyperemic, copious foul-smelling discharge is observed. The
most likely diagnosis is:
A. * Acute bartholinitis
B. Furuncle of outer labia
C. Acute vulvovaginitis
D. Bartholin’s cyst
E. Carcinoma of vulva

21. Examination of placenta revealed a defect. An obstetrician performed manual investigation of


uterine cavity, uterine massage. Prophylaxis of endometritis in the postpartum period should
involve the following actions:
A. * Antibacterial therapy
B. Instrumental revision of uterine cavity
C. Haemostatic therapy
D. Contracting agents
E. Intrauterine instillation of dioxine
22. On the 10th day postpartum a puerperant woman complains of pain and heaviness in the left
mammary gland. Body temperature is 38, 8 0C, Ps- 94 bpm. The left mammary gland is edematic,
the upper external quadrant of skin is hyperemic. Fluctuation symptom is absent. The nipples
discharge drops of milk when pressed. What is a doctor’s further tactics?
A. * Antibiotic therapy, immobilization and expression of breast milk
B. Compress to both mammary glands
C. Inhibition of lactation
D. Physiotherapy
E. Opening of the abscess and drainage of the mammary gland 180
23. A 25-year-old patient during selfexamination detected a tumor in the upper external quadrant of the
right mammary gland. On palpation: painless, dense, mobile growth 2 cm in diameter is detected in
the mammary gland; no changes in the peripheral lymph nodes are observed. On mammary glands
US: in the upper external quadrant of the right mammary gland there is a space-occupying lesion of
increased echogenicity 21х18 mm in size. The most likely diagnosis is:
A. * Fibrous adenoma
B. Lacteal cyst
C. Diffuse mastopathy
D. Breast cancer
E. Mastitis
24. A maternity patient breastfeeding for 1,5 weeks has attended a doctor. She considers the onset of
her disease to be when proportional breast engorgement occurred. Mammary glands are painful.
Body temperature is 36, 6 0C. Expression of breast milk is hindered. The most likely diagnosis is:
A. * Lactostasis
B. Infiltrative mastitis
C. Suppurative mastitis
D. Chronic cystic mastitis
E. Gangrenous mastitis
25. An Rh-negative woman with 32-weeklong term of pregnancy has been examined. It was observed
that Rh-antibodies titer had increased four times within the last 2 weeks and was 1:64. First two
pregnancies ended in antenatal death of fetus caused by hemolytic disease. What tactics of
pregnancy management should be chosen?
A. * Preterm delivery
B. Delivery at 37 weeks term
C. Rh-antibody test in 2 weeks; if Rh-antibodies increase in number conduct delivery
D. Introduction of anti-Rh immunoglobulin
E. US examination to determine signs of fetal erythroblastosis
26. A 48-year-old patient was delivered to a hospital in-patient unit with uterine bleeding that occurred
after the 2-week-long delay of menstruation. Anamnesis states single birth. Examination of the
uterine cervix with speculum mirrors revealed no pathologies. On bimanual examination: uterus is
of normal size, painless, mobile; uterine appendages have no changes. Discharge is bloody and
copious. What primary hemostatic measure should be taken in the given case?

A. * Fractional curettage of uterine cavity


B. Hormonal hemostasis
C. Hemostatics
D. Uterine tamponade
E. Uterotonics
27. A 30-year-old woman complains of infertility during her 10-year-long married life. Menstruations
occur since she was 14 and are irregular, with delays up to a month and longer. Body mass is
excessive. Hirsutism is observed. On bimanual examination: uterine body is decreased in size;
ovaries are increased in size, dense, painless, and mobile. The most likely diagnosis is:
A. * Stein–Leventhal syndrome (Polycystic ovary syndrome)
B. Follicular cyst of ovaries
C. Genital endometriosis
D. Genital tuberculosis
E. Inflammatory tumor of ovaries
28. .A 50-year-old female patient complains of aching pain in the lower abdomen. She has a history of
normal menstrual cycle. At the age of 40, the patient underwent a surgery for gastric ulcer.
Examination findings: abdomen is soft, in the hypogastrium there is a well-defined nodular tumor
of limited mobility. Vaginal examination findings: the cervix is clean, of cylindrical shape. Body of
the uterus cannot be palpated separately. On both sides of the uterus palpation reveals tight tumors
with an uneven surface. The tumors are immobile andl fill the whole pelvic cavity. What is the most
likely diagnosis?
A. * Krukenberg tumor
B. Ovarian fibroid
C. Ovarian granulose cell tumor
D. Bilateral pioovarium
E. Subserous metrofibrioma
29. A 13-year-old girl was admitted to the gynecology department for having a significant bleeding
from the vagina for 10 days. The patient has a history of irregular menstrual cycle since menarche.
Menarche occurred at the age of 11. Recto-abdominal examination revealed no pathology. What is
the provisional diagnosis?
A. * Juvenile uterine bleeding
B. Adenomyosis
C. Injury of the external genitalia
D. Werlhof’s disease
E. Endometrial polyp
30. A 21-year-old female patient consulted a gynecologist about itching, burning, watery vaginal
discharges with a fish-like smell. Speculum examination revealed that the cervical and vaginal
mucosa was of a normal pink color. Vaginal examination revealed no alterations of the uterus and
adnexa. Gram-stained smears included clue cells. What is the most likely pathology?
A. * Bacterial vaginosis (gardnerellosis)
B. Chlamydiosis
C. Gonorrhea
D. Trichomoniasis
E. Candidiasis
31. A 26-year-old secundipara at 40 weeks of gestation arrived at the maternity ward after the
beginning of labor activity. 2 hours before, bursting of waters occurred. The fetus was in a
longitudinal lie with cephalic presentation. Abdominal circumference was 100 cm, fundal height -
42 cm. Contractions occurred every 4-5 minutes and lasted 25 seconds each. Internal obstetric
examination revealed cervical effacement, opening by 4 cm. Fetal bladder was absent. Fetal head
was pressed against the pelvic inlet. What complication arose in childbirth?

A. * Early amniorrhea
B. Primary uterine inertia
C. Secondary uterine inertia
D. Discoordinated labor
E. Clinically narrow pelvis
32. A 39-year-old female patient complains on dyspnea when walking, palpitation, edemata in the
evening. The patient’s height is 164 cm, weight - 104 kg. Objectively: overnutrition. Heart sounds
are weak, and tachycardia is present. The menstrual cycle is not broken. Blood sugar is 5,6 mmol/l,
ACTH-response tests revealed no alterations. X-ray of the Turkish saddle revealed no pathology.
What disease is it?
A. * Alimentary obesity
B. Climax
C. Pituitary obesity
D. Diabetes mellitus
E. Cushing’s syndrome (primary hypercorti-solism)
33. A 28-year-old female patient has been admitted to the gynecology department for abdominal pain,
spotting before and after menstruation for 5 days. The disease is associated with the abortion which
she had 2 years ago. Antiinflammatory treatment had no effect. Bimanual examination findings: the
uterus is enlarged, tight, painful, smooth. Hysteroscopy reveals dark red holes in the fundus with
dark blood coming out of them. What diagnosis can be made on the grounds of these clinical
presentations?
A. * Internal endometriosis
B. Polymenorrhea
C. Hypermenorrhea
D. Submucous fibromatous node
E. Dysfunctional uterine bleeding
34. A woman at 30 weeks pregnant has had an attack of eclampsia at home. On admission to the
maternity hospital BP- 150/100 mm Hg. Estimated fetal weight is 1500 g. There is face and shin
edema. Urine protein is 0,66g /l. Uterine cervix is 5 poin by Bishop scale. An intensive complex
therapy has been started. What is the correct tactics of this case management?
A. * Delivery by cesarean section
B. Continue therapy and prolong pregnancy for 1-2 weeks
C. Continue therapy and prolong pregnancy for 3-4 weeks
D. Labor induction by intravenous oxytocin or prostaglandins
E. Treat preeclampsia and achieve the delivery by way of conservative management
35. A pregnant 26-year-old woman was admitted to a hospital for abdominal pain and bleeding from
the genital tract. Bimanual examination revealed that uterus was the size of 9 weeks of pregnancy,
the cervical canal let a finger through. Fetal tissues could be palpated in the orifice. There was
moderate vaginal bleeding. What is the tactics of choice?
A. * Instrumental extraction of fetal tissue
B. Surveillance
C. Administration of hormones
D. Hemostatic and antianemic therapy
E. Therapy for the maintenance of pregnancy
36. A 36-year-old female pesented to a gynecological hospital with a significant bleeding from the
genital tract and a 1-month delay of menstruation. Bimanual examination revealed soft
barrel-shaped cervix. Uterus was of normal size, somewhat softened. Appendages were
unremarkable on both sides. Speculum examination revealed that the cervix was cyanotic, enlarged,
with the the external orifice disclosed up to 0,5 cm. Urine hCG test was positive. What is the most
likely diagnosis?

A. * Cervical pregnancy
B. Uterogestation
C. Abortion in progress
D. Threatened miscarriage
E. Ectopic pregnancy
37. An 18-year-old girl complains of breast pain and engorgement, headaches, irritability, swelling of
the lower extremities. These symptoms have been observed since menarche and occur 3-4 days
before the regular menstruation. Gynecological examination revealed no pathology. Make a
A. * diagnosis:
Premenstrual syndrome
B. Neurasthenia
C. Renal disease
D. Mastopathy
E. Cardiovascular disorder
38. A 25-year-old female presented to a women’s welfare clinic and reported the inability to get
pregnant within 3 years of regular sexual activity. Examination revealed increased body weight,
male pattern of pubic hair growth, excessive pilosis of thighs, dense enlarged ovaries, monophasic
basal temperature. What is the most likely diagnosis?
A. * Polycystic ovarian syndrome
B. Adnexitis
C. Adrenogenital syndrome
D. Premenstrual syndrome
E. Gonadal dysgenesis
39. A 23-year-old female consulted a gynecologist on the 20th day postpartum period about pain in the
left breast, purulent discharge from the nipple. Objectively: Ps-120/min, t - 39 0C . The left breast is
painful, larger than the right one, hyperemic. In the upper quadrant there is an infiltrate sized 10x15
cm with a softening inside. Blood test results: ESR- 50 mm/h, WBC- 15, 0 109/l. What is the
tactics of choice?
A. * Refer to the surgical department for operati-ve treatment
B. Refer to the gynecology department
C. Refer to the postpartum department
D. Refer to a polyclinic surgeon for conservati-ve treatment
E. Lance the breast abscess in the women’s health clinic
40. A puerperant is 28 years old. It’s the 3rd day post-partum after a second, normal, term delivery. The
body temperature is of 36, 8 0 C , Ps- 72/min, BP- 120/80 mm Hg. Mammary glands are moderately
engorged, the nipples are clean. Abdomen is soft, painless. The fundus is 3 fingers’ breadth below
the umbilicus. Moderate bloody lochia are present. What diagnosis can be made?
A. * Physiological course of the postpartum period
B. Subinvolution of uterus
C. Postpartum metroendometritis
D. Remains of placental tissue after childbirth
E. Lactostasis
41. A puerperant is 32 years old, it’s her first childbirth, term precipitate labor, the III period is
unremarkable, the uterus is contracted, tight. Examination of the birth canal revealed a rupture in
the left posterior vaginal wall that was gutured by catgut. Two hours later, the patient complained
on a feeling of pressure on the anus, pain in the perineum, minor vaginal discharges, edema of the
vulva. These clinical presentations are indicative most likely of:
A. * Vaginal hematoma
B. Hysterocervicorrhexis
C. Hemorrhoids

D. Hysterorrhesis
E. Hypotonic bleeding
42. A 31-year-old female patient complains of infertility, amenorrhea for 2 years after the artificial
abortion that was complicated by endometritis. Objective: examination of the external genitalia
reveals no pathology, there is female pattern of hair distribution. According to the functional tests,
the patient has biphasic ovulatory cycle. What form of infertility is the case?
A. * Uterine
B. Ovarian
C. Pituitary
D. Hypothalamic
E. Immunological
43. A female patient complains of being unable to get pregnant for 5 years. A complete clinical
examination brought the following results: hormonal function is not impaired, urogenital infection
hasn’t been found, on hysterosalpingography both tubes were filled with the contrast medium up to
the isthmic segment, abdominal contrast was not visualized. The patient’s husband is healthy. What
tactics will be most effective?
A. * In-vitro fertilization
B. Insemination with husband’s sperm
C. ICSI within in-vitro fertilization program
D. Hydrotubation
E. Laparoscopic tubal plasty
44. A 19-year-old primiparous woman with a body weight of 54,5 kg gave birth at 38 weeks gestation
to a full-term live girl after a normal vaginal delivery. The girl’s weight was 2180,0 g, body length -
48 cm. It is known from history that the woman has been a smoker for 8 years, and kept smoking
during pregnancy. Pregnancy was complicated by moderate vomiting of pregnancy from 9 to 12
weeks pregnant, edemata of pregnancy from 32 to 38 weeks. What is the most likely cause of low
birth weight?
A. * Fetoplacental insufficiency
B. Low weight of the woman
C. Woman’s age
D. First trimester preeclampsia
E. Third trimester preeclampsia
45. A 23-year-old primigravida at 39 weeks gestation has been admitted to the maternity ward with
irregular contractions. The intensity of uterine contractions is not changing, the intervals between
them stay long. Bimanual examination reveals that the cervix is centered, soft, up to 1,5 cm long.
There is no cervical dilatation. What diagnosis should be made?
A. * Pregnancy I, 39 weeks, preliminary period
B. Pregnancy I, 39 weeks, labor I, 1 period, the latent phase
C. Pregnancy I, 39 weeks, labor I, period 1, the active phase
D. Pregnancy I, 39 weeks, birth I, 1 period, the acceleration phase
E. Pregnancy I, 39 weeks, pathological prelimi-nary period
46. On the 10th day postpartum a puerperant woman complains of pain and heaviness in the left breast.
Body temperature is 38, 8 0C , Ps- 94 bpm. The left breast is edematic, the supero-external quadrant
of skin is hyperemic. Fluctuation symptom is absent. The nipples di-scharge drops of milk when
pressed. What is a doctor’s further tactics?
A. * Antibiotic therapy, immobilization and expression of breast milk
B. Compress to both breasts
C. Inhibition of lactation
D. Physiotherapy

E. Opening of the abscess and drainage of the breast


47. During the breast self-exam a 37-year-old female patient revealed a lump in the lower inner
quadrant of her left breast. Palpation confirms presence of a mobile well-defined neoplasm up to 2
cm large. Peripheral lymph nodes are not changed. What is the way of further management?
A. * Ultrasound examination, mammography, fine-needle aspiration bi-opsy
B. Anti-inflammatory therapy, physiotherapy
C. Radical mastectomy
D. Ultrasound monitoring of genitals during the entire course of antiestrogens therapy, systemic
enzyme therapy, phytotherapy
E. Case follow-up
48. A 25-year-old female has a self-detected tumor in the upper outer quadrant of her right breast. On
palpation there is a painless, firm, mobile lump up to 2 cm in diameter, peripheral lymph nodes are
not changed. In the upper outer quadrant of the right breast ultrasound revealed a massive neoplasm
with increased echogenicity sized 21x18 mm. What is the most likely diagnosis?
A. * Fibroadenoma
B. Lactocele
C. Diffuse mastopathy
D. Mammary cancer
E. Mastitis
49. On dmission a 35-year-old female reports acute abdominal pain, fever up to 38, 8 0C, mucopurulent
discharges. The patient is nulliparous, has a history of 2 artificial abortions. The patient is
unmarried, has sexual contacts. Gynecological examination reveals no uterus changes. Appendages
are enlarged, bilaterally painful. There is profuse purulent vaginal discharge. What study is required
to confirm the diagnosis?
A. * Bacteriologic and bacteriascopic studies
B. Hysteroscopy
C. Curettage of uterine cavity
D. Vaginoscopy
E. Laparoscopy
50. A 20-year-old female consulted a gynecologist about not having menstrual period for 7 months.
History abstracts: early childhood infections and frequent tonsillitis, menarche since 13 years,
regular monthly menstrual cycle of 28 days, painless menstruation lasts 5-6 days. 7 months ago the
patient had an emotional stress. Gynecological examination revealed no alterations in the uterus.
What is the most likely diagnosis?
A. * Secondary amenorrhea
B. Primary amenorrhea
C. Algomenorrhea
D. Spanomenorrhea
E. Cryptomenorrhea
51. A 48-year-old female has been admitted to the gynecology department for pain in the lower right
abdomen and low back pain, constipations. Bimanual examination reveals: the uterus is immobile,
the size of a 10-week pregnancy, has irregular surface. Aspirate from the uterine cavity contains
atypical cells. What is the most appropriate diagnosis?
A. * Hysterocarcinoma
B. Cervical cancer
C. Metrofibroma
D. Colon cancer
E. Chorionepithelioma

52. A pregnant woman is 28 years old. Anamnesis: accelerated labor complicated by the II degree
cervical rupture. The following two pregnancies resulted in spontaneous abortions at the terms of
12 and 14 weeks. On speculum examination: the uterine cervix is scarred from previous ruptures at
9 and 3 hours, the cervical canal is gaping. On vaginal examination: the cervix is 2 cm long, the
external orifice is open 1 cm wide, the internal orifice is half-open; the uterus is enlarged to the
12th week of pregnancy, soft, mobile, painless, the appendages are without changes. What
diagnosis would you make?
A. * Isthmico-cervical insufficiency, habitual noncarrying of pregnancy
B. Threatened spontaneous abortion
C. Incipient abortion, habitual noncarrying of pregnancy
D. Cervical hysteromyoma, habitual noncarryi-ng of pregnancy
E. Cervical pregnancy, 12 weeks
53. An infant has been born at the 41st week of gestation. The pregnancy was complicated with severe
gestosis of the second semester. The weight of the baby is 2400 g, the height is 50 cm. Objectively:
the skin is flabby, the layer of subcutaneous fat is thin, hypomyotonia, neonatal reflexes are weak.
The internal organs are without pathologic changes. This newborn can be estimated as a:
A. * Full-term infant with prenatal growth retardation
B. Premature infant
C. Immature infant
D. Postmature infant
E. Full-term infant with normal body weight
54. A full term baby born from the 1st noncomplicated pregnancy with complicated labor was
diagnosed with cephalohematoma. On the 2nd day of life the child developed jaundice; on the 3rd
day of life there appeared neurological changes: nystagmus, Graefe syndrome. Urine is yellow,
feces are golden-yellow. The mother’s blood group is А (II) Rh-, the child’s - А (II) Rh+. On the
3rd day the results of the child’s blood test are as follows: Hb- 200 g/l, erythrocytes - 6, 1 1012/l,
blood bilirubin - 58 mcmol/l due to the presence of its unconjugated fraction, Ht- 0,57. In this case
the jaundice is caused by:
A. * Craniocerebral birth injury
B. Physiologic jaundice
C. Hemolytic disease of newborn
D. Atresia of bile passages
E. Fetal hepatitis
55. A 26-year-old woman, who gave birth 7 months ago, has been suffering from nausea, morning
sickness, somnolence for the last 2 weeks. The patient breasfeeds; no menstruation. She has been
using no means of contraception. What method would be most efficient in clarification of the
diagnosis?
A. * Ultrasound
B. Small pelvis radiography
C. Palpation of mammary glands and squeezi-ng out colostrum
D. Bimanual abdominovaginal examination
E. Mirror examination
56. A 30-year-old parturient woman was deli-vered to a maternity hospital with full-term pregnancy.
She complains on severe lancinating pain in the uterus that started 1 hour ago, nausea, vomiting,
cold sweat. Anamnesis states cesarean section 2 years ago. Uterine contractions stopped. Skin and
mucous membranes are pale. Heart rate is 100/min., BP is 90/60 mm Hg. Uterus has no clear
margins, is sharply painful. No heartbeat can be auscultated in the fetus. Moderate bloody discharge
from the uterus can be observed. Uterus cervix is 4 cm open. Presenting part is not visible. The
most likely diagnosis is:
A. * Uterine rupture

B. Initial uterine rupture


C. Threatened uterine rupture
D. Premature detachment of normally positi-oned placenta
E. Compression of inferior pudendal vein
57. During the dynamic observation of a parturient woman in the second stage of labor it was
registered that the fetal heart rate decreased to 90-100/min. and did not normali-ze after
contractions. Vaginal examination revealed the complete cervical dilatation, the fetal head filling
the entire posterior surface of the pubic symphysis and sacral hollow; the sagittal suture was in the
anteroposteri-or diameter of the pelvic outlet, the posterior fontanelle was in front under the pubic
arch. What plan for further labour management should be recommended?
A. * Application of forceps minor
B. Caesarean section
C. Episiotomy
D. Application of cavity forceps
E. Stimulation of labour activity through intravenous injection of oxytocin
58. A 16-year-old girl has primary amenorrhea, no pubic hair growth, normally developed mammary
glands; her genotype is 46 ХY; uterus and vagina are absent. What is your diagnosis?
A. * Testicular feminization syndrome
B. Mayer- Rokitansky- Kuster-Hauser syndrome
C. Cushing’s syndrome
D. Sheehan syndrome
E. Cushing’s disease
59. A 27-year-old sexually active woman complains of numerous vesicles on the right sex lip, itch and
burning. Eruptions regularly appear before menstruation and disappear 8-10 days later. What is the
most likely di-agnosis?
A. * Herpes simplex virus
B. Bartholinitis
C. Primary syphilis
D. Cytomegalovirus infection
E. Genital condylomata
60. A 35-year-old woman addressed a gynecological in-patient department with complaints of regular
pains in her lower abdomen, which increase during menstruation, and dark-brown sticky discharge
from the genital tracts. On bimanual examination: the uterine body is slightly enlarged, the
appendages are not palpated. Speculum examination of the uterine cervix reveals bluish spots.
What diagnosis is most likely?
A. * Cervical endometriosis
B. Cervical erosion
C. Cervical polyp
D. Cervical cancer
E. Cervical fibroid
Назва наукового напрямку (модуля): Семестр: 11
Акушерство та гінекологія Тести
Опис:
6 курс
Перелік питань:
1. What is the most appropriate tactic in incomplete abortion?
A. Hemostatic drugs
B. Blood transfusion
C. Spasmolytic
D. * A dilatation and curettage (D&C)
E. Bed rest and Vitamin supplements
2. What is middle weight of uterus at once after labor?
A. 100-200 gr
B. 300-400 gr
C. 500-600 gr
D. 700-800 gr
E. * 900-1000 gr
3. Duration of early postpartum period is:
A. 30 min
B. 3 days
C. 12 days
D. * 2 hours
E. 6-8 weeks
4. Lochia rubra consist of all the following EXEPT:
A. blood
B. shreds of the membranes
C. * parts of placenta
D. decidual membrane
E. erythrocytes
5. Lochia alba consist of all the following EXEPT:
A. * blood
B. mucus
C. leucocytes
D. decidual cells
E. erythrocytes
6. For breast proliferation response:
A. prolactin
B. luteinized hormone
C. * estrogens
D. prostaglandins
E. corticosteroids
7. What complication in the future would develop due to the incorrect suturing of cervical laceration?
A. * cervical ectropion;
B. violation of function of pelvic muscles;
C. prolapse of uterus;
D. endometritis;
E. bleeding.
8. Routine postpartum care should include searching for all of the following EXCEPT:
A. Fever
B. * Diabetes
C. Depression
D. Anemia
E. Thrombophlebitis
9. What is the minimal physiologic blood loss in labor?
A. 100 ml
B. * 250 ml
C. 300 ml
D. 350 ml
E. 450 ml
10. What is the management of the total placenta percreta?
A. * total hysterectomy
B. manual separation and removal of the placenta
C. hysterotomy
D. subtotal hysterectomy
E. ligation of the vessels
11. What is the management of the partial placenta percreta?
A. * total hysterectomy
B. manual separation and removal of the placenta
C. hysterotomy
D. subtotal hysterectomy
E. ligation of the vessels
12. What is the management of the focal placenta percreta?
A. * total hysterectomy
B. manual separation and removal of the placenta
C. hysterotomy
D. subtotal hysterectomy
E. ligation of the vessels
13. Which highest dose of oxytocin is prescribed in the uterine atony?
A. 5 units
B. 10 units
C. 15 units
D. * 20 units
E. 25 units
14. Which dose of misoprostol is prescribed in the uterine atony?
A. * 800 mkg
B. 600 mkg
C. 400 mkg
D. 200 mkg
E. 100 mkg
15. What is the blood loss in the second degree of hemorrhagic shock?
A. 500-750ml
B. 750-1000ml
C. * 1000-1500ml

D. 1500-1800ml
E. >1800ml
16. What is the blood loss in the third degree of hemorrhagic shock?
A. 500-750ml
B. 750-1000ml
C. * 1500-2500ml
D. 1000-1200ml
E. 1200-1300ml
17. What is the blood loss in the fourth degree of hemorrhagic shock?
A. 500-750ml
B. 750-1000ml
C. 1500-2000ml
D. 2000-2200ml
E. * >2500ml
18. All of the above are the main causes of disseminated intravascular clotting (DIC) syndrome
EXCEPT:
A. Placenta abruption
B. Embolic fluid embolism
C. * Chronic bronchitis
D. Pregnancy induced hypertension
E. Hypotonic bleeding
19. What is the volume of infusion therapy in the mild degree of hemorrhagic shock ?
A. 500 ml
B. 1000 ml
C. 1500 ml
D. 2000 ml
E. * 2500 ml
20. Which dose of colloids is recommended in the mild degree of hemorrhagic shock?
A. 5 ml / kg
B. * 10 ml / kg
C. 15 ml / kg
D. 20 ml / kg
E. 25 ml/kg
21. All of the below belong to the widely used colloids in obstetrics EXCEPT:
A. Refortan
B. Gelofusin
C. * Ringer-Lokka
D. Stabisol
E. Fresh-frozen plasma
22. What is the volume of infusion therapy in the moderate degree of hemorrhagic shock?
A. 1000 ml
B. 1500 ml
C. 2000 ml
D. 2500 ml
E. * 3000 ml
23. What is the volume of infusion therapy in the severe degree of hemorrhagic shock?

A. 1500 ml
B. 2000 ml
C. 2500 ml
D. 3000 ml
E. * 4000 ml
24. What is the first stage of Disseminated Intravascular Clotting syndrome?
A. hypocoagulation without generalizing fibrinolysis
B. * hypercoagulation
C. hypocoagulation with generalizing fibrinolysis
D. total fibrinolysis
E. local fibrinolysis
25. What is the second stage of Disseminated Intravascular Clotting syndrome?
A. * hypocoagulation without generalizing fibrinolysis
B. hypercoagulation
C. hypocoagulation with generalizing fibrinolysis
D. total fibrinolysis
E. local fibrinolysis
26. What is the third stage of Disseminated Intravascular Clotting syndrome?
A. hypocoagulation without generalizing fibrinolysis activation
B. hypercoagulation
C. * hypocoagulation with generalizing fibrinolysis activation
D. total fibrinolysis
E. local fibrinolysis
27. What is the fourth stage of Disseminated Intravascular Clotting syndrome?
A. hypocoagulation without generalizing fibrinolysis activation
B. hypercoagulation
C. hypocoagulation with generalizing fibrinolysis activation
D. * total fibrinolysis
E. Local fibrynolysis
28. All of the below are the main signs of Disseminated Intravascular Clotting syndrome EXCEPT:
A. Hemorrhages into skin and mucous membranes
B. Hemorrhages from the places of injections, incisions, uterus
C. Necrosis of some areas of skin and mucous membranes
D. * Hypertension
E. Central nervous system impairment, acute renal, liver, pulmonary insufficiency.
29. Which drug is contraindicated in all stages of Disseminated Intravascular Clotting syndrome?
A. * Heparin
B. Fresh frozen plasma
C. Contrical
D. Transamacha acid
E. Gordox
30. Which drug is contraindicated in all stage of Disseminated Intravascular Clotting syndrome?
A. Fresh frozen plasma
B. * Fibrinogen
C. Aminocapronic acid
D. Tranexamic acid

E. Proteolytic enzymes inhibitors


31. All of the below are the main signs of Cardiorespiratory collapse in amniotic fluid embolism
EXCEPT:
A. Severe pain in the chest
B. Cough
C. Feeling of the death, cyanosis
D. * Hypertension
E. Sudden dyspnea, hypotension
32. Predomination of amniotic pressure over venous is presented in Amniotic fluid embolism in all
below situations EXCEPT:
A. excessive labor contractions
B. * preterm labor
C. placenta abruption
D. uterine cervix dystocia
E. multiple pregnancy
33. Trauma of venous uterine vessels is presented in Amniotic fluid embolism in all below situations
EXCEPT:
A. placenta abruption
B. puerperal hypotonic hemorrhage
C. cesarean section
D. manual removal of placenta
E. * postdate pregnancy
34. Differential diagnosis of Amniotic fluid embolism is performed with all of the below diseases
EXCEPT:
A. Myocardial infarction
B. Pulmonary artery thrombembolism
C. * Pneumonia
D. Air’ embolism
E. Mendelson’ syndrome
35. When does hypotonic uterine bleeding start:
A. at cervical stage
B. at the expulsive stage
C. after the delivery of the fetus
D. * after the placental separation
E. At pelvic stage
36. All of the below are the main reasons for postpartum hemorrhages EXEPT:
A. Birth canal trauma
B. Violation of the uterine contractile activity
C. DIC syndrome
D. * Hypertonic disease
E. Lacerations of birth canal
37. In the early postpartum bleeding you should perform firstly:
A. * Inspection and suturing of birth canal lacerations
B. Colposcopy
C. Hysterotocography
D. Laparoscopy
E. Ultrasonography

38. First aid in case of early postpartum bleeding includes:


A. * Contractile drugs prescription
B. Diuretics administration
C. Vessels ligation
D. Using of mezaton because of low blood pressure
E. Hysterectomy
39. Retained placenta in uterine cavity is diagnosed firstly by:
A. Abnormal shape of the placenta
B. Delay of amniotic membranes
C. * Broken vessels in the placenta
D. Thin placenta
E. Hypertension
40. Retained placenta in uterus in late postpartum period is diagnosed by:
A. Uterine palpation
B. * Ultrasound examination
C. X ray examination
D. Vaginal examination
E. Colposcopy
41. Management of bleeding on 6 day of postpartum period:
A. Manual exploration of the uterine cavity
B. * Uterine curettage
C. Pudendal block
D. Puncture of the Douglas space
E. Nothing above
42. The next step after prescription 2-3 line contractile drugs for atonic postpartum bleeding treatment
A. is:
External massage of the uterus
B. Oxytocin prescription
C. * Bimanual uterine compression
D. Tranexamic acid prescription
E. Balloon tamponade of the uterus
43. The next step after bimanual uterine compression for atonic postpartum bleeding treatment is:
A. External massage of the uterus
B. Oxytocin prescription
C. Manual exploration of uterine cavity
D. Tranexamic acid prescription
E. * Balloon tamponade of the uterus
44. Which blood loss in uterine atony is indication for laparotomy?
A. * 1, 5% and more from body weight
B. more than 250 ml
C. more than 400 ml
D. more than 100 ml
E. 0,5 % from body weight
45. Methods of surgical hemostasis for uterine devascularization?
A. bilateral ligation of uterine vessels
B. bilateral ligation of ovarian vessels
C. compressive sutures

D. bilateral ligation of a.iliaca interna


E. * all of the above
46. Indications for hysterectomy in the case of postpartum bleeding?
A. Placenta accreta
B. Placenta increta
C. Placenta percreta
D. Uterine rupture
E. * All of the above
47. Management of uterine inversion in postpartum period:
A. * Reposition of the uterus
B. Total uterine hysterectomy
C. Subtotal uterine hysterectomy
D. Tamponade of the uterus
E. Ligation of uterine vessels
48. What is the most characteristic sign of amniotic fluid embolism during cesarean section in general
anesthesia?
A. Cough
B. Bradycardia
C. Hypertension
D. Hypothermia
E. * Unexplained decreasing of saturation to 70-85%
49. What is the most characteristic sign of amniotic fluid embolism during cesarean section in general
anesthesia?
A. Cough
B. Bradycardia
C. Hypertension
D. Hypothermia
E. * Wet wheezing in the lungs
50. “Three rule” catheters in hemorrhagic shock management includes:
A. Rectal enema, vein catheterization, urinary bladder catheterization
B. Intestinal intubation, vein catheterization, urinary bladder catheterization
C. * Oxygen require, vein catheterization, urinary bladder catheterization
D. Vein catheterization, urinary bladder catheterization
E. Rectal enema, vein catheterization
51. Amniotic fluid embolism is differentiated with the all of the below pathologies EXEPT:
A. Myocardial infarction
B. Pulmonary artery embolism
C. Air embolism
D. * Shyhan’syndrome
E. Mendelson’ syndrome
52. The bleeding began right after childbirth. The blood loss is 300 ml. Placenta doesn’t separate in
active management. What is the most probable diagnosis?
A. * Abnormal placenta adherence
B. Cervical laceration
C. Uterine atony
D. Uterine rupture
E. Placental abruption

53. What does “А” mean in ABC steps of massive postpartum bleeding management?
A. * Airways evaluation
B. Breathing evaluation
C. Circulation evaluation
D. Keep the woman warm during appropriate available measures
E. Flat position
54. What does “B” mean in ABC steps of massive postpartum bleeding management?
A. Airways evaluation
B. * Breathing evaluation
C. Circulation evaluation
D. Keep the woman warm during appropriate available measures
E. Flat position
55. What does “C” mean in ABC steps of massive postpartum bleeding management?
A. Airways evaluation
B. Breathing evaluation
C. * Circulation evaluation
D. Keep the woman warm during appropriate available measures
E. Flat position
56. Restoration of the airways patency in ABC steps of massive postpartum bleeding includes:
A. * Cleanse the mouth, opening the mouth, laryngomask, tracheal intubation
B. Intravenous access
C. Rapid warmed infusion of fluids
D. Keep the woman warm during appropriate available measures
E. By position flat
57. Breathing maintenance in ABC steps of massive postpartum bleeding:
A. Cleanse the mouth, opening the mouth, laryngomask, tracheal intubation
B. Intravenous access
C. Rapid warmed infusion of fluids
D. Keep the woman warm during appropriate available measures
E. * Oxygen by mask, Ambu bag, pulmonary ventilation
58. What is the recommendation for blood transfusion in massive postpartum bleeding?
A. Blood transfusion is not recommended
B. Blood transfusion on the first day of postpartum period
C. Blood transfusion on the second day of postpartum period
D. Blood transfusion on the third day of postpartum period
E. * Transfuse blood as soon as possible
59. Circulation assessment in ABC steps of massive postpartum bleeding include:
A. Determination of pulse
B. * Determination of pulse rate, blood pressure
C. Determination of blood pressure
D. Determination of temperature
E. Determination of woman weight
60. First steps to maintain circulation in ABC steps of massive postpartum bleeding include:
A. Fresh frozen plasma infusion
B. Blood transfusion
C. Leukocytes mass transfusion

D. Recombinant factor VIIa therapy


E. * Infusion up to 3.5 litres of warmed crystalloid Hartmann’s solution (2 litres) and/or colloid (1–2
litres) as rapidly as required
61. What is the second line contractile drug for atonic postpartum bleeding treatment?
A. Remestyp
B. Oxytocin
C. Water pepper tincture
D. Tranexamic acid
E. * Carbetocin
62. Postpartum hemorrhage is:
A. more than 50 ml
B. more than 100 ml
C. more than 200 ml
D. more than 300 ml
E. * more than 500ml
63. For prevention of bleeding in late postpartum period is necessary to prescribe:
A. Vicasol
B. Ultrasonic investigation of uterus
C. General blood test
D. General urine test
E. * Oxytocin
64. How many units of oxytocin initially you should prescribe for prevention of postpartum bleeding?
A. 5
B. * 10
C. 15
D. 20
E. 25
65. Carbetocin is called as:
A. * Pabal
B. Oxytocin
C. Enzaprost
D. Dinoprost
E. Mezaton
66. How many mkg of carbetocin does 1 ml of pabal contain?
A. * 100
B. 200
C. 300
D. 400
E. 500
67. How do you called drug which contain terlipresyn?
A. * Remestyp
B. Oxytocin
C. Enzaprost
D. Dinoprost
E. Mezaton
68. How many mkg of terlipresyn does 1 ml of remestyp contain?

A. * 100
B. 200
C. 300
D. 400
E. 500
69. Primary postpartum bleeding:
A. * Occur in early postpartum period or during 24 hours after delivery
B. Occur in early postpartum period
C. Occur in late postpartum period
D. Occur till 22 weeks of pregnancy
E. Occur in labor
70. Secondary postpartum bleeding:
A. Occur in early postpartum period or during 24 hours after delivery
B. Occur in early postpartum period
C. * Occur after 24 hours till 6 weeks after delivery
D. Occur till 22 weeks of pregnancy
E. Occur in labor
71. What is the first line contractile drug for atonic postpartum bleeding treament?
A. Remestyp
B. * Oxytocin
C. Enzaprost
D. Dinoprost
E. Ergometryn
72. Which initial dose of tranexamic acid is the maximum for postpartum bleeding treatment?
A. * 1000-1500mkg
B. 200mkg
C. 300mkg
D. 100mkg
E. 500mkg
73. What is the second line contractile drug for atonic postpartum bleeding treatment?
A. Remestyp
B. Oxytocin
C. Water pepper tincture
D. Tranexamic acid
E. * Ergometryn
74. What is the third line contractile drug for atonic postpartum bleeding treatment?
A. Remestyp
B. Oxytocin
C. Water pepper tincture
D. Tranexamic acid
E. * Carboprost
75. What is the second line contractile drug for atonic postpartum bleeding treatment?
A. Remestyp
B. Oxytocin
C. Water pepper tincture
D. Tranexamic acid

E. * Misoprostol
76. What is the next step after prescription 2-3 line contractile drugs for atonic postpartum bleeding
treatment?
A. External massage of the uterus
B. Oxytocin prescription
C. * Bimanual uterine compression
D. Tranexamic acid prescription
E. Balloon tamponade of the uterus
77. What is the next step after bimanual uterine compression for atonic postpartum bleeding treatment?
A. External massage of the uterus
B. Oxytocin prescription
C. Manual exploration of uterine cavity
D. Tranexamic acid prescription
E. * Balloon tamponade of the uterus
78. What is the leading reason for early postpartum bleeding?
A. * Uterine atony
B. Amniotic fluid embolism
C. DIC syndrome
D. Lacerations of the birth canal
E. Abnormal placenta adherence
79. Small hemorrhage in obstetrics is:
A. * blood loss till 250ml which stopped
B. blood loss till 350ml which stopped
C. blood loss till 450ml which doesn’t stopped
D. blood loss till 500 ml which stopped
E. blood loss till 1000ml which doesn’t stopped
80. How does the anti-shock garment work:
A. * Returns blood to vital organs
B. Stimulates uterine contraction
C. Stimulates prostaglandins synthesis
D. Prevents uterine atony
E. Inhibits proteolysis
81. How does the anti-shock garment works:
A. * Decreases blood flow in the compresses organs
B. Stimulate s uterine contraction
C. Stimulates prostaglandins synthesis
D. Prevents uterine atony
E. Inhibits proteolysis
82. What is the best management of labor in breech –cephalic presentation?
A. vaginal delivery
B. * cesarean section
C. vacuum extraction
D. forceps application
E. breech extraction
83. What is the best management of labor in cephalic – breech presentation?
A. * vaginal delivery

B. cesarean section
C. vacuum extraction
D. forceps application
E. breech extraction
84. What is the best management of labor in cephalic – cephalic presentation?
A. * vaginal delivery
B. cesarean section
C. vacuum extraction
D. forceps application
E. breech extraction
85. What is the best management of labor in breech – transverse presentation?
A. vaginal delivery
B. * cesarean section
C. vacuum extraction
D. forceps application
E. breech extraction
86. What are the main compounds of the amniotic fluid?
A. * vernix and ectodermal fetal cells
B. mononuclear cells and macrophages
C. lymphocytes
D. eosinophils and vernix
E. erythrocytes
87. What is the most common cause of oligohydramnios?
A. renal anomalies
B. fetal growth retardation
C. twin–twin transfusion
D. * premature rupture of fetal membranes
E. Diabetus mellitus
88. What is recommended for labor’ induction in patients with polyhydramnion?
A. stimulation of uterine contractions
B. episiotomy
C. cesarean section
D. * early amniotomy
E. obstetrics forceps
89. Polyhydramnios is a risk factor of :
A. postpartum infections
B. * early postpartum bleeding
C. fetal macrosomia
D. fetal malformations
E. Pregnancy induced hypertension
90. Which complication is typical for I stage of labor in patients with polyhydramnios?
A. placenta Previa
B. bleeding
C. maternal infection
D. fetal distress
E. * hypotonic uterine contractions

91. The anatomically contracted pelvis is associated with:


A. * true conjugate 9 cm
B. internal conjugate 11.5 cm
C. external conjugate 20.5 cm
D. diagonal conjugate 13.5 cm
E. cephalopelvic disproportion
92. The clinically contracted pelvis is associated with:
A. true conjugate 9 cm
B. true conjugate 11.5 cm
C. true conjugate 12.5 cm
D. true conjugate 13.5 cm
E. * cephalopelvic disproportion
93. In a nullipara at term the diagonal conjugate is10.5 cm. What is the true conjugate?
A. 5cm
B. 6cm
C. 7cm
D. 8cm
E. * 9cm
94. What is the main cause of cephalo-pelvic disproportion?
A. rachitis
B. * fetal macrosomia
C. preterm labor
D. twins
E. fetal distress
95. Excessive compression of birth canal‘ soft tissues would provoke:
A. uterine rupture
B. cervical rupture
C. * vesicovaginal fistulas
D. perineal rupture
E. vaginal rupture
96. The complication of cephalopelvic disproportion is:
A. * uterine rupture
B. cervical rupture
C. vesicovaginal fistules
D. perineal rupture
E. vaginal rupture
97. The pathological contractile ring is a sign of:
A. anatomically contracted pelvis
B. fetal distress
C. fetal macrosomia
D. * danger of uterine rupture
E. fetal distress
98. Management of the clinically contracted pelvis is:
A. normal vaginal delivery
B. * cesarean section
C. obstetrical version

D. fetal destroying operation


E. external cephalic version
99. Female pelvic sizes are: 25 – 28 – 31 – 20 cm. What is the pelvic type?
A. * normal pelvis
B. generally contracted pelvis
C. flat pelvis
D. flat rachitic pelvis
E. transverse contracted flat pelvis
100. Female pelvic sizes are: 23 – 26 – 29 – 17 cm. What is the pelvic type?
A. normal pelvis
B. transverse contracted pelvis
C. flat pelvis
D. flat rachitic pelvis
E. * generally contracted pelvis
101. Female pelvic sizes are: 23 – 26 – 29 – 18 cm. What is the pelvic type?
A. normal pelvis
B. * generally contracted pelvis
C. flat pelvis
D. flat rachitic pelvis
E. transverse contracted pelvis
102. Female pelvic sizes are: 25 – 28 – 31 – 18 cm. What is the pelvic type?
A. normal pelvis
B. generally contracted pelvis
C. * simple flat pelvis
D. flat rachitic pelvis
E. transverse contracted pelvis
103. Female pelvic sizes are: 26 – 26 – 31 – 17 cm. What is the pelvic type?
A. normal pelvis
B. generally contracted pelvis
C. flat pelvis
D. * flat rachitic pelvis
E. transverse contracted flat pelvis
104. Generally contracted pelvis is characterized by:
A. * Decreasing of all pelvic diameters
B. Decreasing of all pelvic anteroposterior diameters
C. Decreasing of all pelvic transverse diameters
D. Decreasing of true conjugate and increasing of the pelvic outlet
E. Combination of generally contracted and flat pelvis
105. Simple flat pelvis is characterized by:
A. Decreasing of all pelvic diameters
B. * Decreasing of all pelvic anteroposterior diameters
C. Decreasing of all pelvic transverse diameters
D. Decreasing of true conjugate and increasing of the pelvic outlet
E. Combination of generally contracted and flat pelvis
106. Flat rachitic pelvis is characterized by:
A. Decreasing of all pelvic diameters

B. Decreasing of all pelvic anteroposterior diameters


C. Decreasing of all pelvic transversal diameters
D. * Decreasing of true conjugate and increasing of the pelvic outlet
E. Combination of generally contracted and flat pelvis
107. What is the cause of cephalo-pelvic disproportion?
A. * Face presentation anterior
B. Face presentation posterior
C. Preterm labor
D. Twins
E. Fetal distress
108. Transverse contracted pelvis is characterized by:
A. Decreasing of all pelvic diameters
B. Decreasing of all pelvic anteroposterior diameters
C. * Decreasing of all pelvic transverse diameters
D. Decreasing of true conjugate and increasing of the pelvic outlet
E. Combination of generally contracted and flat pelvis
109. The true conjugate of the flat pelvis is 9.5 cm. What is the degree of pelvic contraction?
A. * I degree
B. II degree
C. III degree
D. IV degree
E. V degree
110. The true conjugate of the flat pelvis is 8.5 cm. What is the degree of pelvic contraction?
A. I degree
B. * II degree
C. III degree
D. IV degree
E. V degree
111. The true conjugate of the flat pelvis is 8.0 cm. What is the degree of pelvic contraction?
A. I degree
B. * II degree
C. III degree
D. IV degree
E. V degree
112. The true conjugate of the generally contracted pelvis is 7.0 cm. What is the degree of pelvic
contraction?
A. I degree
B. II degree
C. * III degree
D. IV degree
E. V degree
113. The true conjugate of the generally contracted pelvis is 6.5 cm. What is the degree of pelvic
contraction?
A. I degree
B. II degree
C. * III degree
D. IV degree

E. V degree
114. The true conjugate of the generally contracted pelvis is 5.0 cm. What is the degree of pelvic
contraction?
A. I degree
B. II degree
C. III degree
D. * IV degree
E. V degree
115. Indicate “rare occurred” type of contracted pelvis:
A. * osteomalatic pelvis
B. generally contracted pelvis
C. flat pelvis
D. flat rachitic pelvis
E. generally contracted pelvis
116. Management of the patients with I degree of pelvic contraction and probable fetal weight 2900g:
A. * vaginal delivery
B. cesarean section only
C. vaginal delivery or cesarean section
D. fetal destroying operation
E. labor preparing operation
117. Management of the patients with II degree of pelvic contraction and probable fetal weight 2800g:
A. * vaginal delivery
B. cesarean section only
C. vaginal delivery or cesarean section
D. fetal destroying operation
E. labor preparing operation
118. Management of the patients with III degree of pelvic contraction:
A. vaginal delivery
B. * cesarean section only
C. vaginal delivery or cesarean section
D. fetal destroying operation
E. labor preparing operation
119. Management of the patients with IV degree of pelvic contraction:
A. vaginal delivery
B. * cesarean section only
C. vaginal delivery or cesarean section
D. fetal destroying operation
E. labor preparing operations
120. What is the cause of cephalo-pelvic disproportion?
A. * sinciput vertex presentation and large fetus
B. face presentation posterior
C. occiput presentation
D. foot-link presentation
E. knee-link presentation
121. What is the cause of cephalo-pelvic disproportion?
A. frank breech presentation

B. * sinciput vertex presentation and III degree of pelvic contraction


C. occiput presentation
D. foot-link presentation
E. knee-link presentation
122. What is the aim of prescription of contractile drug (Oxytocin) after incomplete abortion?
A. to prevent intrauterine infection
B. * to avoid placental fragment retention
C. to reduce the risk of hypertension
D. to allow hormone levels to return to normal
E. to avoid future infertility
123. What is the risk factor for an ectopic pregnancy?
A. * history of pelvic inflammatory disease
B. grand multiparity (five or more births)
C. use of an intrauterine device for 1 year
D. use of an oral contraceptive for 5 years
E. use of an condoms for 3 years
124. Which three tests typically are ordered to identify ectopic pregnancy?
A. serum test for beta-hCG, ultrasonography, and amniocentesis
B. serum test for progesterone, laparoscopy, and culdocentesis
C. serum test for estrogen, ultrasonography, and nitrazine paper test
D. * serum test for beta-hCG, ultrasonography, and laparoscopy
E. serum test for beta-hCG, nitrazine paper test, and colposcopy
125. What ultrasonography findings are typical for gestational trophoblastic disease
A. empty gestational sac
B. * grapelike clusters
C. severely malformed fetus
D. ectopic pregnancy
E. two babies
126. Which of the following is a common adverse effect of ritodrine?
A. * tachycardia
B. pohuria
C. hypertension
D. hyporeflexia
E. hypotermia
127. In planning for the care of a 30-year-old woman with pregestational diabetes the most important
factor affecting pregnancy outcome is the:
A. Mother's age
B. Number of years since diabetes was diagnosed
C. Amount of insulin required prenatally
D. * Degree of glycemic control during pregnancy
E. Level of proteinuria
128. What is the aim of a laboratory test for glycosylated hemoglobin Alc for a woman with gestational
diabetes:
A. Is now done for all pregnant women, not just those with or likely to have diabetes.
B. Is a snapshot of glucose control at the moment.
C. * Would be considered evidence of good diabetes control with a result of 2.5% to 5.9%.
D. Is done on the patient’s urine, not her blood.

E. Need for calculation of Insulin Dose


129. Which factor is the most important in diminishing maternal/fetal/neonatal complications in a
pregnant woman with diabetes?
A. The woman’s stable emotional and psychological status
B. Evaluation of retinopathy by an ophthalmologist
C. Total protein excretion and creatinine clearance within normal limits
D. * Degree of glycemic control before and during the pregnancy
E. Constant dose of insulin during pregnancy
130. Which major neonatal complication is carefully monitored after the birth of the infant of a diabetic
mother?
A. * Hypoglycemia
B. Hypercalcemia
C. Hypobilirubinemia
D. Hypoinsulinemia
E. Diuresis
131. Which factor is known to increase the risk of gestational diabetes mellitus?
A. Underweight prior to pregnancy
B. Maternal age less than 25 years
C. * Previous birth of large infant
D. Previous diagnosis of type 2 diabetes mellitus
E. Previous Cesarean Section
132. Glucose metabolism is profoundly affected during pregnancy because:
A. pancreatic function in the islets of Langerhans is affected by pregnancy.
B. a pregnant woman uses glucose at a more rapid rate than a nonpregnant woman.
C. a pregnant woman increases her dietary intake significantly.
D. * placental hormones are antagonistic to insulin, resulting in insulin resistance.
E. glucose metabolism affected during pregnancy just in woman with Diabetes Mellitus
133. Which statement is true about need in insulin for pregnant women with diabetes?
A. “Need in insulin for pregnant women with diabetes increases throughout pregnancy and the
postpartum period”.
B. “Need in insulin for pregnant women with diabetes decreases throughout pregnancy and the
postpartum period”.
C. * “Need in insulin for pregnant women with diabetes varies depending on the stage of gestation”.
D. “Need in insulin for pregnant women with diabetes should not change because the fetus produces
its own insulin”.
E. “Pregnant women with diabetes do not need insulin because the fetus produces enough insulin for
both”
134. Which patient is not at risk for gestational diabetes mellitus?
A. patient with chronic hypertension
B. patient how has had previous unexplained fetal deaths
C. * patient who feels fatigued by the end of the day
D. patient who’s maternal age is older than 25yrs.
E. patient with obesity
135. What is the insulin needs during the first trimester in diabetic pregnant woman?
A. They will increase.
B. * They will decrease.
C. They will remain constant.

D. They will be unpredictable.


E. They depend from baby’s glucose level
136. What is NOT the complication of pregnancy in woman with insulin-dependent diabetes?
A. Candida albicans infection.
B. * Twin-to-twin transfer.
C. Polyhydramnios.
D. Preeclampsia.
E. Diabetic fetopathy
137. What is the common cause of hyperglycemia during pregnancy in diabetic woman during
A. pregnancy?
Fetal macrosomia.
B. Obesity before conception.
C. * Maternal infection.
D. Pregnancy-induced hypertension.
E. Polyhydramnios
138. What is the main clinical symptom of hyperglycemia in diabetic primigravida?
A. * Dehydration.
B. Pallor.
C. Sweating.
D. Nervousness
139. Which of the following situation can be prevented with Cesarean Section in pregnant women with
poorly controlled diabetes and severe preeclampsia?
A. Neonatal hyperbilirubinemia
B. Congenital anomalies.
C. Perinatal asphyxia.
D. * Stillbirth.
E. Preterm Rupture of Membranes
140. What is the cause of fetal macrosomia delivered by diabetic mother?
A. Family history of large infants.
B. Fetal anomalies
C. * Maternal hyperglycemia.
D. Maternal hypertension.
E. Maternal obesity
141. Postpartal insulin needs in insulin-dependent diabetic breastfeeding mothers
A. * fall significantly in the immediate postpartum period.
B. remain the same as during the labor process.
C. usually increase in the immediate postpartum period.
D. need constant adjustment during the first 24 hours.
E. usually unpredictable
142. A history of macrosomic infants, two stillbirths, and polyhydramnios with each pregnancy are the
risk factors of:
A. Toxoplasmosis
B. Abruptio placentae
C. Hydatidiform mole
D. * Diabetes mellitus
E. Preeclampsia
143. Pregnancy affects glucose metabolism because:
A. * Placental hormones increase the resistance of cells to insulin

B. Insulin cells cannot meet the body’s demands as the woman’s weight increases
C. There is a decreased production of insulin during pregnancy
D. The speed of insulin breakdown is decreased during pregnancy
E. Baby needs some quantity of maternal insulin
144. Why woman who uses oral hypoglycemic agents to control diabetes mellitus will need to take
insulin during pregnancy?
A. Insulin can cross the placental barrier to the fetus
B. * Insulin does not cross the placental barrier to the fetus
C. Oral agents do not cross the placenta
D. Oral agents are not sufficient to meet maternal insulin needs
E. Because insulin is free for pregnant woman
145. Intervention for pregnant women with diabetes is based on the knowledge that the need for insulin:
A. increases throughout pregnancy and the postpartum period.
B. decreases throughout pregnancy and the postpartum period.
C. * varies depending on the stage of gestation.
D. should not change because the fetus produces its own insulin.
E. usually unpredictable
146. Signs and symptoms of cardiac decompensation in pregnant woman are
A. A regular heart rate and hypertension
B. An increased urinary output, tachycardia, and dry cough
C. Shortness of breath, bradycardia, and hypertension
D. * Dyspnea, crackles, and an irregular, weak pulse
E. Shortness of breath, increased urinary output, regular heart rate
147. Prophylaxis of subacute bacterial endocarditis (SBE) is given before and after birth when a
pregnant woman has:
A. * Valvular disease
B. Congestive heart disease
C. Arrhythmias
D. Postmyocardial infarction
E. Hypertension
148. Postpartum care of the woman with cardiac disease:
A. Is the same for that of any pregnant woman.
B. * Includes rest, stool softeners, and monitoring the effect of activity.
C. Includes ambulating frequently, alternating with active range of motion.
D. Includes limiting visits with the infant to once per day.
E. Depends of blood pressure
149. A sign of cardiac decompensation in pregnant woman with rheumatic heart disease would be
A. Dry skin, hacking cough
B. Supine hypotension
C. Wheezing with inspiration and expiration
D. * Rapid pulse that is irregular and weak
E. Bradycardia, and hypertension
150. Which type of anesthesia is generally or more effective method of pain relief for the labour of a
woman with cardiac problem
A. * Epidural anestesia
B. Narcotics
C. Breathing techniques

D. All pain relief methods are contraindicated


E. General anestesia
151. What type of anemia is the most common during pregnancy
A. * Iron deficiency
B. Folic acid deficiency
C. Sickle cell hemoglobinopathy
D. Thalassemia
E. hemolytic
152. . Physiologic stress on the heart is the greatest between the __ and__ weeks of gestation because
cardiac output is at peak
A. 18-22
B. 38-40
C. 18-32
D. * 28-32
E. 16-22
153. Risk for cardiac decompensation is also higher during
A. Childbirth and first 24-48 weeks after birth
B. Childbirth and first 24-48 days after birth
C. * Childbirth and first 24-48 hours after birth
D. Childbirth and first 24-48 months after birth
E. Childbirth and 24-48 hours before birth
154. According to the classification of cardiovascular disorders developed by New York Heart
Association Class I implies
A. symptomatic with increased activity
B. symptomatic at rest
C. symptomatic with ordinary activity
D. * asymptomatic at normal level of activity
E. symptomatic with excessive activity
155. According to the classification of cardiovascular disorders developed by New York Heart
Association Class II implies
A. asymptomatic at normal level of activity
B. symptomatic with ordinary activity
C. * symptomatic with increased activity
D. symptomatic at rest
E. symptomatic with excessive activity
156. According to the classification of cardiovascular disorders developed by New York Heart
Association Class III implies
A. * symptomatic with ordinary activity
B. asymptomatic at normal level of activity
C. symptomatic at rest
D. symptomatic with increased activity
E. symptomatic with excessive activity
157. According to the classification of cardiovascular disorders developed by New York Heart
Association Class IV implies
A. asymptomatic at normal level of activity
B. symptomatic with ordinary activity
C. symptomatic with increased activity

D. * symptomatic at rest
E. symptomatic with excessive activity
158. What is congestive heart failure with cardiomyopathy founding the last month of pregnancy or with
the first 5 month postpartum, lack of another cause for heart failure and absence of heart disease
prior to the last month of pregnancy
A. Eisenmenger Syndrome
B. Marfan’s Syndrome
C. * Peripartum cariomyopathy
D. Aortic Stenosis
E. Aortic coarctation
159. Which disease refers the damage of the heart valves and the chordae tendineae cordis as a result of
an infection originating from an inadequately group A ?-hemolitic streptococcal infection of the
throat during pregnancy
A. * Rheumatic heart disease
B. Systemic lupus erythematosus
C. Eisenmenger Syndrome
D. Marfan’s Syndrome
E. Pulmonary Hypertension
160. Complication related to maternal cardiovascular problems is:
A. * Preterm labour
B. Postterm labour
C. Oligohydroamnios
D. Macrosomia
E. Preterm Rupture of Membranes
161. Complication related to maternal cardiovascular problems is:
A. Macrosomia
B. * Stillbirth
C. Low birth weight newborn
D. Preterm Rupture of Membranes
E. Postterm labour
162. Which changes occur during normal pregnancy and affect the woman with cardiac disease
A. Decrease cardiac output
B. Hg concentration rise
C. * Increase intravascular volume
D. Systemic blood pressure falls during the third trimester
E. Bradicardia and hypotension
163. Pregnancy is contraindicated for a woman who has had
A. Heart transplantantation
B. Ventricular septal defect
C. * Pulmonary hypertension
D. Atrial septal defect
E. Hypertension
164. Pregnancy is contraindicated for a woman who has had
A. Patent ductus arteriosus
B. * Shunt lesions associated with Eisenmenger syndrome
C. Artificial heart valves
D. Atrial septal defect

E. Mitral valve prolapse


165. Folic acid deficiency during conception and early pregnancy increase risk of
A. * Neural tube defects
B. Low birth weight newborn
C. Heart failure
D. Gastroshisis
E. Prolonged labour
166. Folic acid deficiency during conception and early pregnancy increase risk of
A. * Cleft lip
B. Chloasma
C. Striae gravidarum
D. Cleft nose
E. Exophthalmia
167. Which of the following forms of heart disease in women of childbearing years usually has a benign
effect on pregnancy?
A. Cardiomyopathy
B. Rheumatic heart disease
C. Congenital heart disease
D. * Mitral valve prolapse
E. Pulmonary hypertension
168. When teaching a pregnant woman with class II heart disease, the physician should:
A. advises her to gain at least 13 kg.
B. explains the importance of a diet high in calcium.
C. * instructs her to avoid strenuous activity.
D. informs her of the need to limit fluid intake.
E. prefer bed rest more than 14 hour per day
169. Which instructions are most important to include in a teaching plan for a client in early pregnancy
that has class I heart disease?
A. She must report any nausea or vomiting.
B. She may experience mild fatigue in early pregnancy.
C. * She must report any chest discomfort or productive cough.
D. She should plan to increase her daily exercise gradually throughout pregnancy.
E. She must report any medication she takes
170. Anti-infective prophylaxis is indicated for a pregnant woman with a history of mitral valve stenosis
related to rheumatic heart disease because the woman is at risk of developing:
A. hypertension
B. upper respiratory infections
C. postpartum infection
D. * bacterial endocarditis
E. heart failure
171. Which of the following should the physician include when planning intrapartum care for a client
with heart disease?
A. Take vital signs according to standard protocols.
B. * Continuously monitor cardiac rhythm.
C. Massage the uterus to hasten delivery of the placenta.
D. Maintain infusion of intravenous fluids to avoid dehydration.
E. Recommend additional physical activity

172. The primary fetal risk when the mother has any type of anemia is for
A. neonatal anemia
B. elevated bilirubin
C. limited infection defenses
D. * reduced oxygen delivery
E. prolonged lbour
173. The action that has the highest priority for a client with class I heart disease during the postpartum
period should be:
A. Promotion of aggressive ambulation
B. * Observation for signs of cardiac decompensation
C. Assessment of the mother's reaction to the birth
D. Advisement about activity levels during the postpartum period
E. Support of Formula Feeding
174. The position that the physician should encourage a client with cardiac disease to assume during
labor is:
A. Supine
B. High-Fowler's
C. * Semi-Fowler's
D. Trendelenburg
E. Knee-Chest
175. Rheumatic heart disease is usually preceded by which infection?
A. * streptococcal pharyngitis
B. syphilis
C. pneumococcal pneumonia
D. chlamydial vaginitis
E. gonorrhea
176. Which factor can complicate diagnosis and surgical treatment for abdominal problems during
pregnancy?
A. * displacement of internal organs
B. present of the fetus
C. retro deviation of uterine body
D. constipations related pregnancy
E. congenital abnormalities
177. Surgery may be necessary in woman with cholecystitis if she has:
A. One attacks of biliary colic
B. Chronic cholecystitis
C. * Obstructive jaundice
D. Chronic pancreatitis
E. Acute gastritis
178. The woman usually return to the her prepregnancy asthma status after
A. * 3 month after giving birth
B. 3 weeks after giving birth
C. 3 years after giving birth
D. 3 hours after giving birth
E. 3 years after giving birth
179. The severity of asthma symptoms usually peaks between
A. 6 and 16 week’s gestation

B. 16 and 26 week’s gestation


C. 26 and 36 day’s after birth
D. * 26 and 36 week’s gestation
E. during labour
180. Appendicitis is more difficult to diagnose during pregnancy because the appendix is:
A. Displaced posterior and Covered by the uterus
B. Displaced to the left
C. Displaced Low and to the right
D. * Displaced High and to the right
E. Displaced anterior and compress between the bladder and the uterus
181. Postoperative care of the pregnant woman who requires abdominal surgery for appendicitis
includes which additional assessment?
A. Intake and output, intravenous site
B. Signs and symptoms of infection
C. Vital signs and incision
D. * Fetal heart rate and uterine activity
E. Signs and symptoms of trombosis
182. ____ is a disorder of the brain causing recurrent seizures; it is the most common neurologic
disorder accompanied pregnancy
A. pregnancy induce hypertension
B. ectopic pregnancy
C. eclampsia
D. * epilepsy
E. hydatidyform mole
183. Preoperative care of a pregnant woman differs from that for a nonpregnant woman in one
significant aspect, namely the presence of the
A. * fetus
B. pregnant uterus
C. hormonal changes
D. organs displacement
E. enlarged abdomen
184. General preoperative observation and ongoing care are the same as for any surgery, with addition of
continuous ___ monitoring if the fetus is considered to be viable.
A. fetal position
B. * FHR
C. fetal movement
D. fetal circulation
E. fetal presentation
185. General preoperative observation and ongoing care are the same as for any surgery, with addition of
continuous ___ monitoring if the fetus is considered to be viable.
A. uterine size
B. uterine consistency
C. * uterine contractions
D. uterine relaxations
E. uterine location
186. Intrapartally fetal oxygenation is improved by placing the woman on an operating table with a ___
to avoid maternal vena cava compression

A. Trendelenburg’s position
B. longitudinal position
C. lithotomy position
D. * lateral tilt position
E. knee-chest position
187. Intrapartally feta oxygenation is improved by placing the woman on an operating table with a
lateral tilt to avoid:
A. maternal malabsorbtion
B. maternal aspiration of gastric masses
C. * maternal vena cava compression
D. maternal aorta compression
E. maternal discomfort
188. The physician would suspect pyelonephritis when a pregnant woman reports:
A. Frequency and urgency of urination
B. Nausea and weight loss
C. Burning sensation when voiding
D. * Tenderness in the flank area
E. Anuria
189. Chronic hypertension is hypertension which
A. * Detected before pregnancy
B. Detected after 20th week
C. Detected at postpartum period
D. Detected during the labor
E. Detected after delivery
190. Pregnancy induced hypertension is hypertension which
A. Detected before pregnancy
B. * Detected after 20th week
C. Detected before 20th week
D. Detected 45 days after delivery
E. Detected during pregnancy
191. Indications for drug therapy in woman with chronic hypertension include
A. * Diastolic BP>110
B. Systolic BP >140
C. Diastolic BP>130
D. Systolic BP >160
E. Elevated BP
192. Which drug is contraindicated for woman with chronic hypertension?
A. Peripheral vasodilatators
B. * Angiotensin-converting enzyme inhibitors
C. β-blockers
D. Ca-channel blockers
E. β2-receptor agonists
193. Which drug is contraindicated for woman with chronic hypertension?
A. Peripheral vasodilatators
B. * Diuretics
C. ?-blockers

D. Ca-channel blockers
E. ?2-receptor agonists
194. What is the first sign of fluid retention suggestive of pregnancy-induced hypertension?
A. Abdominal enlargement
B. Facial swelling
C. * Sudden weight gain
D. Swelling of the feet and ankles
E. General edema
195. The patient with gestational hypertension has all the signs below. The most dangerous symptom is:
A. Diarrhea
B. Decreased urine output
C. * Blurred vision
D. Backache
E. Facial swelling
196. What is the main recommendation for pregnant with chronic hypertension during routine check up?
A. * Activity restriction
B. Balanced nutrition
C. Increased fluid intake to ensure adequate hydration
D. Instruction about the effect of diuretics
E. Increased nutrition
197. The aim of magnesium sulfate therapy for the patient with preeclampsia is to:
A. * Prevent convulsions
B. Promote diaphoresis
C. Increase reflex irritability
D. Act as a saline cathartic
E. Increase peripheral circulation
198. Which assessment should be performed during intravenous magnesium sulfate infusion therapy for
management of severe preeclampsia?
A. * Count respirations and report a rate of less than 12 breaths per minute.
B. Count respirations and report a rate of more than 20 breaths per minute.
C. Check blood pressure and report a rate of less than 100/60.
D. Monitor urinary output and report a rate of less than 100 ml per hour.
E. Monitor reflexes and report increased reflexes
199. Which drug should be available for immediate IV administration when magnesium sulfate toxity is
developed:
A. Ergonovine maleate
B. Oxytocin
C. * Calcium gluconate
D. Hydralazine
E. ?2-receptor agonists
200. Several pregnant clients are waiting to be seen in the triage area of the obstetrical unit. Which client
is the highest priority?
A. A client at 13 weeks' gestation experiencing nausea and vomiting three times a day with + 1
ketones in her urine.
B. A client at 37 weeks' gestation who is an insulin-dependent diabetic and experiencing 3 to 4 fetal
movements per day.

C. * A client at 32 weeks' gestation who has preeclampsia and + 3 proteinuria who is returning for
evaluation of epigastric pain.
D. A primigravida at 17 weeks' gestation complaining of not feeling fetal movement at this point in
her pregnancy.
E. A multigravida at 38 weeks’ gestation with mild irregular abdominal pain
201. What is the most appropriate client centered recommendation for a primigravid client at 30 weeks'
gestation diagnosed with mild preeclampsia
A. Return visit to the prenatal clinic in approximately 4 weeks.
B. Decreased edema after 1 week of a low-protein, low-fiber diet.
C. * Bed rest on the left side during the day, with bath-room privileges.
D. Immediate reporting of adverse reactions to magnesium sulfate therapy.
E. Call physician immediately after development of swelling of the feet and ankles
202. What is NOT the typical complication of preeclampsia?
A. * Hydrocephalic infant.
B. Abruptio placentae.
C. Intrauterine growth retardation.
D. Poor placental perfusion.
E. Preterm labor
203. Which of the following types of diet should be physician discussed with a multigravid client
diagnosed with mild preeclampsia?
A. High-residue diet.
B. Low-sodium diet.
C. * Regular diet.
D. High-protein diet.
E. Low-protein diet.
204. Which of the following assessment findings would alert the physician to suspect magnesium sulfate
toxity?
A. * Decreased deep tendon reflexes.
B. Cool skin temperature.
C. Rapid pulse rate
D. Tingling in the toes.
E. Decrease of ankle edema
205. What is the aim of magnesium sulfate therapy prescribed for treatment of severe preeclampsia
A. Decreased generalized edema within 8 hours.
B. Decreased urinary output during the first 24 hours
C. Sedation and decreased reflex excitability within 48 hours.
D. * Absence of any seizure activity during the first 48 hours.
E. Decreased breath movement less than 12 per minute
206. Which group of drug does magnesium sulfate used for treatment of severe preeclampsia belongs
A. to?
Peripheral vasodilator
B. Antihypertensive.
C. * Central nervous system depressant.
D. Sedative-hypnotic.
E. Diuretic
207. Which of the following assessment findings should be report immediately during a continuous
intravenous infusion of 4 g of magnesium sulfate in Ringer's solution?
A. * Respiratory rate of 12 breaths /minute

B. Patellar reflex of+2.


C. Blood pressure of 160/88 mm Hg.
D. Urinary output exceeding intake.
E. Increase of deep tendon reflexes
208. What is the first action at the beginning of a seizure in client diagnosed with severe preeclampsia?
A. Insert an airway to improve oxygenation.
B. Note the time when the seizure begins and ends.
C. * Call for immediate assistance.
D. Turn the client to her left side.
E. Catheterize central vein
209. What complication of labour can be suspected after eclampsia?
A. * Abruptio placentae.
B. Transverse lie.
C. Placenta accreta.
D. Uterine atony.
E. Placenta previa
210. Which of the following assessments should the physician prescribe during intravenous magnesium
sulfate infusion for treatment of severe preeclampsia?
A. Urinary output even'8 hours.
B. Deep tendon reflexes even' 4 hours.
C. * Respiratory rate every hour.
D. Blood pressure every 6 hours.
E. Uterine contraction every 4 hours
211. Which of the following actions should the physician do first in case of seizure developed in woman
with severe preeclampsia?
A. Pad the side rails of the client's bed.
B. Turn the client to the right side.
C. Insert a padded tongue blade into the client's mouth.
D. * Call for immediate assistance in the client's room.
E. Catheterize central vein
212. What is immediate action in case of seizure
A. * Turn the client's head to the side
B. Place an airway into the client's mouth
C. Check the client for a spontaneous birth
D. Assess the fetal heart rate for decelerations
E. Monitor uterine contraction
213. The physician should teach the pregnant client has been diagnosed with gestational hypertension
that her pregnancy diet now requires:
A. * No changes
B. Limited proteins
C. Restricted sodium
D. Increased carbohydrates
E. Increased liquid
214. What is the next step of assessment after obtaining the vital signs in a client with preeclampsia:
A. Call the nurse
B. * Check the client's reflexes
C. Determine the client's blood type

D. Administer intravenous normal saline


E. Catheterize urinary bladder
215. Before administering IV magnesium sulfate therapy to a client with preeclampsia, the physician
should assess the client's:
A. Temperature and respirations
B. Urinary glucose and specific gravity
C. * Urinary output and patellar reflexes
D. Level of consciousness and funduscopic appearance
E. Fetal heart rate and cervical dilatation
216. Magnesium sulfate is being given for treatment of severe preeclampsia primarily because it is a:
A. Hypotensive that relaxes smooth muscles
B. Cholinergic that increases the release of acetylcholine
C. Muscle relaxant that decreases the severity of uterine contractions
D. * Central nervous system depressant that blocks neuromuscular transmissions
E. Source of microelements
217. Which of the following sign or symptom would most likely indicate the potential for a seizure in
client with severe preeclampsia?
A. Audible crackles
B. Blurring of vision
C. * Epigastric discomfort
D. Generalized facial edema
E. Uterine contraction
218. When measuring the blood pressure to ensure consistency and to facilitate early detection of blood
pressure changes consistent with preeclampsia, the physician should:
A. * Place the woman in a seated or a left lateral position
B. Allow the woman to rest for 15 minutes after positioning before measuring her blood pressure
C. Use the woman's right arm if she is lying on her left side.
D. Use a proper sized cuff that covers at least 50% of her upper arm
E. there are no any rules for measuring blood pressure in pregnant women
219. Which one of the following assessment findings would be indicated by progress mild preeclampsia
to severe preeclampsia?
A. * Proteinuria greater than 4+, in two specimens collected 6 hours apart
B. Dependent edema in the ankles and feet at bedtime
C. Deep tendon reflexes 2+, ankle clonus is absent
D. Blood pressure of 154/94 and 156/100, 6 hours apart
E. Urinary output more than 100 ml per hour
220. The primary expected outcome for care associated with the administration of magnesium sulfate
would be met if the woman:
A. Exhibits a decrease in both systolic and diastolic blood pressure
B. * Experiences no seizures
C. States that she feels more relaxed and calm
D. Urinates more frequently resulting in a decrease in pathologic edema
E. Feel fetal movement well
221. The patient with gestational hypertension has all the signs below. Which sign should be alert
because of development complication:
A. Diarrhea
B. Decreased urine output

C. * Blurred vision
D. Backache
E. Frequent urination
222. Which of the following should concern physician that monitoring primigravida in prenatal clinic
for preeclampsia?
A. Blood pressure increase to 138/86 mm Hg
B. Weight gain of 0.5 kg during the past 2 weeks
C. * A dipstick value of 3+ for protein in her urine
D. Pitting pedal edema at the end of the day
E. Increased urinary output to 2000 ml per 24 hours
223. The physician's primary duty during the seizure in a woman with preeclampsia is to:
A. Insert an oral airway
B. Suction the mouth to prevent aspiration
C. Administer oxygen by mask
D. * Call for help
E. Catheterize central vein
224. Which of the following symptom alert about development of magnesium sulfate toxity?
A. A sleepy, sedated affect
B. * A respiratory rate of 10 breaths per minute
C. Deep tendon reflexes of +2
D. Absent ankle clonus
E. Facial edema
225. HELLP syndrome is associated with an increased risk for adverse perinatal outcomes including
which of the following?
A. * Placental abruption
B. Placenta previa
C. Hepato-splenomegalia
D. Cirrhosis
E. Preterm rupture of membranes
226. HELLP syndrome is associated with an increased risk for adverse perinatal outcomes including
which of the following?
A. Glomerulonephritis
B. Placenta previa
C. * Renal failure
D. Cirrhosis
E. Cystitis
227. HELLP syndrome is associated with an increased risk for adverse perinatal outcomes including
which of the following?
A. Macrosomia
B. Placenta previa
C. Cirrhosis
D. * Maternal and fetal death
E. Constipation
228. The antidote administered to reverse magnesium toxicity is
A. calcium phosphate
B. calcium succinat
C. * calcium gluconate

D. calcium chloridum
E. calcium sulfate
229. Which effect of magnesium sulfate which used for treatment of preeclampsia is suspected?
A. tocolytic.
B. * anticonvulsant.
C. antihypertensive.
D. diuretic.
E. spasmolytic
230. What is the only known cure for preeclampsia?
A. Magnesium sulfate
B. Antihypertensive medications
C. * Delivery of the fetus
D. Administration of aspirin every day of the pregnancy
E. Diuretics
231. Which intrapartal assessment should be avoided when caring for a woman with HELLP syndrome?
A. Auscultation of the heart and lungs
B. * Abdominal palpation
C. Checking deep tendon reflexes
D. Venous sample of blood
E. Sonographyc examination of liver
232. Which of the following assessments in a client diagnosed with pregnancy-induced hypertension
who is taking magnesium sulfate would indicate a therapeutic level of medication?
A. Urinary output of 20 ml/hr
B. * Normal deep tendon reflexes
C. Respiratory rate of 10 to 12 breaths/minute
D. Drowsiness
E. Blurred vision
233. A woman taking magnesium sulfate has respiratory rate of 10 breaths/minute. In addition to
discontinuing the medication, the physician should:
A. vigorously stimulate the woman.
B. instruct her to take deep breaths.
C. * administer calcium gluconate.
D. increase her IV fluids.
E. auscultate fetal heart rate
234. Which of the following is a classic sign of preeclampsia?
A. * proteinuria
B. fever
C. glycosuria
D. vomiting
E. constipation
235. During the menstrual cycle, the endometrium would be at its thickest
A. during the menstrual phase
B. just prior to ovulation
C. just after ovulation
D. * late in the postovulatory phase
E. during proliferative phase
236. The part of the female reproductive system that is shed during menstruation is the

A. myometrium
B. Vaginal mucus
C. tunica albuginea
D. * stratum functionalis of the endometrium
E. Basal lay of endometrium
237. Which sexual practices as high-risk factors for contracting HIV?
A. * Heterosexual intercourse
B. Absent of intercourse
C. A monogamous relationship
D. French kissing
E. Hugs
238. The success of the rhythm method depends on the:
A. Age of the client
B. Frequency of intercourse
C. * Regularity of the menses
D. Range of the client’s temperature
E. Quantity of partners
239. Menopause is defined as:
A. 2 or more irregular periods after age 40
B. The start of hot flashes
C. * No menstrual period for 12 consecutive months
D. No menstrual period for 6 consecutive months
E. An increase in mood swings
240. The average age of menopause is:
A. 60
B. 55
C. * 51
D. 45
E. 40
241. The first step in the workup of primary or secondary amenorrhea is:
A. Bimanual examination
B. Ultrasound of abdomen/pelvic
C. * Pregnancy test
D. Nucleic amplification test
E. Mammography
242. Choose the right period for menstruation/follicular phase
A. Days 7-14
B. Days 14-21
C. Days 5-10
D. * Days 1-13
E. Days 14-21
243. Choose the right period for ovulation:
A. Day 5
B. Day 21
C. Day 28
D. Day 7

E. * Day 14
244. Choose the right period for luteal phase:
A. Days 3-14
B. * Days 15-28
C. Days 5-10
D. Days 7-14
E. Days 10-14
245. Hypomenstrual syndrome includes:
A. Opsomenorrhea, polimenorrhea
B. * Oligomenorrhea, opsomenorrhea, hypomenorrhea
C. Proyomenorrhea, hypomenorrhea
D. Oligomenorrhea, hypermenorrhea
E. Spaniomenorrhea, hypermenorrhea
246. What appearance of the first menstruation in 14 years can testify about?
A. about the presence of inflammatory disease of uterus
B. about the presence of inflammatory disease of adnexa
C. about the presence of abnormal position of uterus
D. * about normal development of organism of girl
E. about the presence of of genius infantilism
247. To hypothalamic amenorrhea does not belong:
A. psychogenic amenorrhea
B. * amenorrhea at a syndrome Shikhane
C. amenorrhea at false pregnancy
D. amenorrhea at adipozogenital dystrophy
E. amenorrhea at a syndrome Kiary-Frommel
248. Menorrhagia is:
A. acyclic uterine bleeding
B. * cyclic uterine bleeding in connection with menstruation cycle
C. painfull and abundant menstruation
D. pre- & post menstruation bloody allocation
E. short period of menstruation cycle
249. What is spaniomenorrhea?
A. menstruations come in 6-8 weeks
B. * menstruations come 1 time per 4-6 monthes
C. menstruations are absent
D. quantity of menstrual blood less than 50ml
E. duration of menstruation 1-2 days
250. How is the state named, when less than 2 days proceed to menstruation?
A. spaniomenorrhea
B. hypomenorrhea
C. proyomenorrhea
D. * oligomenorrhea
E. opsomenorrhea
251. What is the initial evaluation in case of secondary amenorrhea?
A. Prolactin, TSH and FSH followed by B-hCG to exclude pregnancy
B. * B-hcG

C. Prolactin level only


D. Patient's BMI and level of cortisol
E. Patient's age and level of gonadotropine
252. Mean age for menopause is?
A. 40 years
B. 45 years
C. * 51 years
D. 48 years
E. 39 years
253. Which process represents the IIIa type of smear at oncocytological examination?
A. the unchanged epithelium
B. * mild or moderate dysplasia
C. cancer
D. inflammatory process
E. suspicion on malignisation
254. For polyp treatment we use all except:
A. cryodestruction
B. * conization of cervix
C. endocervical curettage
D. Coagulation
E. polyp twisting
255. To the risk factors for dysplasia belong all EXCEPT:
A. * hyperprogesteronemia
B. hyperestrogenemia
C. cervical trauma during delivery and abortion
D. A lot of sexual partner
E. Early sexual activity
256. What sign is not typical for proliferative myoma?
A. increased mitotic activity
B. fast growth
C. plenty of lymphoid cells
D. plenty of plasmatic cells
E. * Atypical growth
257. In diagnostic of cervical dysplasia we use all EXCEPT:
A. colposcopy
B. * metrosalpingography
C. cytogenetic examination
D. histochemical examination
E. cytological examination
258. In what type of ovarion tumour do we have an endometrium hyperplasia?
A. papillary cystoma
B. paraovarian cyst
C. * follicle cyst
D. ovarian cancer
E. yellow body cyst
259. In diagnostic of cervical leucoplacia we use all EXCEPT:
A. * culdoscopy
B. colposcopy
C. biopsy
D. cytology
E. Speculum assessment
260. Node consistency of uterine fibromyoma depends on:
A. node size
B. vessels' amount
C. fatty tissue amount
D. presence of endometrioid tissues
E. * correlation of parenchyma and stroma
261. What is typical for proliferative myoma?
A. presence of atypia
B. atypical localization
C. multiple nodes
D. node calcification
E. * increased mitotic activity
262. What sign is not typical for proliferative myoma?
A. increased mitotic activity
B. fast growth
C. plenty of lymphoid cells
D. plenty of plasmatic cells
E. * Atypical growth
263. What sign is typical for submucous myoma?
A. * hyperpolymenorrhea
B. amenorrhea
C. foamy vaginal discharge
D. symptomless
E. tumour destruction
264. What sign is typical for retrocervical myoma?
A. * rectum dysfunction
B. infertility
C. amenorrhea
D. foamy vaginal discharge
E. hyperpolymenorrhea
265. What endometrium conditions belong to the premalignant disease?
A. glandular-cystic hyperplasia
B. endometritis
C. * polyposis of endometrium
D. endometriosis
E. glandular hyperplasia
266. What is typical for hormonal status of patient with hyperplasia of endometrium?
A. * high level of estrogens
B. high level of pituitary gland hormons
C. high level of androgens
D. high level of progesteron
E. high level of chorionic gonadotropin
267. How pregnancy does influence on fibromyoma’s growth?
A. doesn’t effect at all
B. promote fibromyoma’s elimination
C. promote fibromyoma’s malignization
D. promote necrosis of node
E. * promote fast growth of fibromyoma
268. What substance is in IUD for fibromyoma treatment?
A. * levonorgestrel
B. synestrol
C. 17-oxiprogesteroni capronat
D. Zoladex
E. Etinilestradiol
269. At what type of ovarian tumour does endometrium hyperpasia develop?
A. papillary cystoma
B. paraovarian cyst
C. * follicular cyst
D. ovarian cancer
E. yellow body cyst
270. What treatment should be prescribes for patient with dermoid cyst?
A. antibacterial therapy
B. Zoladex
C. * cystectomy
D. substitutive hormonal therapy
E. combined oral contraceptives
271. Cervical pap smear test is recommended to this age group with the timing of:
A. Twice in a lifetime- <25 years old
B. Once a lifetime- all ages
C. Every 10 years- >65 years
D. * Every 3 years- 21 to 65 years
E. Every year- 5 to 18 years
272. Call the benign uterine fibroids, causing enlarged uterus with an irregular contour:
A. Adenocarcinoma
B. Fibroma
C. * Leiomyoma uteri
D. Carcinoma
E. Mole
273. Yellow or yellow-brown masses, solid ovarian mass on ultrasound, 50% bilateral, regress after
delivery is?
A. Bartholian
B. Cancer
C. Krukenberg tumor
D. * Luteoma
E. Theca luteum cyst
274. Diagnostic method to diagnose uterine Leiomyomas (fibroids):
A. Normal observation

B. MRI of pelvis
C. Bimanual exam
D. Pelvic x-ray
E. * Ultrasound of pelvic
275. Which of them is NOT an indication for endometrial biopsy?
A. Lynch syndrome in patient <45 years
B. Atypical glandular cells on pap test
C. Postmenopausal bleeding
D. Obesity/anovulation
E. * Sheehan syndrome
276. Soft mobile mass, normally asymptomatic, commonly on the base of the labium major (vulva) is:
A. Syphilis related mass
B. Malignancy
C. HSV related cyst
D. HPV (wart)
E. * Bartholin duct cyst
277. Cyst resulting from incomplete regression of the wolffian duct during fetal development?
A. Vaginal cancer
B. * Gartner duct cyst
C. Fibroid
D. Fissure/ Fistula
E. Bartholin cyst
278. Leiomyomata uteri (fibroids) are a common cause of?
A. Vaginal itching
B. * Heavy menstrual bleeding
C. Ectopic pregnancy
D. Severe pelvic and leg pain
E. Vaginal discharge (yellowish)
279. Which of the following is correct regarding polyps?
A. Polyps are the most common cysts in the fallopian tubes
B. Polyps are similar to the fistula
C. Polyps causing secondary amenorrhea but only age after 52y
D. Polyps are always associated with pain
E. * Polyps are not associated with pain
280. Choose the most common benign neoplasm of the female genital tract:
A. Endometrial cancer
B. Cervical cancer
C. Vagina cancer
D. * Uterine leiomyoma (fibroids)
E. Polyps
281. Which of the following describes the uterine leiomyoma?
A. * Discrete/ round ,firm
B. Attached, triangle-like, dense
C. Single, huge, full of pus, dense
D. Cuboid-shaped, pinkish-color, firm
E. Cylendric-chain looking, irregular borders

282. Indicate the key words about uterine myomas and their effects on body:
A. * On physical exam uterus is irregular and mobile
B. On physical exam uterus is innervated and shrinky
C. There is lots of yellowish-brownish discharge
D. Heavy bleeding, leading to anemia (only at age <35.y.)
E. None of them
283. If a uterine mass continue to grow after menopausa, suspect……?
A. PID (Pelvic inflammatory disease)
B. Chronic abscess
C. * Malignancy
D. Fistula
E. Normal, will be self-limited
284. What for the Gardasil vaccine may be used?
A. Endometrial cancer
B. Types of breast cancer
C. For protecting the recurrent miscarriage
D. * Genital warts and cervical cancer
E. Reduces the development of ovarian cancer in young women
285. Indicate the explanation for the cystocele:
A. * Bladder prolapse into the vagina
B. Bladder prolapse under the diaphragm
C. Bladder rupture
D. Fistula in the vaginal canal
E. Partially rupture of the bladder
286. Indicate the types of incontinence:
A. Stress-mechanical
B. Stress-physiological
C. Urge-mechanical
D. Urge-neurological
E. * Stress-urge
287. Which of the following is NOT a risk factor for the cervical cancer?
A. Multiple or high risk sexual partners
B. Tobacco use
C. * Being at menopausal age
D. History of Sexual Transmitted Disease
E. Hpv 16.18
288. Which of the following is true about granulosa cell tumor?
A. Cells in rosette pattern
B. Complex ovarian mass
C. High estradiol and high inhibin
D. Sex cord-stromal tumor
E. * All of them
289. Which among these gynecologic neoplasms has the highest mortality rate?
A. Uterine
B. * Ovarian
C. Cervical

D. Endometrial
E. Vulvar
290. Indicate the types of endometrial cancer:
A. Benign endometroid – atypical endometrial
B. * Type 1, endometroid - Type 2, serous
C. Type 1 serous – Type 2, endometroid
D. Columnar - squamous
E. Type A, fibroid, Type B, serous
291. Indicate the risk factor of vulvar cancer:
A. Alcoholism-down syndrome
B. Sjogren syndrome
C. * HPV (16,18,31), lichen sclerosis, smoking
D. Diabetes, allergy, high protein diet
E. Stress(anxiety), anorexia nervosa
292. Choose the frequency of female genital tract cancers:
A. * Endometrial>ovarian>cervical
B. Ovarian>endometrial>cervical
C. Cervical>ovarian>endometrial
D. Endometrial>cervical>ovarian
E. Ovarian>cervical>endometrial
293. Choose the number of death rate with female genital tract cancers:
A. Endometrial>ovarian>cervical
B. * Ovarian>endometrial>cervical
C. Cervical>endometrial>ovarian
D. Ovarian>cervical>endometrial
E. Endometrial>cervical>ovarian
294. Indicate which of the following best explains teratoma:
A. A tumor very soft, round, filled with clean pus
B. A tumor very large, firm, filled in with blood
C. An abscess with a pinkish surface
D. An abscess covering the os of cervix
E. * A tumor made up of hair, muscle, bone
295. Explain the term called, serous cystadenoma:
A. * Most common ovarian neoplasm, and is benign
B. Most common ovarian neoplasm, highly aggressive
C. Most common uterus neoplasm
D. Cervical hyperplasia
E. Large cyst under the one ovary
296. What is a Brenner tumor?
A. * Group of ovarian neoplasm, majority benign
B. Group of ovarian neoplasm, causing rapid metastasis
C. This tumor is very common in young women
D. The tumor is made up of muscle, hair and bone
E. The tumor is firm-small in size <1cm
297. Explain correctly the krukenberg tumor:
A. Benign tumor of ovaries

B. Benign tumor of uterus


C. Benign tumor on labia major
D. * Malignancy in the ovary that metastasized from other site
E. Malignancy in the cervix that metastasized from primary site
298. Explain what the sertoli-leydig cell tumor is:
A. A small tumor on labia minor
B. A small tumor on posterior wall of vagina
C. A tumor on the opening of the cervix
D. * Is an ovarian tumor that secrets testosterone
E. Is an ovarian tumor with LH overproducing
299. Thick cottage-cheese-like discharge and vaginal inflammation is suggestive of?
A. Cancer
B. Hormonal changes
C. * Candida vaginitis
D. Bacterial vaginitis
E. Trichomoniasis
300. Fitz-Hugh-Curtis syndrome (perihepatitis) mostly caused by:
A. HBV
B. * Chlamydia and gonorrhea
C. E.coli
D. Candida
E. Streptococci
Назва наукового напрямку (модуля): Семестр: 11
Акушерство та гінекологія Ситуаційні задачі
Опис:
6 курс
Перелік питань:
1. The woman is admitted to the maternity home with discontinued uterine contractions and slight
bloody discharges from the vagina. The condition is severe, skin is pale, and consciousness is
confused. BP – 80/40 mm Hg. Fetal heart rate of the fetus is absent. Lover uterine segment is
painful. There was a cesarean section one year ago. What is the clinical diagnosis:
A. Placental abruption
B. Placental presentation
C. * Uterine rupture
D. Premature expulsion of the amniotic fluids
E. Couveler uterus
2. 25-years old pregnant woman at 32 weeks of gestation is admitted to the hospital with complaints
of bloody discharge like spotting. Placenta previa is diagnosed during ultrasonography. The
uterine tone is normal, fetal heart rate is 136 beats per minute. What is the best management of
the pregnant woman?
A. Induction of labor by prostaglandins
B. Blood transfusion
C. Induction of labor by oxytocin
D. Cesarean section.
E. * Tranexamic acid prescription, female monitoring for the intensity of hemorrhage and fetal
wellbeing, dexamethasone administration
3. Pregnant N., 25 years is delivered in the maternity department with complaints of periodic pain in
lower part of abdomen and lumbar region. Bloody discharge has appeared one hour before. Fetal
heart rate is 136 in 1min. At vaginal examination: the uterine cervix is effaced, 6-7 cm dilated.
Spongy tissue is palpated laterally. Blood loss is 250 ml and continues. Your management.
A. Stimulation of labor
B. Amniotomy
C. Obstetric forceps
D. The fetal destroying operation
E. * Cesarean section
4. Sharp pain in the uterine fundus with profuse bleeding has appeared in the in primapara during the
first stage of labor. Uterus is in hypertonus. Fetal heart rate is 200 beats per minute. Uterine
cervix is effaced, and 4 cm dilated on vaginal examination. Blood was presented in amniotic fluid
during amniotomy. Your management.
A. Labor induction
B. Treatment of fetal distress
C. * Cesarean section immediately
D. Tocolytics prescription
E. Coagulants prescription
5. Sharp pain in the uterine fundus with profuse bleeding has appeared in the in primapara during the
first stage of labor. Uterus is in hypertonus. Fetal heart rate is 200 beats per minute. Uterine
cervix is effaced, and 4 cm dilated on vaginal examination. Blood was presented in amniotic fluid
during amniotomy. Your diagnosis.
A. Low lying placenta
B. Placenta previa
C. * Placenta abruption
D. Hypertonic dysfunction
E. DIC-syndrome
6. Uterine contractions have stopped suddenly in 25 years old patient after excessive uterine
contractions. Vasten sign is positive. Bloody excretions are presented. Fetal heart rate is absent.
The condition of patient suddenly became worse, BP went down to 70 mm Hg, pulse 140 in a 1
minute, a skin is pale. What is the reason such condition?
A. Threatened rupture of uterus
B. Placenta abruption
C. * Uterine rupture
D. Couveler uterus
E. Placenta previa
7. Postpartum patient 28 years. 3800 grams girl was born. Placenta was delivered as a result of
active management of the placenta. Bleeding continues. Cervical and vaginal lacerations are absent.
Ruptured vessels are presented during inspection of the placenta. What is the management of this
situation?
A. * To perform manual exploration of uterine cavity
B. Total hysterectomy
C. Contractile drugs prescription
D. External massage of uterus
E. Antishock garment application
8. 25 years old woman is admitted to the hospital on a 38 week of pregnancy with regular uterine
contractions and bloody discharge from vagina. The uterine cervix is effaced and 4 cm dilated, soft
spongy tissue is presented. Bleeding increased in vaginal examination. What is the reason of
bleeding increasing?
A. Amniotic fluid embolism
B. Bleeding from the rupture of uterine cervix
C. Premature removing of the normally located placenta
D. * Complete placenta previa
E. DIC - syndrome
9. Patient at 39 week of gestation is admitted to the hospital with complaints of sudden acute pain in
upper part of abdomen, dizziness, bleeding from vagina. The uterine contractions are regular.
Skin and visible mucus membranes are pale, BP 80/50 mm Hg, pulse 126 per minute. Painful
infiltrate is palpated in the fundal area. Fetal heart rate is 170, arrhythmic. At vaginal examination
the uterine cervix is effaced and 5 cm dilated, amniotic membranes are presented, parts of placenta
are not determined. Fetal head is at -2 station. Bloody discharges are visible. What is the most
likely diagnosis?
A. Uterine rupture
B. * Placenta abruption
C. Placenta previa
D. Hypotonic uterine contraction
E. The uterine tetanus
10. Patient at 39 week of gestation is admitted to the hospital with complaints of acute pain in lower
abdomen, dizziness, and bloody discharge from vagina. The uterine contractions are regular every5
minutes by 40 seconds, amniotic membranes are intact. Objectively: skin and visible mucus
membranes are pale, BP 80/50 mm Hg, pulse is 126 beats per minute. Uterus is dense. Painful
infiltrate is determined on the fundus. Fetal heart rate is 170, arrhythmic. At vaginal examination:
cervix is effaced and dilated to 4cm, amniotic membranes are presented, parts of placenta are not
determined, fetal head of is at -2 station. Bleeding discharge without clots is presented. What
is the management of this situation?
A. Therapeutic rest
B. * Cesarean section

C. Oxytocin prescription
D. Fetal destroying operation
E. Uterine curettage
11. Multipara. Uterine contractions occur every 4-5 hours by 25 seconds. Bloody excretions began at
once after appearance of contractions. Fetal heart rate is 100-110 beats per minute . Uterine
cervix is effaced and dilated till 6 cm, placenta tissue is presented totally. Diagnosis?
A. Threatening rupture of uterus.
B. Couveler uterus
C. Partial placenta previa
D. * Complete placenta previa.
E. Placenta abruption
12. Multipara. Uterine contractions occur every 4-5 hours. Bloody excretions began at once after
appearance of contractions. Fetal heart rate 100-110 in min. At vaginal examination the uterine
cervix is effaced and dilated till 6 cm. In the cervix placental tissue is determined. What will be
adequate management?
A. Therapeutic rest
B. * Cesarean section
C. Oxytocin prescription
D. Fetal destroying operation
E. Uterine curettage
13. Labor started at 39 weeks of pregnancy. There was one induced abortion, which was complicated
by endometritis in the past. At active management of labor placenta doesn’t separated.
Considerable bloody excretions appeared at an attempt to do the manual separation of placenta. The
blood loss is 600 ml. An attempt of manual separation of placenta from the uterus was not
successful. What is the doctor’ management?
A. * Hysterectomy
B. Uterine curettage
C. To continue the manual separation of placenta
D. Vasoconstrictors prescription
E. Contractile drugs prescription
14. Massive hypotonic uterine bleeding began after labor by dead child in postpartum period. The
blood loss was reached to 1600ml. Venous bleeding was presented. Thrombocytes level was 80
x10 9/l. There were hemorrhages from the places of injections. Which stage of DIC-syndrome
was developed at postpartum patient?
A. I
B. * III
C. II
D. V
E. IV
15. The girl by 3100g weight was delivered in 35 years old patient. During active management of
labor placenta doesn’t separated from uterine cavity. What is the most probable diagnosis?
A. Hypertonus of uterus
B. Placenta previa
C. * Placenta accreta
D. Uterine rupture
E. Uterine atony
16. The boy by 3700g was delivered in 36 years old woman. During active management of labor
placenta doesn’t separated from uterine cavity. What is the most probable diagnosis?
A. Hypertonus of uterus

B. Placenta previa
C. * Abnormal placenta adherence
D. Uterine rupture
E. Uterine atony
17. The boy by 3300g was delivered in 36 years old woman. During active management of labor
placenta doesn’t separated from uterine cavity. What is the most probable diagnosis?
A. Hypertonus of uterus
B. Placenta previa
C. * Placenta increta
D. Uterine rupture
E. Uterine atony
18. The boy by 3300g was delivered in 36 years old woman. During active management of labor
placenta doesn’t separated from uterine cavity. What is the most probable diagnosis?
A. Hypertonus of uterus
B. Placenta previa
C. * Placenta percreta
D. Uterine rupture
E. Uterine atony
19. 42 years old patient was admitted to the maternity hospital in 38 weeks of gestation with bloody
discharge like spotting. Complete placenta previa was revealed at ultrasound. Uterine cervix is
closed in vaginal examination. What is the adequate management of labor?
A. Augmentation of labor
B. * Cesarean section
C. Fetal destroying operation
D. Obstetric forceps application
E. Vacuum extraction
20. 30-years old edematous patient is admitted to the hospital in the second stage of labor. Fetal head is
in the pelvic outlet. Profuse hemorrhage have appeared suddenly. Fetal heart rate is 80 beats per
minute. What is the adequate management of labor?
A. Cesarean section
B. Fetal heart rate monitoring
C. * Obstetric forceps
D. Episiotomy
E. Induction of labor
21. 30-years old edematous patient is admitted to the hospital in the second stage of labor. Fetal head
is in the pelvic outlet. Profuse hemorrhage have appeared suddenly. Fetal heart rate is 80 beats
per minute. What is the most probable diagnosis?
A. * Placenta abruption
B. Placenta previa
C. Uterine atony
D. Couveler uterus
E. Placenta increta
22. The bleeding began right after childbirth. The blood loss is 300 ml. In active management of third
stage of labor placenta is nor separated. Signs of the placental separation are negative. What is the
most probable diagnosis?
A. * Subtotal placenta adherens
B. Total placenta adherens
C. Couveler’s uterus

D. Placenta previa
E. Placenta abruption
23. The bleeding began right after childbirth. The blood loss is 550 ml. What is the most appropriate
management?
A. * Manual separation of placenta and exploration of the uterine cavity
B. Prescription of contractile drugs
C. Total hysterectomy
D. Uterine curretage
E. Uterine artery ligation
24. On the 6 day of the postpartum period a 26 years-old woman complaints of profuse bleeding from
vagina. Pelvic examination reveals 23-24 weeks increased uterus with clots inside. During
examination bloody discharge increases. Diagnosis?
A. Total placenta adherens
B. Subtotal placenta adherens
C. Couveler’s uterus
D. * Late postpartum hemorrhage
E. Placenta previa
25. On the 6 day of the postpartum period a 26 years-old woman complaints of profuse bleeding from
vagina. Pelvic examination reveals 22-23 weeks increased uterus with clots inside. During
examination bloody discharge increases. What is the most appropriate management of this
A. * situation?
Uterine curettage
B. Uterine artery ligation
C. Total hysterectomy
D. Manual exploration of the uterine cavity
E. Prescription of contractile drugs
26. 4500g infant was delivered in 42 years old patient. Placenta was delivered by active management.
All membranes and parts of the placenta were presented during inspection. Blood loss was 350
ml. After 5 min bloody excretions increased. At external uterine massage uterus did not contract
well, was soft and large. Diagnosis?
A. Placental abruption
B. * Atonic bleeding
C. Uterine rupture
D. Couveler’s uterus
E. Rupture of the cervix
27. First pregnancy 35 week of gestation. Woman complaints of spotting from the vagina. Blood
loss is 50 ml. Uterine tone is normal. Fetal heart rate is clear, rhythmic, 136 beats per minute. Head
of the fetus is mobile above the pelvic inlet. Uterine cervix is closed. What is the initial
management of the patient?
A. Bed rest
B. Cesarean section immediately
C. Amniotomy
D. * Bed rest, hemostatic agents
E. Bed rest, adequate tokolysis
28. First pregnancy 32week of gestation. Woman complaints of spotting from the vagina. Blood loss
is 50 ml. Uterine tone is normal. Fetal heart rate is clear, rhythmic, 136 beats per minute. Head of
the fetus is mobile above the pelvic inlet. Uterine cervix is closed. What is the initial management
of the patient?
A. Bed rest
B. Cesarean section immediately

C. Amniotomy
D. * Bed rest, hemostatic, corticosteroids prescriptions
E. Bed rest, adequate tokolysis
29. First pregnancy 33 week of gestation. Woman complaints of spotting from the vagina. Blood
loss is 50 ml. Uterine tone is normal. Fetal heart rate is clear, rhythmic, 136 beats per minute. Head
of the fetus is mobile above the pelvic inlet. Uterine cervix is closed. What is the aim of
corticosteroids prescription?
A. Anti-inflammatory
B. Hemostatic
C. Sedative
D. * Prevention of respiratory distress syndrome
E. Adequate tokolysis
30. Massive postpartum bleeding has developed in 34 years old patient with twins. Placenta is intact.
Vagina and perineum are without lacerations. Uterine fundus is 5cm above the umbilicus, soft.
What is the most likely reason of bleeding?
A. Couveler uterus
B. Uterine rupture
C. * Uterine atony
D. Placenta abruption
E. DIC - syndrome
31. Patient is admitted to the hospital with regular uterine contractions. During hospitalization the
condition is satisfactory, pulse - 84 in 1 min, BP - 150/90 and 160/90 mm of Hg. Suddenly patient
complains of severe pain in abdomen, general weakness, and dizziness. Pulse - 120 in min, AT -
80/40 and 90/45 mm Hg. Uterus is tense, very painful. Fetal heart rate is not auscultated. Uterine
cervix is effaced, 5 cm dilated at vaginal examination. Amniotic membrane is whole, tense. The
head is in the pelvic inlet. Insignificant bloody discharge has appeared from vagina. What is the
most likely diagnosis?
A. Uterine rupture
B. * Placental abruption
C. Amniotic fluid embolism
D. Molar pregnancy
E. Placenta previa
32. Considerable bloody discharge has appeared at 30week of gestation in 35 years old patient. Total
placenta previa is diagnosed during ultrasonography. General blood loss is 500 ml and bleeding
continues. What is the management?
A. Tocolitics prescription
B. Cervical cerclage
C. * Cesarean section immediately
D. Hemostatics
E. Vaginal delivery
33. The bleeding began right after childbirth. The blood loss was 300 ml. In attempt to remove placenta
during traction it did not separate. Doctors’ tactics.
A. Cesarean section
B. * Manual removal of placenta
C. Cerclage
D. Total hysterectomy
E. Ligation of uterine vessels

34. 28 years old pregnant women complains of vaginal bleeding at 40 weeks. Fetal heart rate is 140
BPM. The results of the obstetrics examination: cervix is incompletely effaced, 3 cm dilated, the
placental tissue is palpated. Diagnosis?
A. Abnormal placenta adherence
B. * Complete placenta previa
C. Uterine atony
D. Uterine rupture
E. Placental abruption
35. 28 years old pregnant women complaints of vaginal bleeding at 40 week of gestation. Fetal heart
rate is 140 BPM. The results of the obstetrics examination: cervix is incompletely effaced, 4 cm
dilated, the placental tissue is palpated. Management?
A. * Cesarean section
B. Manual removal of placenta
C. Cerclage
D. Total hysterectomy
E. Ligation of ovarian vessels
36. 29 years old woman at 39 weeks of pregnancy complaints of small amount of bloody discharge till
50 ml and regular uterine contractions which have been started 5 hours ago. Fetal head rate is 132
per minute with satisfactory characteristics. Cervix is dilated till 5 cm in obstetric examination The
amniotic sac is present. Fetal head is in plane of pelvic inlet. The edge of the placenta lies adjacent
to the internal os. Diagnosis?
A. Abnormal placenta adherence
B. Complete placenta previa
C. Uterine atony
D. * Incomplete placenta previa
E. Placental abruption
37. 30 years old primapara at 39 week of pregnancy complaints of small amount of bloody discharge
and regular uterine contractions which have been started 5 hours ago. Fetal head rate 132 per
minute with satisfactory characteristics. At obstetric exam cervix is dilated till 2 cm. The amniotic
sac is present. Fetal head is in plane of pelvic inlet. The edge of the placenta lies adjacent to the
internal os. Blood loss is 300 ml and continues. What is the best management of labor?
A. * Cesarean section
B. Manual removal of placenta
C. Amniotomy, oxytocin prescription
D. Total hysterectomy
E. Ligation of ovarian vessels
38. 21 years old, primapara at 38 week of pregnancy complaints of profuse bleeding till 400ml.
Contractions are regular. At obstetric exam cervix is dilated till 4 cm. The amniotic sac is present.
Fetal head is in plane of pelvic inlet. The leading edge of the placenta is 2 cm from the internal
cervical os. What is the best management of labor?
A. * Cesarean section
B. Manual removal of placenta
C. Amniotomy, oxytocin prescription
D. Total hysterectomy
E. Ligation of ovarian vessels
39. 34 years old, primapara at 37 week of pregnancy complaints of small amount of bloody discharge.
Contractions are irregular. Fetal heart rate is 132 per minute with satisfactory characteristics. At
obstetric exam cervix is closed. The leading edge of the placenta is 1 cm from internal cervical os.
What is the management of the pregnancy?

A. * Cesarean section
B. Manual removal of placenta
C. Amniotomy, oxytocin prescription
D. Total hysterectomy
E. Ligation of ovarian vessels
40. What is the first step in postpartum hemorrhage treatment?
A. * Call for help
B. Insert intravenous access
C. Recognize circulatory problem. Administer for face oxygen mask
D. Abdominal examination to confirm uterus well contracted
E. Vaginal examination for laceration
41. 30 years old undergoes spontaneous delivery of a 3900 g boy. 10 units of oxytocin were
administered. After successful umbilical cord traction placenta was delivered. Placental tissue is
expelled with umbilical cord, but vaginal hemorrhage ensues immediately thereafter. The placenta
is clearly not intact. What are appropriate immediate interventions in this situation?
A. Total hysterectomy
B. Subtotal hysterectomy
C. Uterine curettage
D. * Manual exploration of uterine cavity
E. External uterine massage
42. 30 years old undergoes spontaneous delivery of a 3900 g boy. 10 units of oxytocin were
administered. After successful umbilical cord traction placenta was delivered. Placental tissue is
expelled with umbilical cord, but vaginal hemorrhage ensues immediately thereafter. The placenta
is clearly not intact. What is the most probable diagnosis?
A. Placenta accreta
B. Placenta increta
C. Placenta percreta
D. * Retained placenta tissue
E. Placenta previa
43. A 30-year-old pregnant was admitted to a maternity hospital with interm pregnancy. She complains
of severe pain in the uterus that started 1 hour ago, nausea, vomiting, cold sweat. Cesarean section
was performed 2 years ago. Uterine contractions have stopped. Skin and mucous membranes are
pale. Heart rate is 100/min. BP is 90/60 mm Hg. Uterus is sharply painful. Fetal heart rate is not
auscultated. Moderate bloody discharge from the uterus is observed. Uterus cervix is 4 cm open.
Presenting part is not palpated. The most probably diagnosis is:
A. * Uterine rupture
B. Placenta previa
C. Placenta abruption
D. Premature separation of normally localized placenta
E. Compression of inferior pudendal vein
44. 32 years old multipara at 40 week of pregnancy complaints of vaginal bleeding like spotting.
Contractions are every 3-4 minutes by 30 seconds. Cervix is dilated to 6 cm during vaginal
examination. Amniotic membranes are intact. Fetal head is in the pelvic inlet. The leading edge of
the placenta is 3 cm from the internal cervical os at ultrasonography. What is the best management
of labor?
A. Cesarean section
B. Manual removal of placenta
C. * Amniotomy, oxytocin prescription
D. Total hysterectomy

E. Ligation of ovarian vessels


45. 29 years old primapara at 38 week of pregnancy is admitted to the hospital with complaints of
bloody discharge, general weakness and regular uterine contractions which have been started 5
hours ago. In examination pulse is 100 beats per minute, BP – 100/60 and 95/55 mm of Hg. Uterus
is tense, very painful in the fundal area. Fetal heart rate is not auscultated. Uterine cervix is effaced,
5 cm dilated at vaginal examination. Amniotic membranes are intact. The head is in the pelvic
inlet. Blood loss reaches 1000ml. Which degree of hemorrhagic shock is presented in the patient?
A. * I
B. II
C. III
D. IV
E. V
46. 29 years old primapara at 38 week of pregnancy is admitted to the hospital with complaints of
bloody discharge, general weakness and regular uterine contractions which have been started 5
hours ago. In examination pulse is 100 beats per minute, BP – 100/60 and 95/55 mm of Hg. Uterus
is tense, very painful in the fundal area. Fetal heart rate is not auscultated. Uterine cervix is effaced,
5 cm dilated at vaginal examination. Amniotic membranes are intact. The head is in the pelvic
inlet. Blood loss reaches 1000ml.What is the general amount of infusion therapy should be
A. prescribed?
500ml
B. 1000ml
C. 1500ml
D. 2000ml
E. * 2500ml
47. 34 years old primapara at 40 week of pregnancy is admitted to the hospital with profuse vaginal
bleeding, loss of consciousness. In examination pulse is 120 beats per minute, BP – 80/60 and
75/55 mm of Hg. Uterus is tense, very painful in the fundal area. Fetal heart rate is not auscultated.
Uterine cervix is effaced, 6 cm dilated at vaginal examination. Amniotic membranes are intact.
The head is in the pelvic inlet. Blood loss reaches 1500ml. Which degree of hemorrhagic shock is
presented in the patient?
A. I
B. * II
C. III
D. IV
E. V
48. 40 years old primapara is presented on 42-43 week of pregnancy. Uterine contractions are weak.
Fetal head is arrested to pelvic inlet. Vasten sign is positive. Fetal heart rate is 140 bmp, rhythmic.
Cervical dilation is 6 cm. Cranial bones are dense, sagittal suture is not palpated. Which
complication is presented?
A. * Cephalo-pelvic disproportion
B. Fetal distress
C. Uterine inertia
D. False labor
E. Hypertonic uterine dysfunction
49. 40 years old primapara is presented on 42-43 week of pregnancy. Uterine contractions are weak.
Fetal head is arrested to pelvic inlet. Vasten sign is positive. Fetal heart rate is 140 bmp, rhythmic.
Cervical dilation is 7 cm. Cranial bones are dense, sagittal suture is not palpated. What is the reason
of labor complication?
A. * Postterm pregnancy
B. Deflexed presentation
C. Breech presentation

D. Multiple pregnancy
E. Polyhydramnios
50. 39 years old primapara at 40 week of pregnancy is admitted to the hospital with complaints of
regular uterine contractions every 2- 3minutes by 45 seconds. Longitudinal fetal lie, cephalic
presentation was diagnosed. Vasten sign is positive. Fetal heart rate is 142 in 1min, clear,
rhythmic. The sizes of pelvis are normal. Probable fetal weight is 4200 g. Uterine cervix is dilated
to 8 cm, edematous. Amniotic membranes are absent. Fetal head is above the pelvic inlet.
Promontorium is not reached. What is the adequate management of labor?
A. * Immediate cesarean section
B. Fetal destroying operation
C. Medical treatment
D. Augmentation of labor
E. Spasmolytic prescription
51. 39 years old primapara at 40 week of pregnancy is admitted to the hospital with complaints of
regular uterine contractions every 2- 3minutes by 45 seconds. Longitudinal fetal lie, cephalic
presentation were diagnosed. Vasten sign is positive. Fetal heart rate is 142 in 1min, clear,
rhythmic. The sizes of pelvis are normal. Probable fetal weight is 4200 g. Uterine cervix is dilated
to 8 cm, edematous. Amniotic membranes are absent. Fetal head is above the pelvic inlet.
Promontorium is not reached. Which of the below pelvic sizes are considered to be normal for such
A. woman?
20-22-24-29cm
B. 22-22-24-28cm
C. * 25-28-30-20cm
D. 28-28-30-20cm
E. 23-24-25-26cm
52. 37 years old primapara at 41 week of pregnancy is admitted to the hospital with complaints of
regular uterine contractions every 2- 3minutes by 45 seconds. Longitudinal fetal lie, cephalic
presentation were diagnosed. Vasten sign is positive. Fetal heart rate is 142 in 1min, clear,
rhythmic. The sizes of pelvis are normal. Probable fetal weight is 4200 g. Uterine cervix is dilated
to 8 cm, edematous. Amniotic membranes are absent. Fetal head is above the pelvic inlet.
Promontorium is not reached. Positive Vasten sign suggest about:
A. Adequate cervical dilation
B. * Cephalo-pelvic disproportion
C. Uterine inertia
D. Uterine rupture
E. Hypertonic uterine dysfunction
53. M., 22 years old, nullipara. Full term of pregnancy. Labor has been started 8 hours ago. The
membranes ruptured are intact. Pelvic sizes: 25,28,31,20 cm. Fetal heart rate is 140 per minute with
satisfactory characteristics. Per vaginum: the cervix is 8 cm dilated. The amniotic sac is persent.
Fetal head is in the plane of inlet. Face line is in the right oblique size, the chin is palpated near
sacral region of the symphysis. Which complication is presented in the patient?
A. General contracted pelvis
B. * Cephalopelvic disproportion
C. Fetal distress
D. Primary uterine inertia
E. Secondary uterine inertia
54. M., 25 years old, nullipara. Full term of pregnancy. Labor has been started 9 hours ago. The
membrane is intact. Pelvic sizes: 25,28,31,20 cm. Vasten sign is positive. Fetal heart rate is 140 per
minute with satisfactory characteristics. Cervix is 9 cm dilated in vaginal examination. The
amniotic sac is presented. Fetal head is in the plane of inlet. Face line is in the right oblique size,
the chin is palpated near sacral region of the symphysis. What is the reason of cephalopelvic
disproportion?

A. * Face anterior presentation


B. General contracted pelvis
C. Face posterior presentation
D. Sinciput vertex presentation
E. Brow presentation
55. Primipara N., 25 years old. Delivery at term. The labor started 6 hours ago. The membranes
ruptured 1 hour ago. Pelvic sizes: 23,26,29,19 cm. Solovjov index is 15cm. Fetal heart rate is 140
per minute with satisfactory characteristics. Uterine contractions occur every 4-5 minutes. The
probable fetal weight by Volskov is 4200 g. Uterine cervix dilatation is 7 cm. The amniotic sac is
absent. Fetal head is -1 station. Sagittal suture is in the right oblique size. Small and large
fontanels are palpated. The large fontanel is under the symphysis. Which degree of pelvic
contraction is presented in woman?
A. * I
B. II
C. III
D. IV
E. V
56. Primipara N., 25 years old. Delivery at term. The labor started 6 hours ago. The membranes
ruptured 1 hour ago. Pelvic sizes: 23,26,29,19 cm. Solovjov index is 15cm. Fetal heart rate 140
per minute with satisfactory characteristics. Uterine contractions occur every 4-5 minutes. The
probable fetal weight by Volskiy is 4200 g. Uterine cervix dilatation is 8 cm. The amniotic sac is
absent. Fetal head is -1 station. Sagittal suture is in the right oblique size. Small and large
fontanels are palpated. The large fontanel is under the symphysis. Which complication is presented
in labor?
A. * Cephalopelvic disproportion
B. Uterine inertia primary
C. Uterine inertia secondary
D. Uterine rupture
E. Discoordinative uterine activity
57. 24 years old woman is presented in postpartum period after physiological delivery. At objective
examination her temperature is 36, 8 0C, Ps - 72/min, BP - 120/80 mm Hg. Mammary glands are
moderately swollen, nipples are clean without fissures. Lactation is characterized by colostrum.
Abdomen is soft and painless. Uterine fundus height is 3 fingers below the umbilicus. Lochia are
bloody, moderate. The most appropriate day for postpartum period would be:
A. * 3 day
B. 2 day
C. 1 day
D. 5 day
E. 4 day
58. 28 years old woman is presented in postpartum period after physiological delivery. At objective
examination her temperature is 36, 8 0C, Ps - 84/min, BP - 120/80 mm Hg. Mammary glands are
moderately swollen, nipples are clean without fissures. Lactation is characterized by immature
milk. Abdomen is soft and painless. Uterine fundus height is in the midway between symphysis and
umbilicus. Lochia are bloody-serous, moderate. The most appropriate day for postpartum period
would be:
A. 3 day
B. 2 day
C. 1 day
D. * 5 day
E. 4 day

59. 25 years old woman is presented in postpartum period after physiological delivery at objective
examination her temperature is 36,7 0C, Ps - 72/min, BP - 110/70 mm Hg. Mammary glands are
soft, nipples are clean without fissures. Lactation is good. Abdomen is soft and painless. Uterine
fundus is located 8cm below umbilicus. The lochia should be:
A. Bloody
B. * Bloody-serous
C. Mucous
D. Purulent
E. Serous
60. Pregnant C., 26 years, entered maternity department with regular uterine contractions. Term of
pregnancy 39 weeks. Abdominal circumference - 126 cm, uterine fundus height – 41 cm. The fetal
lie is longitudinal, breech presentation. Uterus in normal tonus. The fetal heart rate is 130 in 1 min,
rhythmic. Vaginally: cervical dilatation is full, vagina is filled by an amniotic sac. During
examination about 5 L of amniotic fluid came out, buttocks are in the pelvic cavity. Diagnosis?
A. Large fetus. Breech presentation
B. Multifetal pregnancy. Incomplete presentation of the I fetus
C. * Franc breech presentation. Polyhydramnion
D. Polyhydramnion. Complete breech presentation
E. Polyhydramnion
61. The pregnant S. was admitted in pathologic of pregnant department. Pregnancy ІІ, 37 weeks.
Complaints about the gradual enlargement of abdominal sizes after viral infection. Circumference
of abdomen – 110 cm, uterine height – 36 cm. The fetal lie is longitudinal, cephalic presentation,
head of the rounded shape, dense, above the pelvic inlet. The fetal heart rate is clear, rhythmic 130
in 1 min. The diagnosis: chronic polyhydramnion. Which antibiotics of the first line used for
medical treatment?
A. Doxycycline
B. * Erythromycin
C. Amoxill
D. Gentamycin
E. Clyndamycin
62. Patient in 37 weeks of pregnancy was presented in female dispensary. An abdomen is enlarged due
to a pregnant uterus. Abdominal circumference - 122 cm, uterine height - 40 cm. Two great parts of
fetus are palpated to the right and to the left at the level of umbilicus. Presenting part is not
determined. The fetal heart rate is auscultating in many points at the level of umbilicus, 140 and
130 in 1 min, rhythmic. What is the diagnosis?
A. Fetal macrosomia
B. Polyhydramnion
C. Pregnancy and uterine fibromioma
D. Molar pregnancy
E. * Multifetal pregnancy
63. 122. The pregnant Х. in 32 weeks of pregnancy visited the doctor of female dispensary with
complaints about the increasing of abdominal volume after the acute infection a week ago. The
patient’ condition is satisfactory, the edema are absent. The abdominal circumference is 98 cm,
uterine fundus level – 36 cm. The fetal lie is longitudinal, the fetal head as in 32 weeks of
pregnancy, above the pelvic inlet, mobile. The fetal heart rate 120 in 1 min. What pathology is
A. presented?pregnancy
Multifetal
B. Chronic polyhydramnion
C. Breech position of fetus
D. Transversal position of fetus
E. * Acute polyhydramnion

64. 133. Primipara F., 24 years old. Multiply pregnancy at term. The labor started 6 hours ago. The
membranes are intact. Pelvic sizes: 25,28,31,21 cm. In Leopolds maneuvers – longitudinal lie of
both fetuses, breech presentation of the first fetus and cephalic – of the second one. Fetal heart rates
140 per minute with satisfactory characteristics. Uterine contractions occur every 7-8 minutes. Per
vaginum: the uterine cervix dilatation is 5 cm. The amniotic sac is absent. Buttocks of the first fetus
is presented. Which type of breech presentation is presented?
A. * Multiply pregnancy. The frank breech presentation of the first fetus.
B. Multiply pregnancy. Complete breech presentation of the first fetus.
C. Multiply pregnancy. Complete foot-ling presentation of the first fetus.
D. Multiply pregnancy. Incomplete foot-ling presentation of the first fetus.
E. Multiply pregnancy. Knee-ling presentation of the first fetus.
65. 134. Primipara F., 24 years old. Multiply pregnancy at term. The labor started 6 hours ago. The
membranes are intact. Pelvic sizes: 25,28,31,21 cm. In Leopolds Maneuvers – longitudinal lie of
both fetuses, breech presentation of the first fetus and cephalic – of the second one. Fetal heart rates
140 per minute with satisfactory characteristics. Uterine contractions occur every 7-8 minutes. Per
vaginum: the uterine cervix dilatation is 5 cm. The amniotic sac is absent. Buttocks of the first fetus
is presented. What is the management of labor?
A. Manual aid by Tsovianov II
B. * Cesarean section
C. Subtotal breech extraction
D. Classic manual aid
E. Total breech extraction
66. Pregnant N., 25 weeks of pregnancy. During the last 2 months complains of general weakness,
violation of taste, the promoted fragility of hair and nails. Uterine height is 20cm above the
symphysis. Laboratory findings: erythrocytes - 2,8x10 12/ L, Hb - 98 G/L. The most common
fetal complication would be:
A. Macrosomia
B. Avitaminosis
C. Izoimmunisation
D. Asphyxia
E. * Fetal growth retardation
67. 26 years old patient is presented in the second stage of labor. Uterine contractions are every 2
minutes by 45-50 seconds. Fetal lie is longitudinal, cephalic presentation is presented. Fetal heart
rate is 60 beats per minute. Cervical dilation is 10cm, amniotic fluid are absent, fetal head is
presented at +2 station at vaginal examination. The most likely diagnosis is:
A. Preterm labor
B. Uterine inertia
C. Preeclampsia
D. * Fetal distress
E. Breech presentation
68. 26 years old patient is presented in the second stage of labor. Uterine contractions are every 2
minutes by 45-50 seconds. Fetal lie is longitudinal, cephalic presentation is presented. Fetal heart
rate is 60 beats per minute. Cervical dilation is 10cm, amniotic fluid are absent, fetal head is
presented at +2 station at vaginal examination. The appropriate management is:
A. Augmentation of labor
B. Spasmolytics prescription
C. Corticosteroids prescriptions
D. * Obstetric forceps application
E. Cesarean section

69. 28 years old patient is presented in the second stage of labor. Uterine contractions are every 1-2
minutes by 50-55 seconds. Fetal lie is longitudinal, cephalic presentation is presented. Fetal heart
rate is 70 beats per minute. Cervical dilation is 10cm, amniotic fluid are absent, fetal head is
presented at +3 station at vaginal examination. The appropriate management is:
A. Augmentation of labor
B. Spasmolytics prescription
C. Corticosteroids prescriptions
D. * Vacuum application
E. Cesarean section
70. 32 years old patient is presented in the first stage of labor. Uterine contractions are every 4 minutes
by 25-30 seconds. Fetal lie is longitudinal, cephalic presentation is presented. Fetal heart rate is 60
beats per minute. Cervical dilation is 2 cm, amniotic fluid is absent, fetal head is presented at -2
station at vaginal examination. The appropriate management is:
A. Augmentation of labor
B. Spasmolytics prescription
C. Corticosteroids prescriptions
D. Obstetric forceps application
E. * Cesarean section
71. 36 years old patient is presented in the second stage of labor. Uterine contractions are every 2
minutes by 45-50 seconds. Fetal lie is longitudinal, cephalic presentation is presented. Fetal heart
rate is 60 beats per minute. Cervical dilation is 10cm, amniotic fluid is absent, fetal head is
presented at +1 station at vaginal examination. The most likely diagnosis is:
A. Preterm labor
B. Uterine inertia
C. Preeclampsia
D. * Fetal distress
E. Breech presentation
72. 39 years old patient is presented in the second stage of labor. Uterine contractions are every 1-2
minutes by 50-55 seconds. Fetal lie is longitudinal, cephalic presentation is presented. Fetal heart
rate is 70 beats per minute. Cervical dilation is 10cm, amniotic fluid is absent, fetal head is
presented at +1 station at vaginal examination. The appropriate management is:
A. Augmentation of labor
B. Spasmolytics prescription
C. Corticosteroids prescriptions
D. * Vacuum application
E. Cesarean section
73. 33 years old patient is presented in the second stage of labor. Uterine contractions are every 1-2
minutes by 50-55 seconds. Fetal lie is longitudinal, cephalic presentation is presented. Fetal heart
rate is 70 beats per minute. What you should do firstly?
A. Augmentation of labor
B. Spasmolytics prescription
C. Corticosteroids prescriptions
D. * Vaginal examination
E. Cesarean section
74. Patient F., 33 years old is admitted to Pathologic Pregnancy department at 36 week of gestation.
She complaints of general weakness, dizziness, increasing of fetal activity. Uterine fundus height is
situated at the level of the umbilicus. Fetal lie is longitudinal, cephalic presentation is diagnosed.
Fetal heart rate is 124 beats per minute. The most likely diagnosis is:
A. Multiple pregnancy

B. Oligohydramnios
C. Fetal hypoxia
D. Fetal distress
E. * Fetal growth retardation
75. Patient F., 33 years old is admitted to Pathologic Pregnancy department at 36 week of gestation.
She complaints of general weakness, dizziness, increasing of fetal activity. Uterine fundus height is
situated at the level of the umbilicus. Fetal lie is longitudinal, cephalic presentation is diagnosed.
Fetal heart rate is 124 beats per minute. Appropriate studies to be performed would be:
A. * Biophysical profile, umbilical cord Doppler
B. Fetal cells for karyotyping
C. Fetal echocardiography
D. Amniotic fluid alpha-fetoprotein
E. Cervical culture
76. Patient M., 35 years old is admitted to Pathologic Pregnancy department at 35 week of gestation.
She complaints of general weakness, increasing of fetal activity. Uterine fundus height is situated 2
cm above umbilicus. Fetal lie is longitudinal, cephalic presentation is diagnosed. Fetal heart rate
is 136 beats per minute. Biophysical profile test score is 10. Your Biophysical Profile is considered
A. * to be:
Normal
B. Decreased
C. Abnormal
D. Pathologic
E. Equivocal
77. Patient M., 35 years old is admitted to Pathologic Pregnancy department at 36 week of gestation.
She complaints of general weakness, increasing of fetal activity. Uterine fundus height is situated 3
cm above umbilicus. Fetal lie is longitudinal, breech presentation is diagnosed. Fetal heart rate is
136 beats per minute. Biophysical profile test score is 9. Your Biophysical Profile is considered to
A. * be:
Normal
B. Decreased
C. Abnormal
D. Pathologic
E. Equivocal
78. Patient L., 30 years old is admitted to Pathologic Pregnancy department at 30 week of gestation.
She complaints of general weakness, increasing of fetal activity. Uterine fundus height is situated 4
cm above umbilicus. Fetal lie is longitudinal, breech presentation is diagnosed. Fetal heart rate is
160 beats per minute. Biophysical profile test score is 8. Your Biophysical Profile is considered to
A. * be:
Normal
B. Decreased
C. Abnormal
D. Pathologic
E. Equivocal
79. Patient L., 30 years old is admitted to Pathologic Pregnancy department at 30 week of gestation.
She complaints of general weakness, increasing of fetal activity. Uterine fundus height is situated 4
cm above umbilicus. Fetal lie is longitudinal, breech presentation is diagnosed. Fetal heart rate is
160 beats per minute. Biophysical profile test score is 8. Your Biophysical Profile is considered to
A. * be:
Normal
B. Decreased
C. Abnormal
D. Pathologic
E. Equivocal

80. Patient S., 34 years old is admitted to Pathologic Pregnancy department at 32 week of gestation.
She complaints of weak fetal activity. Uterine fundus height is situated 4 cm above umbilicus. Fetal
lie is transverse. Fetal heart rate is 160 beats per minute. Biophysical profile test score is 6. Your
Biophysical Profile is considered to be:
A. Normal
B. Decreased
C. Abnormal
D. Pathologic
E. * Equivocal
81. Patient W., 29 years old is admitted to Pathologic Pregnancy department at 33 week of gestation.
She complaints of weak fetal activity. Uterine fundus height is situated 4 cm above umbilicus. Fetal
lie is transverse. Fetal heart rate is 160 beats per minute. Biophysical profile test score is 5. Your
Biophysical Profile is considered to be:
A. Normal
B. Decreased
C. Abnormal
D. Pathologic
E. * Equivocal
82. Patient W., 39 years old is admitted to Pathologic Pregnancy department at 34 week of gestation
with moderate degree of preeclampsia. She complaints of weak fetal activity. Uterine fundus height
is situated 4 cm above umbilicus. Fetal lie is longitudinal. Fetal heart rate is 160 beats per minute.
Biophysical profile test score is 4. Your Biophysical Profile is considered to be:
A. Normal
B. Decreased
C. * Abnormal
D. Pathologic
E. Equivocal
83. Patient K., 38 years old is admitted to Pathologic Pregnancy department at 35 week of gestation
with moderate degree of preeclampsia. She complaints of weak fetal activity. Uterine fundus height
is situated 5 cm above umbilicus. Fetal lie is longitudinal. Fetal heart rate is 160 beats per minute.
Biophysical profile test score is 3. Your Biophysical Profile is considered to be:
A. Normal
B. Decreased
C. * Abnormal
D. Pathologic
E. Equivocal
84. Patient A., 41 years old is admitted to Pathologic Pregnancy department at 36 week of gestation
with severe degree of preeclampsia. She complaints of weak fetal activity. Uterine fundus height is
situated 2 cm above umbilicus. Fetal lie is longitudinal. Fetal heart rate is 124 beats per minute.
Biophysical profile test score is 2. Your Biophysical Profile is considered to be:
A. Normal
B. Decreased
C. * Abnormal
D. Pathologic
E. Equivocal
85. Patient Z., 41 years old is admitted to Pathologic Pregnancy department at 36 week of gestation
with severe degree of preeclampsia. She complaints of weak fetal activity. Uterine fundus height is
situated 2 cm above umbilicus. Fetal lie is longitudinal. Fetal heart rate is 124 beats per minute.
Biophysical profile test score is 1. Your Biophysical Profile is considered to be:

A. Normal
B. Decreased
C. * Abnormal
D. Pathologic
E. Equivocal
86. Patient N., 33 years old is presented in the first stage of labor. Uterine contractions are every 3
minutes by 35-40 seconds. Fetal lie is longitudinal, cephalic presentation is diagnosed. Fetal heart
rate is 100 beats per minute. During fetal heart rate monitoring pathological decelerations have
been diagnosed. Cervical dilation is 4cm, amniotic membranes are intact, fetal head is presented at
-2 station at vaginal examination. Sagittal suture is located in the left oblique diameter of the pelvic
inlet; posterior fontanel is to the left posteriorly. Appropriate management of the patient is:
A. Metabolic drugs prescription
B. * Cesarean section immediately
C. Vaginal delivery
D. Biophysical profile
E. Umbilical cord Doppler
87. Patient N., 33 years old is presented in the first stage of labor. Uterine contractions are every 3
minutes by 35-40 seconds. Fetal lie is longitudinal, cephalic presentation is diagnosed. Fetal heart
rate is 100 beats per minute. During fetal heart rate monitoring pathological decelerations have
been diagnosed. Cervical dilation is 4cm, amniotic membranes are intact, fetal head is presented at
-2 station at vaginal examination. Sagittal suture is located in the left oblique diameter of the pelvic
inlet; posterior fontanel is to the left posteriorly. Your diagnosis:
A. * Fetal distress
B. Fetal hypoxia
C. Placental dysfunction
D. Fetal growth retardation
E. Placental insufficiency
88. Patient Q., 42 years old is admitted to Pathologic Pregnancy department at 33 week of gestation
with mild preeclampsia. Longitudinal lie, cephalic presentation is diagnosed. Fetal heart rate is 100
beats per minute. Bishop score cervical evaluation has 4 points. Blood flow is absent at umbilical
cord Doppler during 3 days monitoring. Corticosteroids have been prescribed for her. Biophysical
profile test score is 6. Appropriate management of the patient is:
A. Metabolic drugs prescription
B. * Cesarean section immediately
C. Vaginal delivery
D. Repeat umbilical cord Doppler within 24hours
E. Tocolytic prescription
89. Pregnant B, 20 years, complaints of weak fetal movement. Pregnancy 38 weeks. Uterine
contractions are absent. The fetal heart rate is 170 bpm. The estimation of the biophysical profile of
the fetus is 6 points. Appropriate management of the pregnant is:
A. Fetal distress therapy
B. Cesarean section immediately
C. * Hospitalisation, repeat the biophysical profile within 24 hours
D. Amniocentesis
E. Fetal heart rate monitoring

90. Primapara C., is presented in the first stage of labor which has been started 10 hours ago.
Suddenly she complains of releasing green colored amniotic fluid with meconium. Uterine
contractions are every 3 minutes by 35 seconds. Longitudinal lie, breech presentation is diagnosed.
Fetal heart rate is 90 beats per minute. Pathologic decelerations are presented during fetal heart rate
monitoring. Cervix is dilated to 4 cm at vaginal examination. Buttocks are presented. Appropriate
management of the patient is:
A. Forceps application
B. Vacuum extraction
C. Breech extraction
D. * Cesarean section
E. Fetal heart rate monitoring
91. Multipara with pelvis sizes 25-28-31-20 cm is presented in labor. Pushing efforts are every 2
minutes by 45 seconds. Longitudinal lie, cephalic presentation is diagnosed. The fetal hear rate is
green colored, arrhythmic 80 bpm. At vaginal examination the cervix is totally dilated, Amniotic
sac is absent. fetal head is presented in +2 station. Sagittal suture is in anteroposterior diameter of
the pelvic outlet. What is the best management of this situation?
A. Caesarean section
B. Vacuum extraction of the fetus
C. * Obstetric forceps
D. Conservative tactics of labor
E. Stimulation of the labor activity
92. 25 years old multipara woman is in the second labor for 12hours. Fetal lie is longitudinal, breech
presentation is presented. Heartbeat of fetus is arrhythmic, 80beats per min. Cervix of uterus is
completely dilated, fetal buttocks are on the pelvic floor. What is the best management of such
obstetric situation?
A. Use of obstetric forceps
B. Augmentation of labor
C. Cesarean section
D. Vacuum extraction
E. * Breech extraction
93. A 16-year-old nulligravid woman comes to the emergency department because of heavy vaginal
bleeding. She states that she normally has heavy periods every month but missed a period last
month and this period has been unusually heavy with the passage of large clots. She has no medical
problems, has no history of bleeding difficulties, and takes no medications. Her temperature is 37 C
(98.6 F), blood pressure is 110/70 mm Hg, pulse is 96/minute and respirations are 12/minute. Pelvic
examination shows a moderate amount of blood in the vagina, a closed cervix, and a normal uterus
and adnexae. Hematocrit is 30%. Urine hCG is negative. Which of the following is the most
appropriate management?
A. Expectant management.
B. Hysteroscopy
C. * Oral contraceptive pills
D. Laparoscopy
E. Laparotomy
94. A 12-year-old female comes to the physician because of a vaginal discharge. The discharge started
about 2 months ago and is whitish in color. There is no odor. The patient has no complaints of
itching, burning, or pain. The patient started breast development at 9 years of age and her pubertal
development has proceeded normally to this point. She has not had her first menses and she is not
sexually active. She has no medical problems. Examination is normal for a 12-year-old female.
Microscopic examination of the discharge shows no evidence of pseudohyphae, clue cells, or
trichomonas. Which of the following is the most likely diagnosis?

A. Bacterial vaginosis
B. Candida vulvovaginitis
C. * Physiologic leucorrhea
D. Syphilis
E. Trichomoniasis
95. A 34-year-old woman comes the physician because of lower abdominal cramping. The cramping
started 2 days ago. Examination is unremarkable except for a pelvic examination that reveals a
10-week sized uterus. Urine hCG is positive, and pelvic ultrasound reveals a 10-week intrauterine
pregnancy with a fetal heart rate of 160. The patient states that she is not sure whether to keep the
pregnancy. Which of the following is the most appropriate next step in management?
A. * Counsel the patient or refer to an appropriate counselor
B. Notify the patient's parents
C. Notify the patient's partner
D. Schedule a termination of pregnancy
E. Tell the patient that she is likely to have a miscarriage
96. A 67-year-old woman comes to the physician because of pain with urination and frequent urination.
She has hypertension for which she takes a beta-blocker, but no other medical problems. She states
that she is not sexually active. She does not smoke and drinks cranberry juice daily. Examination
shows mild suprapubic tenderness and genital atrophy but is otherwise unremarkable. Urinalysis
shows 50 to 100 leukocytes/high powered field (hpf) and 5 to 10 erythrocytes/hpf. Which of the
following is the most likely cause of the infection?
A. Cardiac disease
B. Cranberry juice ingestion
C. * Hypoestrogenism
D. Nephrolithiasis
E. Sexual intercourse
97. A 14-year old G0 adolescent reports menarche six months ago, with increasingly heavy menstrual
flow causing her to miss several days of school. Three months ago, her pediatrician started her on
oral contraceptives to control her menstrual periods, but she continues to bleed heavily. Her
previous medical history is unremarkable. The patient has a normal body habitus for her age.
Appropriate breast and pubic hair development is present. Her hemoglobin is 9.1 mg/dl, hematocrit
27.8%, urine pregnancy test negative. Which of the following etiologies for menorrhagia is most
likely the cause of her symptoms?
A. Uterine leiomyoma
B. Thyroid disorder
C. * Coagulation disorder
D. Endometrial hyperplasia
E. Chronic Endometritis
98. A 32 year-old G0 presents with amenorrhea for the last 3 months. She has a long history of
irregular cycles, 26 to 45 days apart, for the last two years. She is otherwise in good health and is
not taking any medications. She is sexually active with her husband and uses condoms for
contraception. She is 5 feet 4 inches tall and weighs 140 pounds. On exam, she has a slightly
enlarged, non-tender uterus. There are no adnexal masses. What is the most appropriate next step in
the management of this patient?
A. Perform a pelvic ultrasound
B. Check a TSH level
C. Check progesterone and estrogen levels
D. * Perform a urine pregnancy test
E. Check FSH and LH levels
99. A 20-year-old G1P0 woman has vaginal spotting and mild cramping for the last 3 days. She had her
last normal menstrual period approximately 6-1/2 weeks ago. She had a positive home pregnancy
test. Her medical and gynecologic histories are negative and non-contributory. On physical exam:
blood pressure 120/72; pulse 64; respirations 18; temperature 98.6°F (37°C). On pelvic exam, she
has scant old blood in the vagina, with a normal appearing cervix and no discharge. On bimanual
exam, her uterus is nontender and small, and there are no adnexal masses palpable. Pertinent labs:
Quantitative Beta-hCG is 750 mIU/ml 48 hours ago; today, the level is 760 mIU/ml; progesterone
level = 3.2 ng/ml; hematocrit is 37%. Transvaginal ultrasound shows a fluid collection in the uterus
and no fetal pole, no masses and no free fluid in the pelvis. Which of the following is the most
appropriate next step in the management of this patient?
A. Exploratory surgery
B. Treat with Methotrexate
C. Treat with Mifepristone
D. * Dilatation and Curettage
E. Repeat hCG level in one week
100. An obese 30 year-old G3P1 Asian woman undergoes an uncomplicated dilation and curettage for a
first trimester miscarriage. Pathology reveals a molar pregnancy. The patient’s medical history is
significant for chronic hypertension. She has a history of a previous uncomplicated term pregnancy,
and termination of a pregnancy at 16 weeks gestation for trisomy 18. What aspect of the patient’s
history places her at increased risk for a molar pregnancy?
A. Obesity
B. Previous history of fetal aneuploidy
C. Chronic hypertension
D. * Asian race
E. Prior term pregnancy
101. A 17 year-old G0 presents with a 3-year history of severe dysmenorrhea shortly after menarche at
age 14. Her menstrual cycles are regular with heavy flow. She has been treated with NSAIDs and
oral contraceptives for the last year without significant improvement. She misses 2-3 days of school
each month due to her menses. She has never been sexually active. Physical examination is
remarkable for Tanner: Stage IV breasts and pubic hair. Pelvic examination is normal, as is a pelvic
ultrasound. Both the patient and her mother are concerned. What is the next best step in the
management of this patient?
A. Empiric treatment with GnRH agonist
B. CT scan of the pelvis
C. * Diagnostic laparoscopy
D. MRI of the pelvis
E. Hysterosalpingography
102. A 22-year old G0 presents with 5 months of amenorrhea since discontinuing her oral contraceptive
pills. She had been on the pill for the last 6 years and had normal menses every 28 days while
taking them. She is in good health and not taking any medications. She is 5 feet 4 inches tall and
weighs 140 pounds. Her examination, including a pelvic examination, is normal. Which of the
following history elements would be most useful in determining the cause of amenorrhea in this
A. patient?
Age at first intercourse
B. History of sexually transmitted infections
C. Parity
D. Recent history of weight loss
E. * History of oligo-ovulatory cycles
103. A 15 year-old G0 presents with severe menstrual pain for the past 12 months. The pain is severe
enough for her to miss school. The pain is not relieved with ibuprofen 600 mg every 4 hours. She is
not sexually active and the workup reveals no pathology. The most appropriate next step in the
management of this patient is to begin combination oral contraceptives. How do oral contraceptives
relieve primary dysmenorrhea?
A. * Creating endometrial atrophy
B. Decreasing inflammation
C. Increasing prolactin levels
D. Decreasing inhibin levels
E. Thickening cervical mucous
104. A 18 year-old woman, married, has regular unprotected sex, presents with low appetite, insomnia
and amenorrhea for 3 months. What is the most likely diagnose and how to confirm it?
A. Hypothyroidism/Free T4 level
B. Hypethyroidism/TsH level
C. Cervical cancer/PAP Smear test
D. Syphylis/VDRL
E. * Pregnancy/BhcG
105. A woman who recently gave birth has elevated prolactin levels, the gland responsible for secretion
of this hormone is derived from which of the following structures?
A. * Anterior Pituitary
B. Cerebral vesicles
C. Infundibulum
D. Neurohypophysis
E. Proctodeum
106. A 55 year old woman stopped menstruating about 3 months ago, worried that she may be pregnant,
she decided to have a pregnancy test, the result is negative, which of the following series of test
results will confirm that the woman is postmenopausal?
A. Increased LH, increased FSH, increased estrogen
B. Decreased LH, decreased FSH, increased estrogen
C. Decreased LH, increased FSH, decreased estrogen
D. Increased LH, decreased FSH, decreased estrogen
E. * Increased LH, increased FSH, decreased estrogen
107. Crampy lower abdominal and back pain during menses in 21 year old woman and normal
examination would suggest?
A. Trauma
B. Cancer
C. * Primary dysmenorrhea
D. Fibrosis
E. PID
108. Choose the most exact method for determination of pathological reason for uterine bleeding in
woman 35 years old:
A. measurement of the basal temperature of the body
B. * diagnostic curettage of the mucous membrane of the uterus
C. hysteroscopy
D. measurement of the concentration of estrogens in the blood serum
E. Measurement of the concentration of progesterone in the blood serum
109. You are counseling a perimenopausal client regarding prevention of osteoporosis. You recommend
that she increase her dietary intake of which of the following?
A. Milk and iron
B. * Calcium and vitamin D
C. Magnesium and vitamin C
D. Magnesium and phosphorus
E. All of the above
110. You are completing the chief complaint interview for a 17-year-old with dysmenorrhea. You will
assess for which of the following symptoms?
A. Food cravings
B. Heart palpitations
C. Abnormal bleeding
D. * Duration of her pain
E. All of the above
111. You are explaining the intervention strategies for PMS to a 28-year-old client. You recommend that
during the latter part of her cycle she limit which of the following?
A. Exercise
B. Fluid intake
C. Fruits and vegetables
D. * Salt and caffeine intake
E. All of the above
112. A 42-year-old woman comes to the physician because of a 1-year history of vaginal bleeding for 2
to 5 days every 2 weeks. The flow varies from light to heavy with passage of clots. Menses
previously occurred at regular 25- to 29day intervals and lasted for 5 days with normal flow. She
has no history of serious illness and takes no medications. She is sexually active with one male
partner, and they use condoms inconsistently. Her mother died of colon cancer, and her maternal
grandmother died of breast cancer. She is 163 cm (5 ft 4 in) tall and weighs 77 kg (170 lb); BMI is
29 kg/m2. Her temperature is 36.6°C (97.8°F), pulse is 90/min, respirations are 12/min, and blood
pressure is 100/60 mm Hg. The uterus is normal sized. The ovaries cannot be palpated. The
remainder of the examination shows no abnormalities. Test of the stool for occult blood is negative.
Which of the following is the most appropriate next step in diagnosis?
A. * Endometrial biopsy
B. Barium enema
C. Progesterone challenge test
D. Colposcopy
E. Cystoscopy
113. A 14-year-old girl is brought to the physician by her mother because of a 2-month history of heavy
vaginal bleeding during menstrual periods. She has had episodes of excessive periodontal bleeding
while brushing her teeth and easy bruising for 6 years. She also had an episode of extended
bleeding after a tooth extraction 4 years ago. Her mother and brother have had similar symptoms.
Physical examination shows patchy ecchymosis over the upper and lower extremities. Laboratory
studies show: Platelet count 234,000/mm3 Bleeding time 17 min Prothrombin time 12 sec (INR=1)
Partial thromboplastin time 46 sec Which of the following is the most likely diagnosis?
A. * von Willebrand disease
B. Factor X (Stuart factor) deficiency
C. Factor XII (Hageman factor) deficiency
D. Hemophilia A
E. Vitamin K deficiency
114. A 36-year-old woman has been trying to conceive for the past 2 years. Her menses occur every 19
to 45 days. She has a past history of some type of sexually transmitted disease, but says she was
treated and cured. She is 163 cm (5 ft 4 in) tall and weighs 109 kg (240 lb); BMI is 41 kg/m2.
Physical examination including a pelvic examination is unremarkable. An endometrial biopsy is
performed based on the menstrual history and a negative pregnancy test. The biopsy shows stromal
breakdown associated with proliferative glands. Which of the following is the most likely cause of
her infertility?
A. * Anovulation
B. Chronic endometritis
C. Endometrial polyps
D. Endometriosis
E. All of the above
115. Patient, aged 22 years complains of delay of menses during 2 months. Taste qualities changed.
Nullipara, no abortions in the past history. Vaginal examination: mucous membrane and uterine
cervix are cyanotic, uterus of ovoid form, enlarged to 7-8 weeks of pregnancy, soft consistency.
Appendages are without peculiarities. Isthmus of the uterus is softened. Crypts of the vagina are
free. What diagnosis is the most probable?
A. * Uterine pregnancy.
B. Uterine leiomyoma.
C. Disorder of ovarian-menstrual cycle.
D. Hydatid mole.
E. Choryonepithelioma.
116. A 48-year-old patient was delivered to a hospital in-patient unit with uterine bleeding that occurred
after the 2-week-long delay of menstruation. Anamnesis states single birth. Examination of the
uterine cervix with mirrors revealed no pathologies. On bimanual examination: uterus is of normal
size, painless, mobile; uterine appendages have no changes. Discharge is bloody and copious. What
primary hemostatic measure should be taken in the given case?
A. * Fractional curettage of uterine cavity
B. Hormonal hemostasis
C. Hemostatics
D. Uterine tamponade
E. Uterotonic
117. A 13-year-old girl was admitted to the gynecology department for having a signifi-cant bleeding
from the genital tract for 10 days. The patient has a history of irregular menstrual cycle since
menarche. Menarche occurred at the age of 11. Recto-abdominal examination revealed no
pathology. What is the provisional diagnosis?
A. * Juvenile uterine bleeding
B. Adenomyosis
C. Injury of the external genitalia
D. Werlhof’s disease
E. Endometrial polyp
118. A 20-year-old female consulted a gynecologist about not having menstrual period for 7 months.
History abstracts: early childhood infections and frequent tonsillitis, menarche since 13 years,
regular monthly menstrual cycle of 28 days, painless menstruation lasts 5-6 days. 7 months ago the
patient had an emotional stress. Gynecological examination revealed no alterations in the uterus.
What is the most likely diagnosis?
A. Algomenorrhea
B. Primary amenorrhea
C. * Secondary amenorrhea
D. Spanomenorrhea

E. Cryptomenorrhea
119. A 28-year-old patient complains of infertility. The patient has been married for 4 years, has regular
sexual life and does not use contraceptives but has never got pregnant. Examination revealed
normal state of the genitals, tubal patency. Basal body temperature recorded over the course of 3
consecutive menstrual cycles appeared to have a single phase. What is the most likely cause of
A. * infertility?
Anovulatory menstrual cycle
B. Immunological infertility
C. Genital endometriosis
D. Chronic salpingoophoritis
E. Ovulatory menstrual cycle
120. 3 months after the first labor a 24- year-old patient complained of amenorrhea. Pregnancy ended in
Caesarian section because of premature detachment of normally positioned placenta which resulted
in blood loss at the rate of 2000 ml due to disturbance of blood clotting. Choose the most suitable
investigation:
A. * Estimation of gonadotropin rate
B. US of small pelvis
C. Progesteron assay
D. Computer tomography of head
E. Estimation of testosteron rate in blood serum
121. A 28-year old woman presents with secondary amenorrhea of six month duration. After a history
and physical and after pregnancy has been excluded, the next step should be:
A. * Measurement of a TSH and prolactin and administer progestational challenge
B. Measurement of gonadotropin assay and progestational challenge
C. Measurement of gonadotropin assay followed by a 21-day cycle of estrogen and progestin
D. Measurement of chromosomes and gonadotropin level
E. Measurement of gonadotropins and a coronal CT scan
122. A 22-year-old patient complains of 8-month delay of menstruation. Anamnesis: menarche since the
age of 12,5. Since the age of 18 menstruations are irregular. No pregnancies. Mammary glands
have normal development; when the nipples are pressed, milk drops are discharged. On
gynecological examination: moderate uterus hypoplasia. On hormonal examination: prolactin level
exceeds the norm two times. On computed tomogram of the sellar region: a space-occupying lesion
4 mm in diameter is detected. The most likely diagnosis is:
A. * Pituitary tumor
B. Lactation amenorrhea
C. Stein-Leventhal syndrome
D. Sheehan’s syndrome
E. Cushing’s disease
123. A 17 year-old G0 sexually active woman presents to the emergency room with acute right lower
quadrant pain and nausea for 12 hours. Her periods have always been irregular, with her last one 6
weeks ago. She is otherwise completely healthy. She appears in mild distress. Physical
examination: temperature 100.2°F (37.9°C); blood pressure 110/60; heart rate 108 beats/min. She
has moderate abdominal tenderness with right greater than left pelvic tenderness. Pelvic exam
reveals normal external genitalia and pink-tinged discharge is noted on speculum examination.
Bimanual/rectovaginal exam confirms mild cervical motion tenderness and fullness in the right
adnexa with moderate tenderness and some voluntary guarding. What is the single most important
test to obtain?
A. Pelvic ultrasound
B. CT scan of the abdomen and pelvis
C. GC and chlamydia DNA probe
D. * Beta-hCG

E. CBC with differential


124. An 18 year-old woman comes to the office due to vaginal spotting for the last two weeks. Her
menstrual periods were regular until last month, occurring every 28-32 days. Menarche was at age
13. She started oral contraceptives 3 months ago. On pelvic examination, the uterus is normal in
size, slightly tender with a mass palpable in the right adnexal region. No adnexal tenderness is
noted. Which of the following tests is the most appropriate next step in the management of this
A. patient?
Endometrial biopsy
B. Coagulation studies
C. Pelvic sonography
D. Abdominal CT scan
E. * Urine pregnancy test
125. A 26-year-old G0 woman presents to the emergency room with 8 hours of severe right lower
quadrant pain associated with nausea. She has a history of suspected endometriosis, which was
diagnosed 2 years ago, based on severe dysmenorrhea. She has been using NSAIDs with her
menses to control the pain. She is not sexually active. She is otherwise in good health. Her
menstrual cycles are normal every 28 days and her last menstrual period was 3 weeks ago. She has
no history of sexually transmitted infections. Her blood pressure is 145/70; pulse is 100;
temperature is 99.2°F. She appears uncomfortable. On abdominal exam, she has moderate
tenderness to palpation in the right lower quadrant. On pelvic exam, she has no lesions or
discharge. A complete bimanual exam was difficult to perform due to her discomfort. Labs: BHCG
A. MRI of the pelvis
B. Doppler pelvic ultrasound
C. CAT scan of the pelvis
D. Begin oral contraceptives
E. * Surgical exploration
126. A 29 years old woman with a history of pelvic inflammatory disease presents to emergency with
severe left lower quadrant crampy pain, spotting and amenorrhea for the past two cycles. There is a
left adnexal mass with tenderness to palpation. The B-hcG is elevated. Further studies would
most likely reveal an implantation at which of the following locations in the fallopian tube?
A. * Ampula
B. Fimbriae
C. Isthmus
D. Infundibulum
E. Uterine segment
127. A 35 years old sexually active woman visits her gynecologist complaining of mild, right-sided,
lower abdominal pain but no other symptoms, there are no peritoneal signs, her surgical history is
significant for an appendectomy at age 10, her last period occurred 14 days ago, which of the
following endometrial changes corresponds to this stage of the patient's menstrual cycle?
A. Apical movement of secretions in the glandular cells
B. Tissue expansion by cellular hypertrophy
C. Degeneration of the glandular structures
D. * Growth of the spinal arteries
E. Glandular glycogen accumulation in the functionalism
128. A 35 years old woman comes to the emergency with a sudden onset of severe right lower
abdominal pain, nausea, vomiting. Ultrasound shows enlarged ovary with decreased or absent
blood flow, the diagnosis might be?
A. PID
B. Ectopic pregnancy
C. Cyst rupture
D. * Ovarian torsion

E. Endometriosis
129. Name a complication of PID, presents with fever, abdominal pain and a complex multiloculated
adnexal mass with thick walls and internal debris.
A. Mole
B. * Tubo-ovarian abscess
C. Cervicitis
D. Bacterial vaginitis
E. Ovarian cancer
130. Patient, aged 20 years was admitted to gynecologic unit with complaints on acute sudden pain in
the lower abdomen in the lumbar area, moderate blood-tinged discharge from reproductive ways.
Delay of menses – 2 weeks. On bimanual examination: uterine cervix is of 3,5 cm length, soft,
cyanotic, through external os tip of the fingers passes. Uterine body is of ovoid form, large one, of
softened consistency. In the left a soft formation, movable, painful on palpation is palpable.
Discharge is blood-tinged, insignificant. Arterial pressure – 95/60 mm Hg., pulse rate – 100
beats/min. What diagnosis is the most probable?
A. Incipient abortion.
B. Threatened abortion.
C. Abortion in progress.
D. Uterine body leiomyoma.
E. * Ectopic pregnancy.
131. Patient P., aged 37 years appealed to gynecologic unit complaining of pain in the lower abdomen,
which becomes worse during 24 day period, nausea, elevation of body temperature to 38,5 oC.
Patient fell ill 15 days ago after surgery for artificial abortion. Objectively: T- 38,5 oC, pulse rate –
100 beats/min. Tongue is dry, abdomen is tense in the lower part, signs of peritoneal irritation are
positive. On bimanual examination: sharp pain develops in replacement of uterine body, contours
of the uterus and appendage are not defined clearly because of tension of the anterior abdominal
wall. Right Pelvic mass is presented Vaginal discharge is pus-like. What diagnosis is the most
A. * probable?
Pelvic peritonitis.
B. Acute salpingoophoritis.
C. Acute appendicitis.
D. Parametritis.
E. Gonorrhea pelvic peritonitis.
132. Patient was admitted to gynecologic unit complaining of menstruation delay during 2 weeks,
staining blood-tinged discharge from reproductive passages, pain in the lower abdomen more
expressed in the left side, nausea, vomiting, weakness. In the past history: chronic adnexitis. On
bimanual examination: uterus body is slightly enlarged in sizes, softened, appendages in the left are
enlarged, painful on palpation, uterine cervix is conical in shape, external os is closed. Posterior
crypt hangs over, very painful. Reaction to chorionic gonadotropin is positive, By ultrasonic
examination: in the uterine cavity a fetal ovum is not determined. What diagnosis is the most
A. probable? amenorrhoe.
Secondary
B. Exacerbation of chronic salpingoophoritis.
C. Shtein-Leventhal’s syndrome.
D. * Ectopic pregnancy.
E. Endometriosis.

133. Patient, aged 20 years was admitted to the hospital with sharp pains in the lower abdomen in the
left. The last normal menstruation was 2 weeks ago, in time. Arterial pressure – 100/70 mm Hg.,
pulse rate – 90 beats/min., body temperature is 36,9*C, patient is pale. Abdomen is painful in the
lower portions. Blumberg-Schyotkin’s symptom is slightly positive. On vaginal examination:
uterine cervix is of conical shape, external os is closed. Uterine is not enlarged, sensitive.
Appendages on the right are not palpable. On the left something painful appendages are palpable.
Crypts hang over, painful ones. What diagnosis is the most probable?
A. Uterine leiomyoma.
B. * Apoplexy of the ovary.
C. Ectopic pregnancy.
D. Inflammation of the uterine appendages.
E. Acute appendicitis.
134. Patient, aged 28 years was admitted to the in-patient unit complaining of sharp pains in the lower
abdomen which developed 2 hours ago and of blood-tinged staining discharge from reproductive
ways. Last menstruation began 2 months ago. On bimanual examination: uterine body is somewhat
enlarged. Displacement along the uterine cervix is sharply painful. Uterine appendages in the left
are enlarged, painful. In the blood: Hb – 102 g/l, ESR – 32, leucocytes – 6,5x109/l, diastase –
8g/hour l. Signs of peritoneal irritation are positive. What is the most probable diagnosis?
A. Uterine leiomyoma.
B. * Disturbed uterine pregnancy.
C. Acute appendicitis.
D. Progressive ectopic pregnancy.
E. Abortion in progress.
135. Patient B., 22 years of age complains of aching pains in the right iliac area, lasting during a week,
nausea in the morning, gustatory changes. In the past history: menstruation delay – 3 weeks.
Objectively: arterial pressure: 110/70 mm Hg., pulse rate – 78 beats/min., body temperature –
37*C. On bimanual examination: uterine is slightly enlarged, softened, movable, painless.
Appendages: in the left side a painful formation of 3x4 cm, dense-elastic consistency, limitedly
movable is palpable. What diagnosis should be made?
A. Cyst of the right ovary.
B. Disturbed uterine pregnancy.
C. * Progressive uterine pregnancy.
D. Uterine pregnancy.
E. Acute appendicitis.
136. Ambulance delivered a patient complaining of short-term loss of consciousness, dizziness, sharp
pain in the right lower abdomen, last menstruation was 2 weeks ago. Skin integuments are pale.
Pulse rate – 110 beats/min., pulse is thread, Hb - 76 g/l, body temperature – 36,8*C, blood pressure
– 80/60 mm Hg., abdomen falls behind on respiration in the lower portions, palpation of the
abdomen on the bottom is painful. On examination: uterine cervix is within the norm, cervical
canal is closed. Uterine body is not enlarged, on palpation insignificant tenderness is noted.
Appendages in the right side on palpation are painful, ovary is enlarged. Posterior fornix of the
vagina hangs over. What is the most probable diagnosis?
A. * Ruptured ectopic pregnancy.
B. Acute appendicitis.
C. Apoplexy of the ovary.
D. Acute salpingoophoritis.
E. Torsion of the cystoma of the right uterine appendages.

137. Woman, aged 28 years appealed for doctor’s advice complaining of acute pain in the left iliac area,
no menstruation delay. By the data of ultrasonic examination 2 months ago ovarian cyst in the left
was revealed. Patient refused from hospitalization. On vaginal examination: in the left side
tumor-like formation, sizes of 5x7 cm., painful one, very movable is determined. What is the most
probable diagnosis?
A. Ectopic pregnancy.
B. Apoplexy of the ovaries.
C. Threatening abortion.
D. * Torsion of the ovarian cyst.
E. Hydatid mole.
138. Patient K. aged 19 years, 3 hours ago after physical culture lesson developed pain in the lower
abdomen in the left, dryness in the mouth. Menstruation was 2 weeks ago. Pulse rate – 92
beats/min., rhythmic one. Arterial pressure – 95/55mm Hg. Tongue is dry and coated. Abdomen
“breathes”, but painful in the right iliac area and suprapubic junction, doubtful symptoms of
peritoneal irritation are noted. What diagnosis should be made?
A. * Apoplexy of the ovary.
B. Acute appendicitis.
C. Acute salpingoophoritis.
D. Disturbed uterine pregnancy.
E. Renal colic in the left.
139. Patient S., aged 41years appealed for doctor’s advice complaining of periodic pains in the lower
abdomen, painful profuse, prolonged menstruation. On bimanual examination: uterine was enlarged
to 9 weeks of pregnancy, along the posterior wall a node, sizes 3x3 cm is palpable, uterine body is
movable, painful, appendages are without peculiarities. What disease should be thought of?
A. Ovarian tumor.
B. Adenomyosis.
C. Uterine pregnancy.
D. Chronic endometritis.
E. * Ischemia of fibromatous node.
140. Woman, aged 30 years was admitted to gynecologic unit for planned surgical intervention because
of tumor-like formation in the area of the right appendages, sizes 9x8 cm., of dense-elastic
consistency, movable, painless. While performing physical work, severe pain developed, woman
lost consciousness. During surgical intervention a thin-walled formation on the long pedicle,
containing hair and teeth was removed. What diagnosis should be made?
A. * Torsion of the dermoid ovarian cyst.
B. Uterine leiomyoma.
C. Ovarian cancer.
D. Hydrosalpinx.
E. Pyo-ovarium.
141. Patient, aged 28 years complains on pain in the lower abdomen, loss of consciousness developed at
home. Menstruation is in time. Skin is pale, pulse rate – 110 beats/min., Hb. – 76 g/l., arterial
pressure is 80/60. Signs of peritoneal irritation are positive. On vaginal examination: uterine body
is not enlarged, appendages are not palpable. Posterior fornix of the vagina hangs over, painful.
Blood is revealed at culdocenfesis. What is the most probable diagnosis?
A. Disturbed ectopic pregnancy.
B. * Ovarian apoplexy.
C. Torsion of the cystoma of the right uterine appendages.
D. Acute salpingoophoritis.
E. Acute appendicitis.

142. Patient, aged 23 years, married. Menstruation has been since 16 years, regular one. Last
menstruation was 7 weeks ago. Regular sexual life. Patient does not use means of contraception. In
the past history: chronic adnexitis,, patient is followed up at the prenatal center, received treatment
several times due to exacerbation of the inflammatory process of the small pelvis organs. By the
data of ultrasonic examination, a formation, sizes - 7x3x4 cm in the area of the right appendages
was revealed. Diagnosis of hydrosalpinx was made. On her way to work patient experienced sharp
pain in the lower abdomen, lost consciousness. She was delivered to gynecologic unit by an
ambulance in a severe state. Skin and mucous membranes are pale, arterial pressure – 75/40 mm
Hg. Pulse rate is 116 beats/min., body temperature – 360C, symptom of irritation of the peritoneum
is positive. What is the most probable diagnosis?
A. * Ruptured ectopic pregnancy.
B. Abortion in progress.
C. Rupture of the ovarian cyst.
D. Pelvioperitonitis.
E. Perforating gastric ulcer.
143. Patient with uterine fibromyoma (first revealed 4 years ago). During observation uterine sizes are
stable (correspond to 8-9 weeks of pregnancy. Patient appealed for doctor’s advice, complaining of
sharp pains in the lower abdomen. On examination: sharply positive symptoms of irritation of the
peritoneum, high leucocytosis. On vaginal examination: uterine is enlarged to 9 weeks of
pregnancy term at the expense of fibromatous nodes, one of them is movable, sharply painful.
Appendages are not palpable. Discharge is mucous, moderate one. What is treatment tactics?
A. Urgent surgical intervention (laparotomy).
B. Observation and spasmolytic therapy.
C. Fractional diagnostic curettage of uterine walls.
D. * Surgical laparoscopy.
E. Observation and therapy with antibiotics.
144. Patient, aged 28 years was admitted to the gynecologic unit complaining of sharp pain in the lower
portions of the abdomen, pain developed suddenly on the 4-th week of menstruation delay. Skin
integuments are pale, pulse rate – 110 beats/min., arterial pressure – 90/60. Abdomen is sharply
painful on palpation in the lower portions, positive symptom of irritation of the peritoneum. On
vaginal examination: uterine is enlarged, deviated to the right, on the left – oblong formation,
painful on palpation is revealed, posterior fornix is sharply painful, hangs over. What is the most
informative method of investigation?
A. Diagnostic laparoscopy.
B. * Paracentesis of the posterior fornix.
C. Test for chorionic gonadotropin.
D. Ultrasonic investigation.
E. Dynamics of total blood analysis.
145. Patient, aged 43 years complains of constant dull pains in the lower abdomen, mostly in the left
side, elevation of body temperature up to 38 0 C. During the last 5 years patient is followed up due
to subserous uterine fibromyoma. On bimanual examination: uterine is enlarged to 10 weeks of
gestation term, solid, tuberous one. In the left side of the uterine, formation coming out of the
uterine, size 6x8 cm., of elastic consistency, sharply painful on palpation is observed. What is the
most probable diagnosis?
A. Pyosalpinx.
B. Extrauterine pregnancy.
C. * Necrosis of myomatous node.
D. Cyst of the left ovary, which became suppurated.
E. Uterine sarcoma.

146. Patient, aged 37 years was admitted to the gynecologic unit complaining of intensive cramp-like
pains in the lower abdomen, bleeding from reproductive passages. Over the period of the last 4
years patient experienced profuse menstruation, sometimes transitory into bleeding. Patient was not
followed up at gynecologist’. On bimanual examination: uterine cervix is smoothened, external os
is opened up to 4 cm., in the cervical canal a dense tumor-like formation is palpable. Uterine is
enlarged, dense. What is the most probable diagnosis?
A. Necrosis of fibromatous node.
B. Abortion in progress.
C. Hydatid mole.
D. * Protruding myoma.
E. Uterine cancer.
147. A patient with acute pelvic pain has a low grade fever, nontender uterus and negative beta hCG test,
and negative urinalysis. The most probable diagnosis would be:
A. * Salpingitis
B. Appendicitis
C. Missed abortion
D. Ectopic pregnancy
E. Rupted ovarian cyst
148. At the vaginal examination of the patient you revealed the following: the exernal os of cervix is
closed, the uterus is slightly enlarged, soft, the right of the appendages is soft and painful. There is
pain with movement of the cervix. Possible diagnosis:
A. * progressive tubal pregnancy.
B. apoplexy of the right ovary.
C. chronic inflammation of the right adnexa.
D. Appendicitis.
E. All answers are correct.
149. Patient 29 years, delivered by the emergency, complains about acute pains in lower parts of an
abdomen. Pains arose up suddenly, at getting up of weight. The last menstruation was 10 days ago,
in the term. Labors — 2, abortions — 2. The last time visited gynecologist half-year ago, ovarian
cyst was definite. Pulse - 100 in a minute, rhythmic, breathing 22 in a minute. Abdomen is tense,
acutely painful, especially on the left. Objectively: the uterine cervix is cylinder, deformed by old
postnatal ruptures, clean. The uterine body is not determined due to tension of abdominal wall.
Right adnexa not palpated. A tumor without clear contours is palpated in the region of the left
adnexa, elastic consistency; the mobile is limited, painful. Parametriums are free. What most
A. reliable diagnosis?
The ruptured ectopic pregnancy.
B. Torsion of pedicle of ovarian cyst.
C. Apoplexy of ovary.
D. * Rupture the cysts of ovary.
E. Appendicitis
150. The patient 36 years complains on pain in lower parts of abdomen on the left side, which arose up
suddenly. Objectively: external genital organs without pathology, the uterine cervix is cylindric,
clean. The body of uterus is enlarged to 12—13 weeks of pregnancy, the mobile is limited. One of
fibroids on the left near a fundus acutely painful. Adnexa are not determined, its region is
unpainful. Parametrium is free. Excretions serous. Blood test: Haemoglobin — 120 g/l, leucocytes
— 12x109 /л. What is the most reliable diagnosis?
A. Chorionepithelioma.
B. Spontaneous rupture of pregnant uterus.
C. * Necrosis of fibroid.
D. The interrupted pregnancy in the interstitsial region of fallopian tube.
E. Destructive form of the molar pregnancy.

151. The patient 36 years old complains of pain in lower parts of abdomen, that reminds the labor
contractions, weakness. The menstruations last 2 years are more abundant, of long duration. The
last menstruation began 2 days ago. Objectively: A skin and mucous membranes are pale, pulse 88
in 1 min. Abdomen is soft, unpainful. Gynecological status: external genital organs without
pathology, the uterine cervix is cylinder, a canal freely skips 2 fingers. From a cervix a tumor 3x6
cm hangs down to the vagina, pedicle by thickness to 1 cm enters to the cavity of uterus. The tumor
is a dark-purple color, at contact bleeds. The body of uterus is enlarged to 7-8 weeks of pregnancy,
unequal surface, mobile, not painful. Adnexa are not determined. What is the most reliable
A. * diagnosis?
The protruding fibroid
B. Endophytic growth of cancer of uterine cervix
C. Chorionepithelioma, metastasis in the uterine cervix
D. Inevitable abortion in 7-8 weeks of pregnancy
E. Exophytic growth of cancer of uterine cervix
152. In the gynecological department a woman 25 years appealed with complaints about the rise of
temperature of body to 38,60С, pain in lower parts of abdomen, dysuria. Became ill 3 days to that,
when the indicated complaints appeared after artificial abortion. At gynecological examination: the
uterine cervix is cylinder, external os is closed. Body of uterus a few enlarged, painful, soft. The
adnexa of uterus are not palpated. Excretions festering-bloody. In the blood test leycocytosis with
displacement of formula of blood to the left, speed-up ESR. What diagnosis is most credible?
A. Acute endocervicitis
B. Acute salpingoophoritis
C. * Acute endometritis
D. Acute cystitis
E. Piosalpinx
153. A 40-year-old woman comes to the physician for an annual examination. She has no complaints.
She has menses every 28-30 days that last for 3 days. She has no intermenstrual bleeding. She has
asthma, for which she uses an occasional inhaler. She had a tubal ligation 10 years ago. She has no
known drug allergies. Examination is unremarkable, including a normal pelvic examination. One of
her friends was recently diagnosed with endometrial cancer, and the patient wants to know when
and if she needs to be screened for this. Which of the following is the most appropriate response?
A. * Screening for endometrial cancer is not cost effective or warranted
B. Screening is with endometrial biopsy and starts at age 40
C. Screening is with endometrial biopsy and starts at age 50
D. Screening is with ultrasound and starts at age 40
E. Screening is with ultrasound and starts at age 50
154. A 21-year-old G0 woman has her first Pap smear, and it shows a high-grade squamous
intraepithelial lesion (HSIL.) Colposcopy is performed, and three biopsies and an endocervical
curettage are obtained. The biopsy and endocervical curettage were read as normal. Which of the
following would be the most appropriate next step in the management of this patient?
A. Pap smear in 6 months
B. Colposcopy with directed biopsies in 3-6 months
C. Cryotherapy
D. * Cervical conization
E. Treat for presumed infection and repeat Pap in 4-6 weeks
155. An 88 year-old G2P2 nursing home resident is brought in for evaluation of blood found in her
diapers. She has a long-standing history of incontinence. This is the first time that her caregivers
have noted blood. They describe it as “quarter size.” Her nurses think that she may have been
itching, as they frequently find her scratching through the diaper. On review of her medical record,
biopsy documented lichen sclerosus of the vulva was diagnosed fifteen years ago. She has not been
on any therapy for this condition for years. Examination of the external genitalia reveals an
elevated, firm irregular lesion arising from the left labia. The lesion measures 2.5 cm in greatest
dimension. The remainder of the external genitalia shows evidence of excoriation of thin, white
skin with a wrinkled parchment appearance. The vagina and cervix are atrophic. No masses are
noted on bimanual or rectovaginal exam and a sample of her stool is negative for blood. No
nodularity is noted in her groin. Which of the following is the most appropriate next step in the
management of this patient?
A. Begin steroids for her lichen sclerosis
B. Benadryl and xylocaine jelly for symptomatic relief
C. * Biopsy the lesion
D. Exam under anesthesia with colposcopy and CO2 laser ablation
E. Complete vulvectomy and lymph node dissection
156. A 57 year-old G2P2 woman is seen for a routine visit. She states she and her 75 year-old husband
stopped having sexual intercourse 3 years ago when he had an operation for prostate cancer.
Menopause occurred at age 50 and she denies taking hormones. Her husband now wishes to resume
intercourse and is able to get an erection with sildenafil (Viagra). Attempts at intercourse have been
unsuccessful due to the pain she experiences when insertion is attempted. Examination is normal
except for a narrowed vagina with atrophic mucosa. Which of the following is the most appropriate
recommendation at this time?
A. Progesterone cream
B. * Estrogen cream
C. Oral estrogen
D. testosterone cream
E. Vaginal dilators
157. A 39 year-old G1P1 woman comes to see you because of increased bleeding due to her known
uterine fibroids, especially during her menses. She reports that her bleeding is so heavy that she has
to miss two days of work every month. She has been using oral contraceptives and NSAIDs. Her
most recent hematocrit was 27%. She is undecided about having more children. You discuss with
her short and long-term options to decrease her bleeding. What is the next best step in the
management of this patient?
A. Blood transfusion
B. * Gonadotropin-releasing hormone agonists
C. Endometrial ablation
D. Hysterectomy
E. Uterine artery embolization
158. A 28-year-old G0 woman has her first abnormal Pap, which was read as high-grade squamous
intraepithelial lesion (HSIL.) She has no complaints. She smokes one pack of cigarettes per day.
Her pelvic exam is normal. Colposcopy is performed. The cervix is noted to have an ectropion and
there is abundant aceto-white epithelium. Mosaicism, punctations and several disorderly, atypical
vessels are noted. Several biopsies are obtained and sent to pathology. Which of the findings on this
patient’s colposcopy is most concerning?
A. Ectropion
B. Acetol-white epithelium
C. Mosaicism
D. Punctations
E. * Disorderly, atypical vessels

159. The patient K, 48 years old, came for regular check-up. Which process the type I does reflects
in Pap smear test?
A. * Normal epithelium.
B. Moderate dysplasia.
C. Cancer.
D. Inflammation.
E. Malignization suspect.
160. The patient C, 38 years old, complains on excessive with odor discharge from the vagina lately.
Which process does the type IIA reflect in Pap smear test?
A. Normal epithelium.
B. Moderate dysplasia.
C. Cancer.
D. * Inflammation.
E. Malignization suspect.
161. A 76-year-old G3P3 presents to the office with worsening stress urinary incontinence for the last 3
months. She reports an increase in urinary frequency, urgency and nocturia. On exam, she has a
moderate size cystocele and rectocele. A urine culture is negative. A post-void residual is 50 cc. A
cystometrogram shows two bladder contractions while filling. Which of the following is the most
likely diagnosis in this patient?
A. Genuine stress incontinence
B. * Urge incontinence
C. Overflow incontinence
D. Functional incontinence
E. Continuous incontinence
162. An 81 year-old G3P3 woman presents to your office with a history of light vaginal spotting. She
states this has occurred recently and in association with a thin yellow discharge. She never
experienced any vaginal bleeding since menopause at the age of 52, and denies ever having been on
hormone replacement therapy. She is otherwise reasonably healthy, except for osteoporosis,
well-controlled hypertension, and diabetes. She is physically active and still drives to all her
appointments. She is no longer sexually active since the death of her husband 2 years ago. On
examination, she is noted to have severe atrophic changes affecting her vulva and vagina. A small
Pederson speculum allows for visualization of a normal multiparous cervix, and the bimanual
examination is notable for a small, mobile uterus. Rectovaginal exam confirms no suspicious
adnexal masses or nodularity. Which of the following is the most appropriate management for this
patient?
A. Pelvic transvaginal ultrasound
B. * Office endometrial biopsy
C. Reassurance and observation for further bleeding
D. Vaginal estrogen therapy
E. Dilation and curettage
163. A 35 year-old G3P3 comes to the office to discuss tubal ligation as she desires permanent
sterilization. What are the non-contraceptive health benefits of female sterilization?
A. Reduced risk of endometriosis
B. * Reduced risk of ovarian cancer
C. Protection against endometrial cancer
D. Reduction in menstrual blood flow
E. Reduced risk of sexually transmitted infections

164. A 38 year-old G1P1 woman comes to the office for an annual exam. She has noticed some urinary
frequency over the past month. She has no dysuria, hematuria, urgency or incontinence. She has
normal cycles, no history of abnormal Pap smears or sexually transmitted infections and is sexually
active, with 1 partner. She smokes a quarter of a pack of cigarettes daily, and drinks one glass of
wine per day. Her mother had breast cancer at age 30. Her general examination is normal. On
pelvic exam, she has normal external genitalia; vagina and cervix are without lesions. Her uterus is
normal size, anteverted and nontender. Her left adnexa is normal, right adnexa has a mobile,
slightly tender 4 cm mass. Laboratory results show a normal urinalysis, a negative urine pregnancy
test and a normal Pap smear. What is the most appropriate next step in the management of this
A. * patient?
Perform a transvaginal ultrasound
B. Perform a diagnostic laparoscopy
C. Recommend a CT-guided drainage of the mass
D. Order a KUB plain film
E. Perform an exploratory laparotomy
165. A 52 year-old nulliparous woman presents with long-standing vulvar and vaginal pain and burning.
She has been unable to tolerate intercourse with her husband because of introital pain. She had
difficulty sitting for prolonged periods of time or wearing restrictive clothing because of worsening
vulvar pain. She recently noticed that her gums bleed more frequently. She avoids any topical
over-the-counter therapies because they intensify her pain. Her physical examination is remarkable
for inflamed gingiva and a whitish reticular skin change on her buccal mucosa. A fine papular rash
is present around her wrists bilaterally. Pelvic examination reveals white plaques with intervening
red erosions on the labia minora as shown in picture below. A speculum cannot be inserted into her
vagina because of extensive adhesions. The cervix cannot be visualized. Which of the following is
the most likely diagnosis in this patient?
A. Squamous cell hyperplasia
B. Lichen sclerosus
C. * Lichen planus
D. Genital psoriasis
E. Vulvar cancer
166. Heavy menses with clots, constipation, urinary frequency, pelvic pain, heaviness, enlarged uterus
are suggestive for
A. * Fibrosis
B. Malignancy
C. PID
D. Vaginitis
E. All of them
167. You are collecting data from a 37-year-old client who you suspect may have fibroid tumors. You
expect her subjective data to include which of the following symptoms?
A. Urinary urgency
B. Difficult defecation
C. Cyclic migraine headaches
D. * Deep pelvic paindyspareunia
E. All of the above
168. History of obesity, nulliparity, chronic anovulation, irregular intermenstrual bleeding, non tender
uterus in 47-year-old woman gives indication for?
A. * Endometrial cancer/ hyperplasia
B. Hormonal changes
C. Sexual transmitted disease
D. Pelvic Inflammatory disease
E. Fibroids

169. A 52-year-old woman comes to the physician because of a 1-month history of headache, weakness,
tingling of her extremities, muscle cramping, and fatigue. Her blood pressure is 170/110 mm Hg.
Physical examination shows no other abnormalities. Laboratory studies show a decreased serum
potassium concentration, metabolic alkalosis, and decreased plasma renin activity; serum sodium
concentration is within the reference range. Urine catecholamine concentrations are within the
reference range. Which of the following is the most likely diagnosis?
A. * Adrenal adenoma
B. Focal segmental glomerulosclerosis
C. Hypothalamic tumor
D. Juxtaglomerular cell tumor
E. Renal artery stenosis
170. A twenty year old is found to have moderate dysplasia (high grade SIL) on a routine PAP. Moderate
inflammation is noted on the PAP. The next appropriate step in management is?
A. repeat PAP in 3 months
B. treat inflammation and repeat PAP in 3 months
C. * colposcopy and biopsies
D. wire loop excision of the transformation zone
E. cold knife conization
171. Which treatment is followed by a 40 years patient with cervical dysplasia with the deformation of
the cervical canal:
A. * diatermokonization of cervix
B. diathermocoagulation of cervical
C. laser
D. treatment by solkovagin
E. hysterectomy without appendages
172. A 29-year-old woman comes to the physician for follow-up of a right breast lump. The patient first
noticed the lump 4 months ago. It was aspirated at that time, and cytology was negative, but the
cyst recurred about 1 month later. The cyst was re-aspirated 2 months ago and, again, the cytology
was negative. The lump has recurred. Examination reveals a mass at 10 o'clock, approximately 4
cm from the areola. Ultrasound demonstrates a cystic lesion. Which of the following is the most
appropriate next step in management?
A. Mammography in 1 year
B. Ultrasound in 1 year
C. Tamoxifen therapy
D. * Open biopsy
E. Mastectomy
173. A 43-year-old African American woman comes to the physician because of her concern regarding
breast cancer. She has no complaints at present. In past years, she had noted bilateral breast
tenderness prior to her menses, but this has since abated. She has no medical problems. She had
two cesarean deliveries, but no other surgeries. She takes a low-dose oral contraceptive pill and has
no known drug allergies. She does not smoke, and her family history is negative. Physical
examination is normal. All mammograms (yearly since age 40) have been negative to date. She
wants to know whether BRCA1 and BRCA2 screening would be appropriate for her. Which of the
following is the correct response?
A. * BRCA1 and 2 screening is not recommended
B. BRCA1 and 2 screening should be performed after age 50
C. * BRCA1 and 2 screening should be performed if breast pain recurs
D. BRCA1 screening is recommended
E. BRCA2 screening is recommended

174. A 50 year-old G4P4 woman presents for her yearly checkup. She states her menses are of normal
flow every 32 days, with minimal cramping and 5-day duration. Review of systems is negative. She
has no medical problems. She had a bilateral tubal ligation following her last child, and a
laparoscopic cholecystectomy 5 years ago. She has a history of LGSIL Pap smear with colposcopy
and cryotherapy 15 years ago; her Paps have been normal since then. She does not smoke, drink
alcohol or use any drugs. She is sexually active with one partner with no problems. Her general
exam, including a breast exam and pelvic exam, is normal. In addition to performing a Pap smear,
which of the following is the most appropriate screening test for this patient?
A. Pelvic ultrasound
B. Endometrial biopsy 1% C. 94% D. 2% E.
C. * Mammogram
D. DEXA scan
E. Colposcopy
175. A 25 year-old G1P1 comes to the office due to left breast pain and fever. She is breast feeding her 2
1/2- week old infant. The symptoms began earlier in the day and are not relieved by
acetaminophen. Blood pressure 120/60; pulse 64; temperature 37.7° C. On exam, she has erythema
on the upper outer quadrant of the left breast, which is tender to touch; there are no palpable
masses. In addition to starting oral antibiotics, what is the most appropriate next step in the
A. management of this patient?
Discontinue breastfeeding
B. * Add ibuprofen for pain relief
C. Obtain a breast ultrasound
D. Use a topical antifungal
E. Admission to the hospital
176. A 23 year-old G1P1 delivered her first baby two days ago after an uncomplicated labor and vaginal
delivery. She wants to breast feed and has been working with the lactation team. Prior to discharge,
her temperature was 100.4°F, 38°C (other vitals were normal). She denies urinary frequency or
dysuria and her lochia is mild without odor. On examination, her lungs are clear, cardiac exam
normal, and abdomen and uterine fundus are nontender. Her breasts are firm and tender throughout,
without erythema and nipples are intact. Which of the following is the most likely cause of her
A. fever?
Endomyometritis
B. Septic pelvic thrombophlebitis
C. Mastitis
D. * Breast engorgement
E. Vaginitis
177. A 61 years old woman comes to the office because of an enlarging breast mass. Exam reveals a
firm, fixed, 3 cm mass in the right upper outer quadrant, the biopsy of mass reveals loss of cell
differentiation and lack of tissue organization, which if the following term best corresponds to this
finding?
A. * Anaplasia
B. Desmoplasia
C. Dysplasia
D. Hyperplasia
E. Metaplasia
178. A 67 years old woman with advanced metastatic breast cancer comes to the clinic for follow-up
visit, her chief complaint at this time is weight loss and a reduced appetite. Which of the following
is thought to be a major contributor to her chief complaint?
A. Clathrin
B. Histamin
C. Interferon
D. * Tumor necrosis factor (TNF)

E. Interleukin
179. 45 years old woman presents with blood tinged discharge from the right nipple. The nipple appears
slightly retracted and subareolar nodule can be felt on palpation. Mammographic exam shows
linear shadows attributable to calcification. The lesion consists of dilated ducts and are surrounded
by fibrosis and chronic inflammation, focal calcium deposition is present, which of the following is
the diagnosis?
A. Mondor disease
B. Adenocarcinoma
C. * Mammary duct Eurasia
D. Fat necrosis
E. Abscess
180. A 61 years old woman comes to the office because of an enlarging breast mass. Exam reveals a
firm, fixed, 3 cm mass in the right upper outer quadrant, the biopsy of mass reveals loss of cell
differentiation and lack of tissue organization, which if the following term best corresponds to this
finding?
A. Metaplasia
B. Hyperplasia
C. Dysplasia
D. * Anaplasia
E. Desmoplasia
181. A 67 years old woman with advanced metastatic breast cancer comes to the clinic for follow-up
visit, her chief complaint at this time is weight loss and a reduced appetite. Which of the following
is thought to be a major contributor to her chief complaint?
A. Clathrin
B. Histamin
C. Interferon
D. Interleukin-2
E. * Tumor necrosis factor (TNF)
182. A 30 years old woman complains of breast tenderness that becomes worse during the premenstrual
period, multiple small masses are appreciable on palpation, cytologic exam of a fine needle aspirate
reveals no malignant cells, biopsy shows multifocal cyst formation, areas of fibrosis, calcification
and apocrine metaplasia, this condition is associated with an increased risk of cancer if:
A. Apocrine Metaplasia is marked
B. Calcification is prominent
C. Cysts are larger than 0.5 cm
D. * Epithelial hyperplasia is florid
E. Fibrosis is predominant
183. An 18 years old girl whose grandmother was recently diagnosed with breast cancer discovers a
large, round, movable nodule in her left breast, she visits the physician with concern about the
nodule. Which of the diagnosis should rank highest on the physician’s differential?
A. Cystosarcoma phyllodes
B. * Fibroadenoma
C. Infiltrating ductal carcinoma
D. Intraductal papilloma
E. Fibrocystic breast disease

184. You are counseling a 40-year-old client who has come to the clinic because she fund a “lump” in
her breast last night. She is frantic because she believes she has cancer. The clinical breast
examination reveals firm, smooth, discrete masses in both breast. You reinforce the physicisn’s
impression that she is feeling is most likely a noncancerous “lump” and that she should have
which of the following evaluation procedures first?
A. Lumpectomy
B. * Mammogram
C. Excisional biopsy
D. Stereotactic biopsy
E. All of the above
185. A 19-year-old woman noticed a mass in her left breast 2 weeks ago while doing monthly breast
self-examination. Her mother died of metastatic breast cancer at the age of 40 years. Examination
shows large dense breasts; a 2-cm, firm, mobile mass is palpated in the upper outer quadrant of the
left breast. There are no changes in the skin or nipple, and there is no palpable axillary adenopathy.
Which of the following is the most likely diagnosis?
A. Lobular carcinoma
B. Intraductal papilloma
C. Infiltrating ductal carcinoma
D. Fibrocystic changes of the breast
E. * Fibroadenoma
186. A 27 year-old G0 woman comes to the clinic as she has been unable to conceive for the last year.
She is in good health and has not used any hormonal contraception in the past. She had normal
cycles in the past every 28 days until about 6 months ago. At that time, she began to have irregular
menses every 2-3 months, with some spotting in between. She is not taking any medications. She
has no history of abnormal Pap smears or sexually transmitted infections. Her physical examination
is normal. Laboratory tests show: Results Normal Values TSH 10 mIU/ml 0.5-4.0 mIU/ml Free
T40.2 ng/dl 0.8-1.8 ng/dl Prolactin 40 ng/ml
A. * Begin combined oral contraceptive
B. Begin bromocriptine
C. Order a Clomid ovulation challenge test
D. Obtain a brain MRI
E. Order a thyroid gland ultrasound
187. 25 year old woman comes to the physician for the regular check-up, during meeting, she asks the
physician about the direct role of hcG in pregnancy, which of the following is the best reply?
A. Egg producing
B. * Maintenance of the corpus luteum
C. Uterine contraction
D. Ovaries relaxant
E. Maintenance of placenta
188. A 35-year-old woman is considered infertile after ____ of trying to conceive
A. 1 month
B. 2 months
C. 4 months
D. * 6 months
E. 12 months

189. Patient 22 years, complains about absence of pregnancy during 5 years of marriage. For the last 3
years conducted medical and resort treatment. Two months ago metrosalpingography was done —
tubes’ permeability is normal. The last menstruation was with the delay on 2 weeks, painful. 2 days
ago bloody excretions in a small amount and pain in lower parts of abdomen appeared again.
Objectively: the uterine cervix is cyanotic, external os is closed. The uterus in normal position,
enlarged to 5 weeks of pregnancy, is soft, mobile. Douglas punch isn’t painfull. Right adnexa are
not determined. The left adnexa are enlarged in sizes to 6x3 cm, painful at palpation. Parametriums
are free. Excretions are bloody, in a little quantity. Blood test: Haemoglobin — 90g/l, red cells —
3,0 h 10 /l, leucocytes— 8,6 h 10 /l. What is the most reliable diagnosis?
A. * Pregnancy 6-5 weeks. Abortion, that began. Cyst of the left ovary.
B. Pyosalpinx after metrosalpingography
C. The molar pregnancy
D. Violation of menstrual cycle
E. Ectopic pregnancy, that was ruptured on the type of tube abortion
190. The patient 25 years old, diagnosed with endocrine form of infertility. Which drug stimulates
ovulation of patients with endocrine form of infertility?
A. progesterone
B. * Clomiphene Citrate
C. androkur
D. femoston
E. dufaston
191. The patient 29 years old, complains of infertility. Sexual life has been leading in marriage for 4
years, hasn’t been preventing pregnancy. She didn’t have any pregnancy yet. An examination of
women is established: the development of genital organs were normal; The fallopian tubes are
passable. Basal [rectal] temperature for three menstrual cycles had single phase. The most probable
cause of infertility?
A. * Anovulatory menstrual cycle
B. Chronic adnexitis
C. Anomalies of genital organs
D. Immunological infertility
E. Genital endometrioses
192. The patient complains about infertility. Menstruations started at 16 years, are not regular, 5-6 days
after 28-35 days, to marriage - painful. Sexual life during 4 years. Did not use contraceptives,
become not pregnant. In childhood was ill on a measles, scarlet fever. Vaginally: the uterus is of
normal size, mobile, unpainful, adnexa are not palpated. In speculum: the uterine cervix is normal.
Analysis of sperm of husband - 55 mln of spermatozoa in 1 ml, 75% of them are mobile. Shuvarscy
test is positive. A basal temperature during 2 cycles is monotonous. Diagnosis?
A. Infertility I, tubal genesis
B. Infertility I, tubal-peritoneal genesis
C. * Infertility I, anovulatory cycles
D. Infertility I, masculine genesis
E. Infertility I, anomaly of development of privy parts
193. The woman, suffering from infertility came to the gynecologist with complain of delayed
menstruation. Which pregnancy test will be the most reliable in the early stages?
A. * Measurement of human chorionic gonadotropin in the blood
B. Immune hemagglutination inhibition test.
C. Reaction Galey Maynini
D. Measurement of the concentration of estrogen in the blood
E. Measurement of concentration of progesterone in the blood

194. While discussing reproductive health with a group of female adolescents, one of the adolescents
asks the doctor, "Where is the ovum fertilized?" The doctor responds by stating that fertilization
normally occurs at which of the following sites?
A. Uterus.
B. Vagina.
C. * Fallopian tube.
D. Cervix.
E. Ovary
195. A female patient, aged 25, suffers endocrine form of infertility for 5 years. What should be included
in investigations of this patient?
A. Ultrasonic monitoring of growth of follicles during the menstrual cycle.
B. Measuring basal temperature.
C. Determine the level of hormones in the blood.
D. Smears on the "hormonal mirror."
E. * All answers are correct.
196. A couple visiting the infertility clinic for the first time asks the doctor, "What causes infertility in a
woman?" Which of the following would the doctor include in the response as one of the most
common factors?
A. Absence of uterus.
B. Overproduction of prolactin.
C. * Anovulation.
D. Immunologic factors.
E. Genetic factors.
197. 25-year-old patient notes the absence of pregnancy within 5 years. Operations were performed
twice at the tubal pregnancy. What method can solve the problem of the generative function in this
woman?
A. Correction factors of ovarian
B. * In Vitro Fertilization (IVF) and embryo transplantation
C. Intrauterine artificial insemination
D. Intracervical artificial insemination
E. Insemination sperm donor
198. What is not included in the complex examination of patients with endocrine form of infertility?
A. * definition of the Fallopian tubes
B. basal body temperature
C. determine the level of hormones in the blood
D. smears on the "hormonal mirror"
E. ultrasound control the growth of follicles during the menstrual cycle
199. A 26-year-old woman has attended maternity center complaining of her inability to become
pregnant despite 3 years of regular sex life. Examination revealed the following: increased body
weight; male-type pubic hair; excessive pilosis of thighs; ovaries are dense and enlarged; basal
body temperature is monophasic. The most likely diagnosis is:
A. * Ovaries sclerocystosis
B. Inflammation of uterine appendages
C. Adrenogenital syndrome
D. Premenstrual syndrome
E. Gonadal dysgenesis

200. A 31-year-old female patient complai-ns of infertility, amenorrhea for 2 years after the artificial
abortion that was complicated by endometritis. Objectively: examination of the external genitalia
revals no pathology, there is female pattern of hair distribution. According to the functional tests,
the patient has biphasic ovulatory cycle. What form of infertility is the case?
A. * Uterine
B. Ovarian
C. Pituitary
D. Hypothalamic
E. Immunologica
201. The patient 29 years old, complains of infertility. Sexual life has been leading in marriage for 4
years, hasn’t been preventing pregnancy. She didn’t have any pregnancy yet. An examination of
women is established: the development of genital organs were normal; The fallopian tubes are
passable. Basal [rectal] temperature for three menstrual cycles had single phase. The most probable
cause of infertility?
A. * Anovulatory menstrual cycle
B. Chronic adnexitis
C. Anomalies of genital organs
D. Immunological infertility
E. Genital endometriosis
202. The patient complains about infertility. Menstruations started at 16 years, are not regular, 5-6 days
after 28-35 days, to marriage - painful. Sexual life during 4 years. Did not use contraceptives,
become not pregnant. In childhood was ill on a measles, scarlet fever. Vaginally: the uterus is of
normal size, mobile, unpainful, adnexa are not palpated. In speculum: the uterine cervix is normal.
Analysis of sperm of husband - 55 mln of spermatozoa in 1 ml, 75% of them are mobile. Shuvarscy
test is positive. A basal temperature during 2 cycles is monotonous. Diagnosis?
A. Infertility I, tubal genesis
B. Infertility I, tubal-peritoneal genesis
C. * Infertility I, anovulatory cycles
D. Infertility I, masculine genesis
E. Infertility I, anomaly of development of privy parts
203. The woman, suffering from infertility came to the gynecologist with complain of delayed
menstruation. Which pregnancy test will be the most reliable in the early stages?
A. * Measurement of human chorionic gonadotropin in the blood
B. Immune hemagglutination inhibition test.
C. Reaction Galey Maynini
D. Measurement of the concentration of estrogen in the blood
E. Measurement of concentration of progesterone in the blood
204. A client is scheduled to have in vitro fertilization (IVF) as an infertility treatment. Which of the
following client statements about IVF indicates that the client understands this procedure?
A. "IVF requires supplemental estrogen to enhance the implantation process."
B. "The pregnancy rate with IVF is higher than that with gamete intrafallopian transfer."
C. * "IVF involves bypassing the blocked or absent fallopian tubes."
D. "Both ova and sperm are instilled into the openened of a fallopian tube."
E. Nothing of the above
205. A female patient, aged 25, suffers endocrine form of infertility for 5 years. What should be included
in investigations of this patient?
A. Ultrasonic monitoring of growth of follicles during the menstrual cycle.
B. Measuring basal temperature.
C. Determine the level of hormones in the blood.

D. Smears on the "hormonal mirror."


E. * All answers are correct.
206. 25-year-old patient notes the absence of pregnancy within 5 years. Operations were performed
twice at the tubal pregnancy. What method can solve the question of the generative function in this
woman?
A. Correction factors of ovarian
B. * In Vitro Fertilization (IVF) and embryo transplantation
C. Intrauterine artificial insemination
D. Intracervical artificial insemination
E. Insemination sperm donor
207. A 16-year-old female comes to the physician because of an increased vaginal discharge. She
developed this symptom 2 days ago. She also complains of dysuria. She is sexually active with one
partner and uses condoms intermittently. Examination reveals some erythema of the cervix but is
otherwise unremarkable. A urine culture is sent which comes back negative. Sexually transmitted
disease testing is performed and the patient is found to have gonorrhea. While treating this patient's
gonorrhea infection, treatment must also be given for which of the following?
A. Bacterial vaginosis
B. * Chlamydia
C. Herpes
D. Syphilis
E. Trichomoniasis
208. A 23 year-old G0 reports having a solitary, painful vulvar lesion that has been present for three
days. This lesion has occurred twice in the past. She states that herpes culture was done by her
doctor during her last outbreak and was negative. She is getting frustrated in that she does not know
her diagnosis. She has no significant previous medical history. She uses oral contraceptives and
condoms. She has had four sexual partners in her lifetime. On physical examination, a cluster of
three irregular erosions with a superficial crust is noted on the posterior fornix. Urine pregnancy
test is negative. You suspect recurrent genital herpes. How do you explain the negative culture?
A. Cultures were taken too early
B. The more definitive test would be serum herpes antibody testing
C. The cultures were refrigerated prior to transport to the lab
D. * Herpes cultures have a 10-20% false negative rate
E. The herpes virus cannot be recovered with recurrent infections
209. A 22 year-old P0 presents with a one-month history of profuse vaginal discharge with mild odor.
She has a new sexual partner with whom she has had unprotected intercourse. She reports mild to
moderate irritation, pruritus and pain. She thought she had a yeast infection, but had no
improvement after using an over-the counter antifungal cream. She is concerned about sexually
transmitted infections. Her medical history is significant for lupus and chronic steroid use. Pelvic
examination shows normal external genitalia, an erythematous vagina with a copious, frothy yellow
discharge and multiple petechial on the cervix. Vaginal pH is 7. Saline wet mount reveals motile,
flagellated organisms and multiple white blood cells. Which of the following is the most
appropriate treatment for this patient?
A. Clindamycin
B. Azithromycin
C. * Metronidazole
D. Ampicillin
E. Doxycycline

210. A 26 year-old G2P2 reports that she is sexually active with a new male partner. She is using oral
contraception for birth control and, as such, did not use a condom. She reports the new onset of
vulvar burning and irritation. She thought she had a cold about 10 days ago. Given her history,
which of the following is the most likely diagnosis in this patient?
A. * Herpes simplex virus
B. Primary syphilis
C. Secondary syphilis
D. Human immunodeficiency virus
E. Trichomonas
211. A 39 year-old G0 presents to the clinic reporting non-tender spots on her vulva for about a week.
No pruritus or pain is present. She also notes a brownish rash on the palms of her hands. She admits
to IV drug abuse. She was diagnosed as HIV-positive two years ago, but has not been compliant
with suggested treatment. On examination, three elevated plaques with rolled edges are noted on
the vulva. They are non-tender. A brown macular rash is noted on the palms of her hands and the
soles of her feet. What is the most appropriate next step in the management of this patient?
A. * Obtain a treponemal-specific test
B. Biopsy of the lesion
C. Colposcopic evaluation of the vulvar lesions
D. Culture the base of the lesion
E. Perform a wet prep
212. A 20 year old woman presents with a 2-day history of dysuria and increased urinary frequency, she
is recently married and was not sexually active prior to the marriage, gynecologic exam reveals no
evidence of discharge, vaginitis or cervicitis, urinalysis reveals 14 white blood cells per
high-powered field with many gram-negative rods, which of the following is the most appropriate
pharmacotherapy?
A. * Trimethropim-Sulfamethoxazole
B. Ceftriaxone
C. Fluconazole
D. Gentamicin
E. Metronidazole
213. Woman with large, deep ulcers on genital with gray/yellow exudate, the ulcers are
well-demarcated borders and soft, the patient also has severe lymphadenopathy, which of the
following could be the case?
A. * Haemophilus (chancroid)
B. HSV-vesicles
C. Lymphogranuloma venerum
D. HPV (wart)
E. None of them
214. A 37 years old woman has been examined and a fishy-odor discharge is noted, itching/ burning and
inflamed vagina not to see, what is the diagnosis?
A. Candida vaginitis
B. Cancer
C. * Bacterial vaginosis
D. Cervicitis
E. HPV
215. The chief complaint interview on a client reveals vaginal discharge with itching and burning. The
client also reveals she experiences dyspareunia. If her diagnosis is monilial vulvovaginitis, you
would expect the wet mount slide to contain which of the following?
A. Bacteria

B. Clue cells
C. Trichomonads
D. * Budding hyphae
E. All of the above
216. A 20.y.old woman is diagnosed with trichomonas and prescribed an antibiotic. She calls her
physician complaining of the flushing, nausea, and emesis. What antibiotic was the patient
prescribed and what should she have been warned of?
A. * Metronidazole - alcohol
B. Metronidazole - sunlight
C. Metronidazole – smoking
D. Azithromycin – spicy food
E. Azithromycin – oral contraceptive
217. The patient complains of feeling of itching, pain in the vagina, large selection of white disharge. On
examination: vaginal mucosa edematous flushed, in the lateral fornices - the accumulation of white
layers, similar to the chees. What is the previous diagnosis?
A. Trichomonas colpitis
B. chlamydial colpitis
C. urogenital mycoplasmosis
D. bacterial vaginosis
E. * candidiasis vaginitis
218. The patient D, 22 years old was diagnosed Trichomonas colpitis. Select a product for the treatment
of this patient
A. * tinidozol
B. ciprofloxacin
C. ampicillin
D. Biseptol
E. Diflucan
219. A 90-year-old G7P7 woman presents with severe vaginal prolapse. The entire apex, anterior and
posterior wall are prolapsed beyond the introitus. She cannot urinate without reduction of the
prolapse. Hydronephrosis was noted on ultrasound of the kidneys and it is thought to be related to
the prolapse. She has a long-standing history of diabetes and cardiac disease. She is not a candidate
for general or regional anesthesia. She has failed a trial of pessaries. Which of the following is the
next best step in the management of this patient?
A. Do nothing and observe.
B. Anterior and posterior repair
C. * Colpocleisis
D. Sacrospinous fixation
E. Sacrocolpopexy
220. A 48-year-old G0 woman comes to the office for a health maintenance exam. She is healthy and not
taking any medications. She has no history of abnormal Pap smears or sexually transmitted
diseases. She is not currently sexually active. Her menstrual cycles are normal and her last cycle
was three weeks ago. She smokes one pack of cigarettes per day. Her mother was diagnosed with
endometriosis and had a hysterectomy and removal of the ovaries at age 38. She is 5 feet 4 inches
tall and weighs 130 pounds. On pelvic examination, the patient had a palpable left adnexal mass.
An ultrasound was obtained, which showed a 4 cm complex left ovarian cyst and a 2 cm simple
cyst on the right ovary. What is the most appropriate next step in the management of this patient?
A. CAT scan of the abdomen and pelvis
B. MRI of the pelvis
C. Abdominal hysterectomy and bilateral salpingoophorectomy (TAH/BSO)

D. * Repeat ultrasound in 2 months


E. Oral contraceptives
221. A 45 year-old G5P5 premenopausal woman was initially seen in your office for work-up and
evaluation of a FIGO grade 2 endometrial cancer that was diagnosed by her gynecologist. Which of
the following is the most appropriate treatment for this patient?
A. Total laparoscopic hysterectomy with bilateral salpingoophorectomy
B. Vaginal hysterectomy with bilateral salpingoophorectomy
C. * Total abdominal hysterectomy, bilateral salpingoophorectomy, bilateral pelvic and
paraaorticlymphadenectomy, pelvic washings
D. Supracervical abdominal hysterectomy with ovarianpreservation
E. Megace (megestrol acetate)
222. A 31 year-old G0 woman has been diagnosed with uterine fibroids. A fluid contrast ultrasound
confirmed the presence of two intramural fibroids measuring 5 x 6 cm and 2 x 3 cm that appear to
be distorting the patient’s uterine cavity. The patient has a two-year history of infertility. She has
had a thorough infertility work up. No etiology for her infertility has been identified. Which of the
following treatments is most appropriate for this patient?
A. Hysteroscopy
B. Uterine curettage
C. Gonadotropin-releasing hormone agonist
D. Uterine artery embolization
E. * Myomectomy
223. A 72 year-old G3P2 postmenopausal woman is referred to your office by her internist after a
work-up for abdominal bloating revealed a large pelvic mass on transvaginal ultrasound and an
elevated CA-125. She has no significant medical history and only a prior appendectomy. The CT
scan showed a large heterogenous pelvic mass measuring 20 x 13 x 10 cm. There was a moderate
amount of ascites and likely “omental caking.” There was no significant pelvic or abdominal
lymphadenopathy, and the chest x-ray showed only a small right pleural effusion. On pelvic
examination, there is minimal cul-de-sac nodularity and the mass is readily palpable and somewhat
mobile. There is an obvious fluid wave. Which of the following is the most appropriate next step in
the management of this patient?
A. Thoracentesis
B. Exploratory laparotomy and tumor debulking (cytoreduction)
C. Paracentesis
D. Diagnostic laparoscopy and tissue biopsies
E. Neoadjuvant chemotherapy
224. Which of the following is the risk factor for vesicovaginal fistula?
A. Child birth trauma
B. Prolonged labour
C. Pelvic irradiation
D. Pelvic surgery
E. * All of them
225. A previously healthy 27-year-old nulligravid woman comes to the emergency department because
of a 2-day history of moderate-to-severe pain and swelling of the left labia. She is sexually active
and uses condoms inconsistently. Her temperature is 37.2°C (99°F), pulse is 92/min, respirations
are 18/min, and blood pressure is 115/75 mm Hg. Pelvic examination shows a 4 x 3-cm, tender,
fluctuant mass medial to the left labium majus compromising the introital opening. Which of the
following is the most appropriate next step in management?
A. Vulvectomy
B. Ultrasound-guided needle aspiration of the mass
C. Administration of intravenous penicillin G

D. Administration of intravenous metronidazole


E. Incision and drainage
226. 26 years old patient has formed formation of purulent inflammation of the appendages of the
uterus. What would you recommend?
A. * puncture through the posterior vaginal vault, draining pus cavity and the introduction of antibiotics
into it
B. surgery
C. pirogenal therapy
D. electrophoresis of zinc
E. Nothing above
227. Woman I., aged 38 years was admitted to gynecologic department with complaints on cramp-like
pains in the lower abdomen and moderate blood-tinged discharge from vagina. In past-history:
labor 1, medical abortions – 2. Patient notes delay of menstruation, instantaneous loss of
consciousness. Objectively: skin integuments are pale, covered with cold perspiration. Arterial
pressure – 90/50 mm Hg., pulse rate – 120 beats/min. On vaginal examination: somewhat enlarged
uterine, in the right – movable formation of ovoid form, soft consistency is palpable. What
treatment should be carried out?
A. * Surgical intervention.
B. Treatment with estrogens.
C. Treatment with androgens.
D. Symptomatic treatment.
E. Treatment with gestagens.
228. Patient, aged 32 years was admitted to gynecologic department with complaints on sharp pain in
the lower abdomen. Menses were 2 weeks ago, timely. On bimanual vaginal examination: vagina
and uterine cervix are without peculiarities. Examination of the uterine body and appendages is
impossible to perform due to tenderness and tension of the anterior abdominal wall. Posterior
fornix overhangs, painful. Apoplexy of the ovary is suspected. What should be done to precise
A. diagnosis?
To perform bimanual examination again under narcosis.
B. * Paracentesis of the abdominal cavity through posterior fornix of vagina.
C. Ultrasonic investigation.
D. Symptomatic treatment
E. Hysteroscopic examination.
229. Patient, aged 23 years was admitted to the gynecologic unit complaining of pain in the lower
abdomen, general weakness, collaptoid state, blood-tinged discharge from the reproductive
passages. In the past history: patient had chronic adnexitis; last 2 months – no menstruation,
sometimes bloody discharge was noted. Objectively: skin and mucous membranes are pale. Arterial
pressure – 90/60, pulse rate – 100 beats/min. Abdomen is swelled a little, painful in the lower
portions. Shchyotkin’s symptom is positive. Vaginal examination: uterine is enlarged up to 5-6
weeks, soft, painful one on palpation. Appendages are not palpable clearly due to a sharp tension of
the anterior abdominal wall. Posterior fornix hangs over, painful one. Dark blood-tinged discharge
continues. What method of investigation is the most informative?
A. Hysteroscopy.
B. Laparoscopy.
C. Ultrasonic investigation.
D. * Abdominal paracentesis.
E. Dopplerometry.

230. A 30-year-old woman complains of irregular copious painful menstruations, pain irradiates to the
rectum. Anamnesis states 10- year-long infertility. On bimanual examination: uterus is of normal
size; uterine appendages on the both sides are corded, with rectricted mobility, painful; there are
dense nodular painful growths detected in the posterior fornix. A doctor suspects endometriosis.
What method allows to verify this diagnosis?
A. * Laparoscopy
B. Diagnostic curettage of uterine cavity
C. Paracentesis of posterior fornix
D. Uterine probing
E. Hysteroscopy
231. Which treatment is followed by a 48 years patient with severe cervical dysplasia and ovarian cyst:
A. diatermokonization of cervix
B. diathermocoagulation of cervix
C. * hysterectomy with appendages
D. treatment by solkovagin
E. hysterectomy without appendages
232. Patient 22 years. Complains about pain in a right labia pudenda majora, rise of body temperature to
38.0°C. At the review of genital organs the considerable increasing of right large sexual lip
definites, especially in the lower third. Erythema, edema, at palpation acutely painful, fluctuation is
determined. To conduct vaginal examination due to acute pain is impossible. Blood test:
Leucocytes — 10,0 x 109 per cu mm. What method is main?
A. * The dissection and drainage of abscess.
B. To withdraw a bartolin gland within the limits of healthy tissue.
C. To appoint physical therapy procedures.
D. To appoint compresses with liniment.
E. To expect a spontaneous regeneration of abscess.
233. 26 years old patient has formed formation of purulent inflammation of the appendages of the
uterus. What would you recommend?
A. * puncture through the posterior vaginal vault, draining pus cavity and the introduction of
B. antibiotics into it
C. surgery
D. pirogenal therapy
E. electrophoresis of zinc
234. A 24 year-old G0 presents with a one-year history of introital and deep thrust dyspareunia. She also
has a 2- year history of severe dysmenorrhea, despite the use of oral contraceptives. She underwent
a diagnostic laparoscopy 6 months ago that showed minimal endometriosis with small implants in
the posterior cul de sac only, which were ablated with a CO2 laser. On further questioning, she
reports significant urinary frequency, urgency and nocturia. A recent urine culture was negative.
What is the most likely diagnosis in this patient?
A. Acute cystitis 5%
B. * Interstitial cystitis
C. Acute urethral syndrome
D. Acute urethritis
E. Salpingitis
235. A woman who has just undergone a first trimester abortion will be using oral contraceptives. To be
protected from pregnancy, she should be advised to:
A. Avoid sexual contact for at least 10 days after starting the pill.
B. Use condoms and foam for the first few weeks as backup.
C. * Use another method of contraception for one week after starting the pill.

D. Begin sexual relations once vaginal bleeding has ended.


E. Contraceptive pill start work after next period
236. A woman was treated recently for toxic shock syndrome. She has intercourse occasionally and uses
over-the-counter protection. Based on her history, what contraceptive method should she and her
partner avoid?
A. * Cervical cap
B. Condom
C. Vaginal film
D. Vaginal sheath
E. Combine contraceptive pill
237. An unmarried young woman visit to family planning center for consultation. She describes her sex
life as “active” with “many” partners. She wants a contraceptive method that is reliable and does
not interfere with sex. She requests an intrauterine device (IUD). The physician's most appropriate
response would be:
A. “The IUD does not interfere with sex.”
B. * “The risk of pelvic inflammatory disease (PID) will be higher for you.”
C. “The IUD will protect you from sexually transmitted infections.”
D. “Pregnancy rates are high with the IUDs.”
E. “The IUD is the best choice for you”
238. A married couple is discussing male and female sterilization with the physician. Which of the
following statements is most appropriate for the physician to make?
A. “Male and female sterilization methods are 100% effective.”
B. “A vasectomy may have a slight effect on sexual performance.”
C. “Tubal ligation can be easily reversed if you change your mind in the future.”
D. * “Major complications after sterilization are rare.”
E. “A vasectomy is more preferable than tubal ligation”
239. A woman had unprotected intercourse 36 hours ago and is concerned that she may become
pregnant because it is her “fertile” time. She asks the physician about emergency contraception.
The physician would tell her that:
A. It is too late since she needed to begin treatment within 24 hours after intercourse.
B. Prevent, an emergency contraceptive method, is 98% effective in preventing pregnancy.
C. * An over-the-counter antiemetic can be taken 1 hour prior to each contraceptive dose to prevent
nausea and vomiting.
D. The most effective approach is to use a progestin-only preparation.
E. don’t worry you have additional 48 hour to make decision about taking the pill
240. A client 34 years old with 14 years history of diabetes asks the physician for advice regarding
methods of birth control. Which method of birth control is most suitable for the client with
A. diabetes?
Vaginal ring
B. Oral contraceptives
C. * Diaphragm
D. Contraceptive sponge
E. Progestine only pill
241. The adolescent client asks the physician why she has a 26-day cycle and her friend has a 29-day
cycle? The physician's best response is: "The length of your menstrual cycle is individual and is
controlled by hormones from the:
A. Uterus and no other structure
B. Ovaries and the uterus
C. Pituitary gland and uterus

D. * Ovaries and pituitary gland


E. Pituitary gland and no other structure
242. At 32 weeks' gestation, a 15-year-old primgravida client who is 1.57 m tall has gained a total of 9
kg, with a 450g gain in the last 2 weeks. Urinalysis reveals negative glucose and a trace of protein.
Which of the following factors increases this client's risk for preeclampsia?
A. Total weight gain.
B. Short stature.
C. * Adolescent age group.
D. D. Proteinuria.
E. E. Gestational age
243. A 32-year-old multigravida returns to the clinic for a routine prenatal visit at 36 weeks' gestation.
She has had a prior pregnancy with pregnancy induced hypertension. The assessments during this
visit include BP 140/90, Ps 80, and + 2 edema of the ankles and feet. Based on the client's past
history and current assessment, what further information should the physician obtain to determine
if this client is becoming preeclamptic?
A. Headaches.
B. Blood glucose level.
C. * Proteinuria.
D. Edema in lower extremities.
E. Gestational age of previous pregnancy when preeclampsia was diagnosed
244. A primigravid client's baseline blood pressure at her initial visit at 12 weeks' gestation was 110/70
mm Hg. During an assessment at 38 weeks' gestation, which of the following data would indicate
mild preeclampsia?
A. Blood pressure of 160/110 mm Hg on two separate occasions.
B. Proteinuria, more than 5 g in 24 hours.
C. Serum creatinine concentration of 1.4 ml/dl.
D. * Weight gain of 900g in the last week.
E. Swelling of the feet and ankles
245. During a routine checkup of a 16-year-old client at 34weeks' gestation diagnosed with mild
preeclampsia, assessment reveals that the client has gained 900 g in the past week and her current
blood pressure is 130/85 mm Hg. Which of the following assessment findings would provide
further evidence to support the client's diagnosis?
A. Pounding headache after reading.
B. History of urinary tract infection.
C. Frequent voiding in large amounts.
D. * Mild edema in hands and face.
E. Blood pressure last week 140/85 mm Hg
246. A 17-year-old client at 33 weeks' gestation diagnosed with mild preeclampsia is prescribed bed rest.
The physician instructs the client to contact the nurse immediately if she experiences which of the
following?
A. * Blurred vision.
B. Ankle edema.
C. Increased energy levels.
D. Mild backache.
E. General weakness
247. One week after her prenatal visit, a primigravid client at 38 weeks' gestation diagnosed with mild
preeclampsia calls the clinic physician complaining of a continuous headache for the past 2 days
accompanied by nausea. The client does not want to take aspirin. Which of the following responses
by the physician would be most appropriate?

A. "Take two acetaminophen tablets. They aren't as likely to upset your stomach."
B. * "You should be examined today. Come to the clinic this morning."
C. "You need to lie down and rest. Have you tried placing a cool compress over your head?"
D. "Take aspirin with codeine.”
E. "Don’t worry. Sometimes pregnant woman feels headache."
248. When reviewing the prenatal records of a 16-year-old primigravid client at 37 weeks' gestation
diagnosed with severe preeclampsia, the physician would interpret which of the following as most
indicative of the client's diagnosis?
A. Blood pressure of 138/94 mm Hg.
B. * Severe blurring of vision.
C. Less than 2 g of protein in a 24-hour sample.
D. Weight gain of 450 g in 1 week.
E. Ankle edema
249. The physician orders intravenous magnesium sulfate for a primigravid client at 38 weeks' gestation
diagnosed with severe preeclampsia. Which of the following medications would the physician have
readily available at the client's bedside?
A. Diazepam.
B. Hydralazine.
C. * Calcium gluconate.
D. Oxytocin.
E. Nifedipine
250. For the 29-year-old multigravid client at 37 weeks' gestation who is receiving intravenous
magnesium sulfate for severe preeclampsia, which of the following assessment findings would alert
to suspect hypermagnesemia?
A. * Decreased deep tendon reflexes.
B. Cool skin temperature.
C. Rapid pulse rate
D. Tingling in the toes.
E. Decrease of ankle edema
251. A 28-year-old multigravid client at 37 weeks' gestation arrives at the emergency department via
ambulance with a blood pressure of 160/104 mm Hg and +3 reflexes without clonus. The client,
who is diagnosed with severe preeclampsia, asks the physician, "What is the cure for my high
blood pressure?" Which of the following would the physician identify as the primary cure?
A. Administration of glucocorticoids.
B. * Vaginal or cesarean delivery of the fetus.
C. Sedation with anti-seizure medication.
D. Reduction of fluid retention with thiazide diuretics.
E. Administration of antihypertensive medication.
252. Which of the following would the physician identify as the priority to achieve when developing the
plan of care for a primigravid client at 38 weeks' gestation who is hospitalized with severe
preeclampsia and receiving intravenous magnesium sulfate?
A. Decreased generalized edema within 8 hours.
B. Decreased urinary output during the first 24 hours
C. Sedation and decreased reflex excitability within 48 hours.
D. * Absence of any seizure activity during the first 48 hours.
E. Decreased breath movement less than 12 per minute

253. When administering intravenous magnesium sulfate as ordered for a client at 34 weeks' gestation
with severe preeclampsia, the physician would explain to the client and her family that this drug
acts as which of the following?
A. Peripheral vasodilator
B. Antihypertensive.
C. * Central nervous system depressant.
D. Sedative-hypnotic.
E. Diuretic
254. Soon after admission of a primigravid client at 38 weeks' gestation with severe preeclampsia, the
physician orders a continuous intravenous infusion of 5% dextrose in Ringer's solution and 4 g of
magnesium sulfate. While the medication is being administered, which of the following assessment
findings should the physician report immediately?
A. * Respiratory rate of 12 breaths /minute
B. Patellar reflex of+2.
C. Blood pressure of 160/88 mm Hg.
D. Urinary output exceeding intake.
E. Increase of deep tendon reflexes
255. As the physician enters the room of a newly admitted primigravid client diagnosed with severe
preeclampsia, the client begins to experience a seizure. Which of the following should the
physician do first?
A. Insert an airway to improve oxygenation.
B. Note the time when the seizure begins and ends.
C. * Call for immediate assistance.
D. Turn the client to her left side.
E. Catheterize central vein
256. After administering hydralazine 5 mg intravenously as ordered for a primigravid client with severe
preeclampsia at 39 weeks' gestation, the physician would be alert for which of the following?
A. * Tachycardia.
B. Bradypnea.
C. Polyuria.
D. Dysphagia.
E. Constipation
257. A primigravid client at 40 weeks' gestation with severe preeclampsia exhibits hyperactive, very
brisk patellar reflexes with two beats of ankle clonus present. The physician documents the patellar
reflexes as which of the following?
A. 1+
B. 2+
C. 3+
D. * 4+
E. 5+
258. A 16-year-old unmarried primigravid client at 37 weeks' gestation with severe preeclampsia is in
early active labor. Her mother is at the bedside. The client's blood pressure is 164/110 mm Hg.
Which of the following would alert the physician that the client may be about to experience a
seizure?
A. Decreased contraction intensity.
B. Decreased temperature.
C. * Epigastric pain.
D. Hyporeflexia.

E. Increased ankle edema


259. If a client at 36 weeks' gestation with eclampsia begins to exhibit signs of labor after an eclamptic
seizure, for which of the following would the physician assess?
A. * Abruptio placentae.
B. Transverse lie.
C. Placenta accreta.
D. Uterine atony.
E. Placenta previa
260. For a multigravid client at 39 weeks' gestation with suspected HELLP syndrome, the physician
would immediately notify which of the following laboratory test results?
A. Hyperfibrinogenemia.
B. Decreased liver enzymes
C. * Thrombocytopenia.
D. Hypernatremia.
E. Anemia
261. A 16-year-old primigravid client admitted at 38 weeks' gestation with severe pregnancy-induced
hypertension is given intravenous magnesium sulfate and lactated Ringer's solution. Which of the
following assessments should the physician prescribe?
A. Urinary output even'8 hours.
B. Deep tendon reflexes even' 4 hours.
C. * Respiratory rate every hour.
D. Blood pressure every 6 hours.
E. Uterine contraction every 4 hours
262. Screening at 24 weeks’ reveals that a pregnant woman has gestational diabetes mellitus (GDM). In
planning her care, the doctor and the woman mutually agree that an expected outcome is to prevent
injury to the fetus as a result of GDM. The doctor identifies that the fetus is at greatest risk for:
A. * Macrosomia
B. Congenital anomalies of the central nervous system
C. Preterm birth
D. Low birth weight
E. Prolong labor
263. In her 36th week of gestation, a client with type 1 diabetes has a 4300 g infant by cesarean birth.
When caring for an infant of a diabetic mother (IDM), the doctor should monitor for signs of:
A. Meconium ileus
B. Physiologic jaundice
C. Increased intracranial pressure
D. * Respiratory distress syndrome
E. Excessive weight loss
264. A client at 6 weeks' gestation who has type 1 diabetes is attending the prenatal clinic for the first
time. Why do the insulin requirements may decrease during the first trimester?
A. Body metabolism is sluggish in the first trimester
B. * Morning sickness may lead to decreased food intake
C. Fetal requirements of glucose in this period are minimal
D. Hormones of pregnancy decrease the body's need for insulin
E. Placental hormones work as insulin
265. A primigravida with pregestational type 1 diabetes is at her first prenatal visit. When discussing
changes in insulin needs during pregnancy and after birth, the doctor explains that based on her
blood glucose levels she should expect to increase her insulin dosage between the:

A. 10th and 12th weeks of gestation


B. 18th and 22nd weeks of gestation
C. * 24th and 28th weeks of gestation
D. 36th week of gestation and the time of birth
E. In postpartum period
266. A primigravid client who was successfully treated for preterm labor at 30 weeks' gestation had a
history of mild hyperthyroidism before becoming pregnant. Which recommendation the doctor
should give the client about treatment when planning her discharge?
A. * Continue taking low-dose oral propylthiouracil (PTU) as ordered.
B. Discontinue taking the methimazole (Tapazole) until after delivery.
C. Consider breast-feeding the neonate after the delivery.
D. Contact the physician if bradycardia occurs.
E. Taking medication can cause congenital fetal abnormalities
267. A 30-year-old multigravid client at 8 weeks' gestation has a history of insulin-dependent diabetes
since age 18. What is the insulin needs during the first trimester?
A. They will increase.
B. * They will decrease.
C. They will remain constant.
D. They will be unpredictable.
E. They depend from baby’s glucose level
268. Primigravid client at 10 weeks' gestation who has a 5-year history of insulin-dependent diabetes
admit to the hospital for her first prenatal visit. What is NOT the complication of pregnancy in this
woman?
A. Candida albicans infection.
B. * Twin-to-twin transfer.
C. Polyhydramnios.
D. Preeclampsia.
E. Diabetic fetopathy
269. At previous antepartal visit, the physician taught a pregnant woman diagnosed with class II cardiac
disorder about measures to use to lower her risk for cardiac decompensation. The woman would
demonstrate need for further instruction if she
A. Increased roughage in her diet
B. * Remains on bed rest only getting out of bed go to the bathroom
C. Sleep 10 hours every night and rests after meals
D. States she will call the physician immediately if she experiences any pain or swelling in her legs
E. Knows signs of cardiac decompensation
270. A client with a past medical history of ventricular septal defect repaired in infancy is seen at the
prenatal clinic. She is complaining of dyspnea with exertion and being very tired. Her vital signs
are t 36.7°C, Ps 80/min, Breath movement 20/min, BP 116/72 mm Hg. She has + 2 pedal edema
and clear breath sounds. As the physician plans this client's care, which class of organic heart
disease she has according to the New York Heart Association functional classification?
A. Class I
B. * Class II
C. Class III
D. Class IV
E. Class V

271. While caring for a primigravid client with class II heart disease at 28 weeks' gestation, the
physician would instruct the client to contact her physician immediately if the client experiences
which of the following?
A. Mild ankle edema.
B. Emotional stress on the job
C. Weight gain of 500 g in 1 week.
D. * Increased dyspnea at rest.
E. General weakness
272. In the 30years old primapara intensive uterine contractions with an interval of 1-2 min, duration 50
sec have begun. In time of the fetal head delivery the patient complaints on severe pain in the
perineum. The perineum is 5 cm, its skin become pale. What is it necessary to perform:
A. * Perineotomy
B. Episiotomy.
C. Protection of the perineum.
D. Vacuum - extraction of the fetus.
E. Waiting tactics.
273. The primapara 24 is admitted in to the hospital due to high body temperature – 38,7 0, 1 stage of
labor, regular uterine contractions. Sizes of pelvis: 25-28-30-20 cm. Abdominal circumference is
100 cm, level of uterine fundus 28 cm, presenting part is absent. Right side the fetal head is
palpated, left – the breech, fetal heart sounds are absent. Vaginal examination: the uterine cervix is
fully dilated, amniotic membrane is whole. What is the tactic of labor conducting?
A. Cesarean section after full dilatation
B. Cesarean section immediately
C. External version of the fetus on a head
D. Stimulation of uterine contractions
E. * Classic obstetric version of the fetus
274. In patient 25 years, labor III. The pelvic sizes: 24-27-30-19 cm. After stormy uterine contractions
and pushing at a highly standing fetal head and positive Vasten’ sign uterine contractions were
stopped suddenly, bloody excretions from a vagina appeared, fetal heart rate is not listened. The
condition of patient suddenly became worse, blood pressure went down to 70 mm Hg, pulse 140 in
a 1 minute, the skin is pale. Reason of the shock condition?
A. * Uterine rupture
B. Threatened rupture of uterus
C. Abruption placentae
D. Syndrome of squeezing of lower hollow vein
E. Placenta previa
275. A 30 years old woman has the 2-nd labour that has been lasting for 14 hours. Hearbeat of fetus is
muffled, arrhythmic, 100/min. Vaginal examination: cervix of uterus is completely opened, fetus
head is at the pelvis outlet. Saggital suture is in the straight diameter, small fontanel is near
symphysis. What is the further tactics of handling the delivery?
A. * Use of obstetrical forceps
B. Stimulation of labor activity by oxytocin
C. Cesarean section
D. Cranio-cutaneous (Ivanov's) forceps
E. Use of cavity forceps

276. A woman is 34 years old, it is her tenth labor at full term. It is known from the anamnesis that the
labor started 11 hours ago, labor was active, painful contractions started after discharge of waters
and became continuous. Suddenly the parturient got knife-like pain in the lower abdomen and labor
activity stopped. Examination revealed positive symptoms of peritoneum irritation, ill-defined
uterus outlines. Fetus was easily palpable, movable. Fetal heartbeats weren’t auscultable. What is
the most probable diagnosis?
A. * Rupture of uterus
B. Uterine inertia
C. Discoordinated labor activity
D. Risk of uterus rupture
E. II labor period
277. Multipara, the second stage of delivery, fetal head is in the pelvic cavity. Contractions last for 60
seconds in 2 minutes. Fetal heart beat slowed to 100 per min. What is the management?
A. * Applying obstetric forceps
B. Oxytocin stimulation of uterus activity
C. Perform a classic podalic version
D. C-section
E. nothing above
278. The woman is admitted to the maternity home with discontinued uterine contractions and slight
bloody discharges from the vagina. The condition is severe, the skin is pale, consciousness is
confused. BP – 80/40 mm Hg. The heart rate of the fetus is not determined. In anamnesis there was
a cesarean section a year ago. Establish the diagnosis:
A. Placental presentation
B. Placental presentation
C. * Uterine rupture.
D. Premature expultion of the amniotic fluids
E. Nothing above
279. In the primapara, 30 years, intensive uterine contractions with an interval of 1-2 min, duration 50
sec have begun. In vaginal examination cervical dilation is complete. Amniotic sac is intact. Fetal
head is present in 0 station. What is it necessary to perform:
A. Vacuum - extraction of the fetus.
B. Protection of the perineum.
C. Episiotomy.
D. * Amniotomy
E. Nothing
280. Pregnant N., 25 years is delivered in the maternity department with complaints of periodic pains in
lower part of abdomen and lumbal region, during 7 hours and bloody excretions from a vagina,
which appeared 1 hour ago. Amniotic fluid are present. Pregnancy 4, labor is first, previous 3
pregnancies ended by artificial abortion. Fetal heart rate is 136 in 1min. At vaginal examination: the
uterine cervix is effaced, opening 6-7 cm, from one side soft spongy tissue is palpated before the
presenting head, the amniotic membrane is whole. Blood loss is 50 ml. What is the doctor’ tactic?
A. Stimulation of labor
B. * Amniotomy
C. Obstetric forceps
D. The fetal destroying operation
E. Cesarean section

281. In Primapara in I period of labor acute pain in the region of uterine fundus appeared suddenly, with
insignificant bloody excretions from a vagina. Uterus in hypertonus. Fetal heart rate – 175. In
anamnesis: acutening of chronic pyelonephritis during pregnancy. At vaginal examination: the
uterine cervix is effaced, opening of uterine cervix 4 cm. Amniotomy is perfomed. Amniotic fluid
released with blood. What is the doctor’ tactic?
A. Stimulation of labor.
B. Treatment of fetal hypoxia
C. * Cesarean section immediately.
D. To appoint tocolitics
E. To appoint coagulants
282. Patient C., 26 years, 18 hours are found in labor: pushing appeared hour ago – on 30 sec. in 3-4
minutes. Fetal heart rate is arhythmical, to 100 in 1 min. It is definite at vaginal examination, that a
head is found in narrow part of cavity of small pelvis. Your subsequent obstetric tactic?
A. * Obstetrical forceps
B. The cardiomonitoring supervision
C. Cesarean section.
D. To execute perineotomia.
E. To conduct labor conservative
283. Pregnant 24 years, the first pregnancy, I labor. Regular uterine contractions. At vaginal
examination: the cervical opening is 4 cm, an amniotic sac is whole, the fetal head is fixed in the
pelvic inlet. Sagittal suture is in a transversal size, the small fontanel is in the center of pelvis to the
left. What is the moment of the labor biomechanism?
A. The V moment of the labor biomechanism
B. The ІІ moment of the labor biomechanism
C. The ІІІ moment of the labor biomechanism
D. The IV moment of the labor biomechanism
E. * I moment of the labor biomechanism
284. The first delivery, II period. The fetal lie is longitudinal. The head presents, that it can't be
determinate by external maneuvers. In internal examination: the uterine cervix is effaced, dilatation
is full, membranes are absent. The sagittal suture is in a direct size, small fontanel is under the
pubis. In the pushing the fetal head appears from a vulva. What area of pelvis a fetal head
A. occupies?
Pelvic inlet
B. That is pressed to pelvic inlet plane
C. * The area of pelvic outlet
D. The area of wide part of a cavity of a small pelvis
E. The area of narrow part of a cavity of a small pelvis
285. Patient S., labor first, at term. Uterine contractions on 45-50 sec, after 2-3 min.. Fetal heart rate is
rhythmic, 144 in 1 min, a head is in the pelvic inlet. Vaginally: cervix is effaced, dilating is full,
amniotic membrane is absent. What is period of labor?
A. First.
B. * Second.
C. Preliminary.
D. Finishing.
E. Third.
286. Primapara N., 20 years, II pregnancy, I labor. The fetal lie is longitudinal, the fetal back is
anteriorly. The fetal heart rate is clear, rhythmic. Vaginal examination: the cervix is effaced,
opening is full, an amniotic sac is absent. Head of fetus in the plane of pelvic outlet. Sagittal suture
is in a direct size, small fontanel is under the pubis. What moment of the labor biomechanism at the
anterior type of occipital presentation is ended?

A. The І moment of the labor biomechanism


B. * II moment of the labor biomechanism
C. The ІІІ moment of the labor biomechanism
D. The IV moment of the labor biomechanism
E. The V moment of the labor biomechanism
287. The patient is admitted to delivery department. In examination longitudinal lie, I position, posterior
variety of the fetus is exposed. What is the leading point at the posterior type of occipital
presentation?
A. Small fontanel
B. * The midpoint of sagittal suture
C. Large fontanel
D. Chin
E. Subtongue bone
288. Patient II, labor first, at term. The patient’ condition is satisfactory. The new-born is just delivered.
The umbilical cord hangs down from a vagina and increases in its length. Bleeding is not present.
Uterus is in normal tonus. How do you called this positive sign of placenta separation?
A. Dovshenko sign
B. * Alfeld sign
C. Shreder' sign
D. Pupil sign
E. Hehar sign
Назва наукового напрямку (модуля): Семестр: 11
Акушерство та гінеколонія СРС
Опис:
6 курс 11семестр
Перелік питань:
1. Carol LeBec, age 16, comes to the clinic for contraceptive counseling and selection. Before
counseling Ms. LeBec, what should the physician do first?
A. Obtain a complete health history.
B. Perform a thorough physical examination.
C. * Evaluate personal beliefs about family planning.
D. Determine why the client seeks counseling.
E. Ask about menarche
2. Sintia, age 20, comes to the clinic for contraceptive counseling and selection. The physician advises
to teach Sintia about oral contraceptives because they are easy to use. How should the physician
proceed?
A. Review all contraceptive choices so that the client will understand the ease of taking one pill every
day.
B. Discuss barrier methods because the client may forget to take the pill every day.
C. Ask the client which method she would like to learn about
D. * Check for contraindications and discuss methods that best fit the client's needs and life-style.
E. Explain advantages of barrier method
3. Laura Hunt, age 37, comes to the family planning clinic for contraceptive advice. Her history
reveals frequent urinary tract infections. Which of the following contraceptive methods would be
most appropriate for her?
A. * condom
B. cervical cap
C. contraceptive sponge
D. diaphragm
E. spermicide
4. Neatha, age 32, comes to the family planning clinic for contraceptive advice. Her history reveals
frequent urinary tract infections. Which of the following assessment findings would contraindicate
an oral contraceptive for Ms. Hunt?
A. * history of a benign liver tumor
B. 10 pounds under her ideal weight
C. history of dysmenorrhea since age 15
D. family history of ovarian cancer
E. monogamous relationship
5. A 15-year-old girl is brought to the office by her mother because of abdominal pain and
constipation for the past several weeks. Her mother says, "She is getting almost all A's in school
and she is on the track team." You ask the patient about her diet and she responds, "I'm kind of a
picky eater." She requests a laxative to help with her constipation. She is 158 cm tall and weighs 43
kg; BMI is 18 kg/m2. Pulse is 65/min. Specific additional history should be obtained regarding
which of the following?
A. Color, caliber, and frequency of bowel movements
B. Exposure to sexually transmitted diseases.
C. Family history of irritable bowel syndrome.
D. * Menstrual history.
E. Use of illicit drugs.
6. An 18 year-old college student is seen in the emergency room, claiming she was raped by a 29
year-old janitor in her dorm 4 hours ago. He threatened her with a knife and she did not resist. She
appears calm and has a flattened affect when the history is taken. Currently, she is sexually active
with a fellow student and is taking birth control pills. The alleged attacker used a condom, which
she helped him put on. The student is 173cm tall and weighs 82kg. She is a cheerleader for the
college football team. Examination reveals no bruising and gynecologic examination reveals no
apparent injuries. Which of the following statements is the most likely explanation for this patient’s
presentation?
A. The attack was not emotionally traumatic
B. * The student is in shock
C. The attack never happened
D. The student cooperated with the attacker
E. The student suffers from chronic depression
7. A 32 year-old G0 woman comes to the office due to the inability to conceive for last two years. She
reports having been on oral contraceptives for 8 years prior. She had menarche at age 14 and has
had irregular cycles about every 3 months until she started oral contraceptives, which regulated her
cycles. In the last year, she has had about 5 cycles in total; her last menstrual period was 6 weeks
ago. She is otherwise in good health and has not had any surgeries. She has no history of abnormal
Pap smears or sexually transmitted infections. She is 176cm and weighs 90kg. On general
appearance, she seems to be hirsute on the face and the abdomen. The rest of her exam is otherwise
normal. Which of the following is most likely to help identify the underlying cause of this woman’s
infertility?
A. Lutenizing hormone levels
B. * Testosterone levels
C. Follicle stimulating hormone levels
D. Thyroid function tests
E. Progesterone levels
8. A 34 year-old woman comes to you for a chief complaint of hirsutism. She states that this has been
present since menarche, but has gotten worse in the past two years. Her menses have become more
irregular, now every 28-45 days apart. She states that she quit smoking and gained approximately
thirty pounds in the past three years. Her mother is obese, diabetic and has hirsutism. There is no
hair seen on her chin, but she shaves every few days. TSH, prolactin, 17-hydroxyprogesterone and
DHEAS are normal. Testosterone is mildly elevated. Which of the following is the most likely
etiology of her hirsutism?
A. * Polycystic ovarian syndrome
B. Ovarian neoplasm
C. Diabetes
D. Cushing’s syndrome
E. Adrenal tumor
9. A 52 year-old G3P3 woman presents to your office with severe hot flashes and vaginal dryness for
6 months. Her last menstrual period was 15 months ago. After discussing the risks and benefits of
hormone therapy with this patient, she decides to begin treatment. This patient is most likely to stop
hormone therapy secondary to what side effect?
A. * Vaginal bleeding
B. Development of breast cancer
C. Hirsutism
D. Nausea
E. Relief of menopausal symptoms
10. A 22 year-old G0 college student returns for follow-up of mood swings and difficulty concentrating
on her schoolwork the week before her menses for the past 12 months. Her past medical history is
unremarkable and physical examination is normal. Which of the following would be an appropriate
treatment option for this patient?
A. * Oral contraceptive pills
B. Reassurance and observation
C. Methylphenidate (Ritalin
D. Gabapentin
E. Ginkgo
11. A 16 year-old girl comes to the doctor to discuss contraception. She recently became sexually
active and states she has never had a menstrual cycle. She regularly attends school and participates
in the band. On physical examination, she is 175cm and weighs 84kg. She has no secondary sexual
characteristics with normal appearing external genitalia. The physician suspects Kallmann
syndrome. Which of the following diagnostic tests will help confirm the diagnosis?
A. An MRI of the pituitary
B. * Olfactory challenge
C. Measurement of testosterone levels
D. Pelvic ultrasound
E. Cortisol levels
12. An 18 year-old G0 comes in for an annual exam with her mother. Her mother comments that she
had severe PMS symptoms in her twenties and thirties. She would like to know if her daughter will
inherit this as well. Which of the following has the strongest association with premenstrual
syndrome?
A. Obesity
B. * Positive family history
C. Insulin dependent diabetes mellitus
D. History of early menarche
E. Vitamin K deficiency
13. A 28 year-old G1P1 presents to your office. She delivered 4 days ago and tearfully reports that
yesterday and the day before she had trouble sleeping, felt anxious and was irritable. She feels
somewhat better today, but is still concerned. What is the most likely diagnosis?
A. Hypothyroidism
B. * Blues
C. Depression
D. Normal postpartum state
E. Anxiety
14. A 24 year-old woman comes to her physician for help with her premenstrual syndrome symptoms.
She complains of “not being herself” for 3 to 4 days before her period and has episodes of crying
and irritability. She denies depressive symptoms and notes she is a stay-at-home mother for her
three children. After a complete history and physical examination, the patient is prescribed a
selective serotonin reuptake inhibitor but, after three months, she returns as there is no change in
her symptoms. Upon further discussion, the patient admits that her husband has a bad temper at
times. Physical examination is normal with the exception of some bruising on the patient’s arms.
She claims that she fell and that she is often clumsy. Which of the following is the most appropriate
next step in the management of this patient?
A. Report the injuries to the police
B. * Offer domestic violence resources to the patient
C. Refer the patient for a psychiatric consultation
D. Arrange for a home visit by a women’s shelter counselor
E. Arrange for security to escort her to a women’s shelter

15. A mother brings her 16 year-old daughter to the doctor because she has not begun menses and has
not kept up in growth with her friends. She performs well in school and participates in
extracurricular activities. On physical examination, she is171cm tall, 78kg and has Tanner stage 1
breast and pubic hair growth. Her breast nipples are widely spaced with a shield chest and neck is
thickened. No genital tract abnormalities are noted on exam. Which of the following is the most
likely cause of her delayed sexual maturation?
A. Partial deletion of the long arm of the X chromosome
B. Mullerian agenesis
C. Down Syndrome
D. * Turner syndrome
E. Rokitansky-Kuster-Hauser Syndrome
16. A 58 year-old G3P3 woman has been postmenopausal for 5 years and is concerned about
osteoporosis. She has declined hormone therapy in the past. Her mother has a history of a hip
fracture at age 82. A physical exam is unremarkable. In addition to weight bearing exercise and
vitamin D supplementation, what optimal daily calcium intake should she take?
A. 500 mg
B. 750 mg
C. 1000 mg
D. * 1200 mg
E. 2000 mg
17. A 21 year-old woman comes to the office because of acne, irregular menses and hirsutism. She
initially was evaluated 6 months ago. At that time, she was diagnosed with idiopathic hirsutism.
She was started on oral contraceptive pills to improve her symptoms. Menstrual periods now occur
every month, but her hirsutism has not significantly improved. In addition to the oral
contraceptives, which of the following would be an appropriate treatment for hirsutism?
A. * Spironolactone
B. Lupron
C. Danazol
D. Depo-Provera
E. Steroids
18. Postmenopausal woman should be routinely screened after age 65 for which of the following:
A. Cirrhosis
B. Ovarian cancer
C. Small cell carcinoma
D. * Osteoporosis
E. Dementia
19. A 56 year-old woman presents with complains of insomnia, vaginal dryness and amenorrhea for 13
months. What is the most likely diagnose?
A. Sheehan syndrome
B. * Menopause
C. Hypothyroidism
D. Hypethyroidism
E. Ovarian cancer
20. Terry, age 27, comes to the family planning clinic for contraceptive advice. Her history reveals
frequent urinary tract infections. Terry inquires about the symptothermal contraceptive method. In
this method, the client notes which of the following signs to help detect the fertile period?
A. breast enlargement and tenderness
B. * clear, thin, elastic cervical mucus
C. decreased libido

D. weight gain of 2 to 3 pounds


E. irregular period
21. Betty, age 24, has been using an oral contraceptive for the past 5 years. Now she wants to become
pregnant. What advice should the physician give Betty?
A. Attempt to conceive as soon as possible after discontinuing the oral contraceptive.
B. Discontinue the oral contraceptive and wait 1 month before attempting to conceive.
C. * Discontinue the oral contraceptive and wait 3 months before attempting to conceive.
D. Attempt to conceive after a physical assessment confirms a return to normal hormone levels.
E. You should visit to reproductologist for consultation.
22. Two days ago Mary had intercourse during which her partner's condom broke. She has come to the
clinic today for a morning-after contraceptive. Which of the following statements accurately
describes this form of contraception?
A. The morning-after contraceptive must be used within 24 hours of unprotected intercourse.
B. * The morning-after contraceptive may cause mild nausea for 1 to 2 days.
C. The morning-after contraceptive creates a local, sterile inflammatory reaction in the uterus.
D. The morning-after contraceptive has a relatively low effectiveness rate.
E. The morning-after contraceptive is not used after broking of condom.
23. Maria, age 26, married, calls the clinic because she has forgotten to take her oral contraceptive for
the past 2 days. What advice should the physician give to Maria?
A. Discard the rest of the pack and use another contraceptive method for the rest of the cycle.
B. * Take two pills for the next 2 days and use another contraceptive method for the rest of the cycle.
C. Continue to take the remaining pills for the rest of the cycle.
D. Take two pills immediately and then take one pill daily for the rest of the cycle.
E. IUD should be placed during next 5 days
24. The physician is teaching Sarah unmarried, age 24, about diaphragm use and care. Which of the
following statements should be part of this teaching session?
A. Use only petroleum jelly for vaginal lubrication, if needed.
B. Check the diaphragm for tears or holes after each use.
C. Have the diaphragm refitted after a loss or gain of 10 pounds or more.
D. * Store the diaphragm in its container in a cool, dry place.
E. Continue to use diaphragm during the period
25. Maria, age 18, has become pregnant as a result of a rape 10 weeks ago. She asks the physician
about pregnancy interruption. The physician describes various procedures, including vacuum
curettage and dilatation and curettage (D & C). How do they compare?
A. D & C requires uterus scraping with a metal curette; vacuum curettage does not
B. D & C is performed more frequently than vacuum curettage.
C. D & C uses crushing instruments; vacuum curettage uses suction equipment
D. * D & C carries a higher risk of complications than vacuum curettage.
E. D & C carries a lower risk of complications than vacuum curettage
26. Donna, age 23, has become pregnant as a result of a rape 7 weeks ago. She asks the physician about
pregnancy interruption. The physician describes various procedures, including vacuum curettage
and dilatation and curettage (D & C). If Donna postpones the decision until she is 10 weeks
pregnant, which method of pregnancy interruption is most likely to be used?
A. dilatation and curettage
B. dilatation and evacuation
C. * prostaglandin suppository
D. RU-4S6 administration
E. cesarean section

27. Liz, age 28, has become pregnant as a result of a rape 10 weeks ago. She asks the physician about
pregnancy interruption. The physician describes various procedures, including vacuum curettage
and dilatation and curettage (D & C). After Liz undergoes a first-trimester pregnancy interruption,
the physician provides postoperative care. Which of the following statements should be part of the
postoperative teaching session?
A. * Wipe the perineum from front to back.
B. Douche once a week until the follow-up visit
C. Avoid sexual intercourse for 3 to 4 weeks.
D. Expect the next menstrual period in about 3 weeks.
E. Intrauterine Device (IUD) is a method of choice for you
28. While pregnant with her fourth child, Anita discusses sterilization with her husband John. They ask
the physician to tell them about tubal ligation and its effects. How does this procedure terminate
fertility?
A. It prevents ovulation and menstruation.
B. It removes the entire uterus.
C. It occludes the cervix.
D. * It blocks ova from the fallopian tubes.
E. Suppress ovulation
29. While pregnant with her fourth child, Olivia discusses sterilization with her husband Mike. They
ask the physician to tell them about tubal ligation and its effects. Olivia decides to undergo
sterilization after delivery while she is still in the hospital. Which procedure is most appropriate for
A. * her?
minilaparotomy
B. laparoscopy
C. hysteroscopy
D. hysterectomy
E. colposcopy
30. Rick is scheduled for a vasectomy on Friday. His partner Anne asks the physician when they can
begin having unprotected intercourse. What is the physician's best response?
A. when desired because sterilization is immediate
B. as soon as scrotal edema and tenderness disappear
C. * when the sperm count reflects sterilization
D. after about 6 to 10 ejaculations
E. after one month
31. Rita is scheduled for a tubal ligation on Wednesday. Her partner John asks the physician when they
can begin having unprotected intercourse. What is the physician's best response?
A. * when desired because sterilization is immediate
B. as soon as wound edema and tenderness disappear
C. after next menstrual period
D. 6 month later
E. you should use barrier method at least 2 weeks
32. A 26-year-old woman is scheduled for a first trimester abortion in the morning. A laminaria tent is
inserted as part of the vacuum aspiration procedure. The physician explains to the woman that
laminaria is used to:
A. Stimulate the uterus to contract.
B. Prevent postabortion infection.
C. Reduce pain by numbing the cervix.
D. * Dilate the cervix for easier insertion of the aspirator.
E. Prevent postabortion hemorrhage

33. A married 23 years old woman visit to family planning center for consultation. She uses the basal
body temperature method of contraception. She describes her sexual life as mogamous with
husband. She wants to use Basal Body Temperature as family planning method. She asks physician
what does the Basal Body Temperature (BBT) chart tell about.
A. Problems with ovulation
B. * Time of ovulation
C. Time of next ovulation
D. None of these
E. Time of previous ovulation
34. A married 23 years old woman visit to family planning center for contraceptive advice. She
describes her sex life as “active” with “many” partners. She wants a contraceptive method that is
reliable and does not interfere with sex. Which method of contraception will be most appropriate
A. for this client:
Vaginal rings
B. * Condoms
C. Patches
D. Progestin only pill
E. Intrauterine device
35. Mona, is a 23-year-old healthy, non-smoking woman without medical complications. She has no
contraindications for the use of oral contraceptives. When during her cycle should BB begin her
oral contraceptives?
A. * Take the first tablet on the first day of menses; this avoids the risk of early ovulation and the need
to use alternative methods of contraception
B. Take the first tablet on the first Sunday after the beginning of menstruation and use alternative
method of contraception for the first 14 days.
C. Take the first tablet on the first day of menses; and use alternative method of contraception for the
first 14 days
D. Take the first tablet on the first Sunday after the beginning of menstruation this avoids the risk of
early ovulation and the need to use alternative methods of contraception
E. Take the first tablet days after visit; and use alternative method of contraception for the next cycle
36. Nadia is a 30-year-old mother of three children. She has been exclusively breastfeeding her
youngest child, who is 5 months old. Nadia is planning to begin supplementing her infant’s diet and
weaning her baby next month. Today, she has come to clinic to make choice about family planning
method. Which method of contraception will be most available for Nadia?
A. Lactational amenorrhea
B. * Progestin only pill
C. Sterilization
D. Calendar method
E. Basal body temperature method
37. Nelly is a 26-year-old mother of two children. She has been exclusively breastfeeding her youngest
child, who is 5,5 months old. Nelly is planning to begin supplementing her infant’s diet and
weaning her baby next month. Today, she has come to clinic to make choice about family planning
method. She wants to use combine oral contraceptives. What is the mechanism of action of this
A. method?
make the cervical mucus thin
B. * prevent the release of eggs from the ovaries
C. stimulate peristaltic waves of fallopian tube
D. destroy ovum
E. decrease speed of sperm movement

38. Ashley, a 23-year-old unmarried, comes for an initial visit to request birth control. She asks about
birth control pills, but also wants information about the newer hormonal methods on the market.
Vital signs: 1,68 m tall, 95 kg., BMI 33.9, Ps 76, RR 16, T 36,6.7 C, BP 142/88 mm Hg. What
information do you NOT need prior to starting Ashley on hormonal contraception?
A. Blood pressure
B. * Smoking history
C. Pap smear and pelvic exam
D. History of migraines with auras
E. Family history of thrombotic disorders
39. Olga, a 33-year-old unmarried, comes for an initial visit to request birth control. She asks about
birth control pills, but also wants information about the newer hormonal methods on the market.
Vital signs: 1,68 m tall, 95 kg., BMI 33.9, Ps 76, RR 16, T 36,6.7 C, BP 142/88 mm Hg. Olga
should watch for all of the following side effects with an OCP EXCEPT
A. headache
B. nausea
C. spotting
D. breast tenderness
E. * amenorrhea
40. Kanisha, a 35-year-old female comes to your clinic to ask about contraception. She and her
husband have one child. She wants some type of birth control that she doesn't have to think about
for as long as possible. She is otherwise healthy, and her only complaint is heavy periods. She is
currently using the “rhythm method” and adding condoms during high-risk times. Which of the
following contraceptives would you recommend?
A. Tubal ligation
B. Basal body temperature method
C. Spermicides
D. IUD (Intrauterine Device)
E. * Progestin only pill
41. When performing a clinical breast examination on a client, a physician palpates a thickened area
where the skin folds under the breast. What is the physician’s best action?
A. * Proceed with the examination.
B. Determine if the thickness is bilateral.
C. Ask the client how long the thickness has been present.
D. Attempt to elicit the same response with the client in a different position.
E. Schedule for MRI
42. The client who has discovered a lump in one breast is very upset when calling to schedule a
mammogram. What is the physician’s best response?
A. “It is a good thing you called. All lumps are considered cancerous until proven otherwise.”
B. “Unless you have a relative with breast cancer, this lump is most likely benign.”
C. “Diagnosing cancer at this early stage is most likely to result in a cure.”
D. * “Many women have breast lumps, and 90% of the lumps are benign.”
E. “Routine mammogram allows you rule out cancer”
43. During the visit for a school physical, the 13-year old girl being examines questions the asymmetry
of her breasts. The best response is
A. * One breast may grow faster than the other during development
B. I will give you a referral for a mammogram
C. You will probably have fibrocystic disease when you are older
D. This may be an indication of hormonal imbalance. We will check again in 6 month

E. It is first symptom of cancer


44. This is the first visit for a woman, age 38. The physician instructs her that a baseline mammogram
is recommended for woman between the age of 35 and 39 and that the clinical exam schedule
would be based on age. The recommendation for women 40 to 49 is:
A. * every year
B. every 2 years
C. twice a year
D. only the baseline exam is needed unless the woman has symptoms
E. woman of this age have no any need in mammogram examination
45. The client is 45 years old and has just been diagnosed with fibrocystic breast disease. She asks
what this means in terms of her health. What is the physician’s best response?
A. “This problem greatly increases your risk for breast cancer, so be sure to schedule yearly
mammograms.”
B. “This problem progressively increases as you age, especially if you have never been pregnant.”
C. * “This problem will probably diminish with menopause if you don't take replacement hormones.”
D. “This problem is genetic and you should teach your daughters about it.”

You might also like